Download as doc, pdf, or txt
Download as doc, pdf, or txt
You are on page 1of 85

Final Exam: Chapters 1-14 Name ______________________________________

ISV Managerial Accounting, 4e Instructor ___________________________________


Section # ______________ Date _______________

Part I II III IV V Total

Points 60 18 19 14 14 125

Score

PART I — MULTIPLE CHOICE (60 points)

Instructions: Designate the best answer for each of the following questions.

____ 1. A responsibility center that incurs costs (and expenses) and generates revenues is classified as a(n)
a. cost center.
b. revenue center.
c. profit center.
d. investment center.

____ 2. The most useful measure for evaluating a manager's performance in controlling revenues and costs in a profit center is
a. contribution margin.
b. contribution net income.
c. contribution gross profit.
d. controllable margin.

____ 3. Ramsey Corporation desires to earn target net income of $90,000. If the selling price per unit is $30, unit variable cost is
$24, and total fixed costs are $360,000, the number of units that the company must sell to earn its target net income is
a. 30,000.
b. 75,000.
c. 45,000.
d. 60,000.

____ 4. Shane Corporation uses a process cost accounting system. Given the following data, compute the number of units
transferred out during the current period.
Beginning Work in Process 20,000 units (1/2 complete)
Ending Work in Process 25,000 units (1/3 complete)
Started into Production 150,000 units
a. 125,000
b. 141,667
c. 145,000
d. 150,000
____ 5. Witten Company applies overhead on the basis of machine hours. Given the following data, compute overhead applied
and the under- or overapplication of overhead for the period:
Estimated annual overhead cost $1,200,000
Actual annual overhead cost $1,150,000
Estimated machine hours 300,000
Actual machine hours 280,000
a. $1,120,000 applied and $30,000 underapplied
b. $1,200,000 applied and $30,000 overapplied
c. $1,120,000 applied and $30,000 overapplied
d. $1,150,000 applied and neither under- nor overapplied

____ 6. The following data has been collected for use in analyzing the behavior of main-tenance costs of Ridell Corporation:
Month Maintenance Costs Machine Hours
January $121,000 20,000
February 125,000 23,000
March 128,000 24,000
April 159,000 34,000
May 168,000 36,000
June 178,000 38,000
July 181,000 40,000
Using the high-low method to separate the maintenance costs into their variable and fixed cost components, these
components are
a. $5 per hour plus $20,000.
b. $5 per hour plus $30,000.
c. $4 per hour plus $41,000.
d. $3 per hour plus $61,000.

____ 7. Given the following information for Hett Company, compute the company's ROI: Sales — $1,000,000; Controllable
Margin — $120,000; Average Operating Assets — $500,000.
a. 40%
b. 50%
c. 12%
d. 24%

____ 8. Given the following data for Glennon Company, compute (A) total manufacturing costs and (B) costs of goods
manufactured:
Direct materials used $120,000 Beginning work in process $20,000
Direct labor 50,000 Ending work in process 10,000
Manufacturing overhead 150,000 Beginning finished goods 25,000
Operating expenses 175,000 Ending finished goods 15,000
(A) (B)
a. $310,000 $330,000
b. $320,000 $310,000
c. $320,000 $330,000
d. $330,000 $340,000
____ 9. The production cost report shows both quantities and costs. Costs are reported in three sections: (1) costs accounted for,
(2) unit costs, and (3) costs charged to department. The sections are listed in the following order:
a. (1), (2), (3).
b. (1), (3), (2).
c. (2), (1), (3).
d. (2), (3), (1).

____ 10. The starting point of a master budget is the preparation of the
a. cash budget.
b. sales budget.
c. production budget.
d. budgeted balance sheet.

____ 11. The most useful measure for evaluating the performance of the manager of an investment center is
a. contribution margin.
b. controllable margin.
c. return on investment.
d. income from operations.

____ 12. Which of the following capital budgeting techniques explicitly takes the time value of money into consideration?
a. Annual rate of return
b. Internal rate of return
c. Net present value
d. Both (b) and (c) above

____ 13. The cost classification scheme most relevant to responsibility accounting is
a. controllable vs. uncontrollable.
b. fixed vs. variable.
c. semivariable vs. mixed.
d. direct vs. indirect.

Use the following information for questions 14 and 15.


Grant Company estimates its sales at 60,000 units in the first quarter and that sales will increase by 6,000 units each quarter over the
year. It has, and desires, a 25% ending inventory of finished goods. Each unit sells for $25. 40% of the sales are for cash. 70% of the
credit customers pay within the quarter. The remainder is received in the quarter following sale.

____ 14. Cash collections for the third quarter are budgeted at
a. $1,017,000.
b. $1,476,000.
c. $1,773,000.
d. $2,052,000.

____ 15. Production in units for the third quarter should be budgeted at
a. 73,500.
b. 69,000.
c. 91,500.
d. 72,000.
____ 16. Stine Company incurs the following costs in producing 50,000 units of product:
Direct materials $100,000
Direct labor 50,000
Variable manufacturing overhead 100,000
Fixed manufacturing overhead 300,000
An outside supplier has offered to supply the 50,000 units at $7.00 each. All of Stine's related variable costs, but only
$200,000 of the fixed costs would be eliminated if the offer is accepted. Acceptance will result in a
a. savings of $200,000.
b. loss of $100,000.
c. savings of $100,000.
d. loss of $200,000.

____ 17. Finney Company has a production process where two products result from a joint processing procedure; both can be sold
immediately or processed further. Given the following additional per unit information, determine which of the products
should be processed further.
Allocated Additional New
Product Joint Cost Selling Price Processing Cost Selling Price
A $100 $200 $180 $400
B 60 100 50 160
a. A
b. B
c. Both
d. Neither

____ 18. A flexible budget


a. is also called a static budget.
b. can be considered a series of related static budgets.
c. can be prepared for sales or production budgets, but not for an operating expense budget.
d. typically uses an activity index different from that used in developing the predetermined overhead rate.

____ 19. Carey Company's equipment account increased $800,000 during the period; the related accumulated depreciation
increased $60,000. New equipment was purchased at a cost of $1,400,000 and used equipment was sold at a loss of
$40,000. Depreciation expense was $200,000. Proceeds from the sale of the used equipment were
a. $420,000.
b. $500,000.
c. $560,000.
d. $640,000.

____ 20. Which of the following combinations presents correct examples of liquidity, profitability, and solvency ratios, respectively?
Liquidity Profitability Solvency
a. Inventory turnover Inventory turnover Times interest earned
b. Current ratio Inventory turnover Debt to total assets
c. Receivables turnover Return on assets Times interest earned
d. Quick ratio Payout ratio Return on assets
____ 21. A company’s planned activity level for next year is expected to be 100,000 machine hours. At this level of activity, the
company budgeted the following manufacturing overhead costs:
Variable Fixed
Indirect materials $60,000 Depreciation $25,000
Indirect labor 80,000 Taxes 5,000
Factory supplies 10,000 Supervision 20,000
A flexible budget prepared at the 90,000 machine hours level of activity would allow total manufacturing overhead costs of
a. $135,000.
b. $180,000.
c. $185,000.
d. $150,000.

____ 22. A company developed the following per unit materials standards for its product: 3 gallons of direct materials at $5 per
gallon. If 4,000 units of product were produced last month and 12,500 gallons of direct materials were used, the direct
materials quantity variance was
a. $1,500 favorable.
b. $2,500 unfavorable.
c. $1,500 unfavorable.
d. $2,500 favorable.

____ 23. The standard direct labor cost for producing one unit of product is 5 direct labor hours at a standard rate of pay of $8. Last
month, 5,000 units were produced and 24,500 direct labor hours were actually worked at a total cost of $180,000. The
direct labor quantity variance was
a. $4,000 unfavorable.
b. $6,000 unfavorable.
c. $6,000 favorable.
d. $4,000 favorable.

____ *24. Smythe Company applies overhead to products based on direct labor hours. Manufacturing overhead at the expected
normal level of activity is $50,000 per month plus $5 per direct labor hour. During June, actual manufacturing overhead
costs amounted to $85,000 when 6,100 actual direct labor hours were worked. The standard number of direct labor hours
that should have been worked for the output achieved was 6,000 direct labor hours. The overhead controllable variance
for June was
a. $4,500 unfavorable.
b. $3,400 favorable.
c. $5,000 unfavorable.
d. $5,000 favorable.

____ 25. Under the time-and-material-pricing approach, the charges for any particular job include each of the following except the
a. labor charge.
b. charge for materials.
c. material loading charge.
d. overhead charge.

____ 26. The transfer pricing approach that does not reflect the selling division’s true profit-ability is the
a. cost-based approach.
b. market-based approach.
c. negotiated price approach.
d. time-and-material-pricing approach.

Use the following information for questions 27 and 28.

Robot Toy Company manufactures two products: X-O-Tron and Mechoman. Robot’s overhead costs consist of setting up machines,
$200,000; machining, $450,000; and inspecting, $150,000 Additional information on the two products is:
X-O-Tron Mechoman
Direct labor hours 15,000 25,000
Machine setups 600 400
Machine hours 24,000 26,000
Inspections 800 700

____ 27. Overhead applied to Mechoman using traditional costing is


a. $320,000.
b. $384,000.
c. $416,000.
d. $500,000.

____ 28. Overhead applied to X-O-Tron using activity-based costing is


a. $300,000.
b. $384,000.
c. $416,000.
d. $480,000.

____ 29. An appropriate cost driver for an assembling cost pool is the number of
a. purchase orders.
b. setups.
c. parts.
d. direct labor hours.

____ 30. Which of the following is included in the cost of goods manufactured under absorption costing but not under variable
costing?
a. Direct materials
b. Variable factory overhead
c. Fixed factory overhead
d. Direct labor

Instructions: Designate the terminology that best represents the definition or statement given below by placing the identifying letter(s)
in the space provided. No term should be used more than once.
A. Activity-based costing N. Job cost sheet
B. Annual rate of return O. Noncontrollable costs
C. Budgetary control P. Non-value-added activity
D. Contribution margin Q. Operating budgets
E. Contribution margin ratio *R. Overhead controllable variance
F. Controllable costs *S. Overhead volume variance
G. Absorption costing T. Physical units
H. Cost accounting U. Process cost systems
I. Cost centers V. Product costs
J. Cost of capital W. Profit center
K. Equivalent units of production X. Value-added activity
L. Fixed costs Y. Variable costs
M. Free cash flow Z. Variances

____ 1. Costs that a manager has the authority to incur within a given period of time.
____ 2. A form used to record the costs chargeable to a job.
____ 3. A responsibility center that incurs costs and also generates revenues.
____ 4. The difference between overhead budgeted for standard hours allowed and overhead incurred.
____ 5. The amount of revenue remaining after deducting variable costs.
____ 6. Used to apply costs to similar products that are mass produced in a continuous fashion.
____ 7. Costs that vary in total directly and proportionately with changes in the activity level.
____ 8. The differences between actual costs and standard costs.
____ 9. Determines profitability of a capital expenditure by dividing expected net income by the average investment.
____ 10. The rate a company must pay to obtain funds from creditors and stockholders.
____ 11. Costs that are an integral part of producing the finished product.
____ 12. Allocates overhead to multiple cost pools and assigns the cost pools to products by means of cost drivers.
____ 13. Involves the measuring, recording, and reporting of product costs.
____ 14. A measure of the work done during the period, expressed in fully completed units.
____ 15. A costing approach in which all manufacturing costs are charged to the product.
____ 16. Increase the worth of a product or service to customers.
____ 17. The amount of cash from operations after deducting capital expenditures and cash dividends paid.
____ 18. Individual budgets that culminate in a budgeted income statement.
The Olson Company developed the following standard costs for its product in 2008:
Standard Cost Card Unit Standard Cost
Direct materials (5 pounds @ $4 per pound) $20
Direct labor (4 hours @ $8 per hour) 32
Manufacturing overhead
Variable (4 hours @ $4 per hour) 16
Fixed (4 hours @ $3 per hour) 12
$80

The company planned to work 100,000 direct labor hours and produce 25,000 units of product in 2008. Actual results for 2008 are as
follows:

 24,000 units of product were produced.


 Actual direct materials purchased and used during the year amounted to 122,000 pounds at a cost of $475,800.
 Actual direct labor costs were $779,000 for 95,000 direct labor hours worked.
 Total actual manufacturing overhead incurred amounted to $685,500.

Instructions
Calculate the following variances showing all computations supporting your answers. Indicate if the variances are favorable (F) or
unfavorable (U).
(a) Direct materials price and direct materials quantity variances.
(b) Direct labor price and direct labor quantity variances.
*(c) Overhead controllable and overhead volume variances.

PART IV — RATIO ANALYSIS (14 points)


The condensed financial statements of Jenner Corporation for 2008 are presented below.

Jenner Corporation Jenner Corporation


Balance Sheet Income Statement
December 31, 2008 For the Year Ended December 31, 2008

Assets Revenues $2,000,000


Current assets Expenses
Cash and short-term Cost of goods sold 960,000
investments $ 30,000 Selling and administrative
Accounts receivable 70,000 expenses 740,000
Inventories 140,000 Interest expense 50,000
Total current assets 240,000 Total expenses 1,750,000
Property, plant, and Income before income taxes 250,000
equipment (net) 760,000 Income tax expense 100,000
Total assets $1,000,000 Net income $ 150,000

Liabilities and Stockholders' Equity


Current liabilities $ 100,000
Long-term liabilities 350,000
Stockholders' equity 550,000
Total liabilities and
stockholders' equity $1,000,000

Additional data as of December 31, 2007: Inventory = $100,000; Total assets = $800,000; Stockholders' equity = $450,000.

Instructions: Compute the following ratios for 2008 showing supporting calculations.

(a) Current ratio = __________________________________________________________________________.

(b) Debt to total assets ratio = _________________________________________________________________.

(c) Times interest earned = ___________________________________________________________________.

(d) Inventory turnover = _____________________________________________________________________.

(e) Profit margin = __________________________________________________________________________.

(f) Return on stockholders' equity = ____________________________________________________________.


(g) Return on assets = ______________________________________________________________________.

PART V — MISCELLANEOUS MANAGERIAL MINI-PROBLEMS (14 points)


Carson Corporation manufactures paper shredding equipment. You are requested to "audit" a sampling of computations made by
Carson's internal accountants via your independent recalculation of the information.

Instructions: Compute the requested information for each of the following independent situations (present supporting calculations).

(a) Carson uses a process costing system. 2,000 units were in process at the beginning of the period, 60% complete. 20,000 units
were started into production during the period; 1,000 were in process at the end of the period, 60% complete. Compute equivalent
units for conversion costs.

(b) Carson sells each unit for $500. Variable costs per unit equal $300. Total fixed costs equal $800,000. Carson is currently selling
5,000 units per period and would like to earn net income of $400,000. Compute: (1) break-even point in dollars; (2) sales units
necessary to attain desired income; and (3) margin of safety ratio for current operations.

(1) Break-even point = $________________________________________________.

(2) Desired sales = ___________________________________________ units.

(3) Margin of safety = _____________________________________________%.

Solutions — Final Exam: Chapters 1-14

PART I — MULTIPLE CHOICE (60 points)


1. c 7. d 13. a 19. a 25. d
2. d 8. c 14. c 20. c 26. a
3. b 9. d 15. a 21. c 27. d
4. c 10. b 16. c 22. b 28. c
5. a 11. c 17. c 23. d 29. c
6. d 12. d 18. b *24. c 30. c

PART II — MATCHING (18 points)


1. F 6. U 11. V 16. X
2. N 7. Y 12. A 17. M
3. W 8. Z 13. H 18. Q
*4. R 9. B 14. K
5. D 10. J 15. G

PART III — VARIANCE ANALYSIS (19 points)


(a) Direct materials price and direct materials quantity variances.
Direct materials price variance
(122,000 × $3.90) – (122,000 × $4.00) = MPV
$475,800 – $488,000 = $12,200 F

Direct materials quantity variance


(122,000 × $4.00) – (120,000 × $4.00) = MQV
$488,000 – $480,000 = $8,000 U

(b) Direct labor price and direct labor quantity variances.


Direct labor price variance
(Actual Hours × Actual Rate) – (Actual Hours × Standard Rate) = LPV
(95,000 × $8.20) – (95,000 × $8) = LPV
$779,000 – $760,000 = $19,000 U

Direct labor quantity variance


(Actual Hours × Standard Rate) – (Standard Hours × Standard Rate) = LQV
(95,000 × $8) – (96,000 × $8) = LQV
$760,000 – $768,000 = $8,000 F

*(c) Overhead controllable variance


Actual overhead $685,000
Budgeted overhead — 24,000 × 4 = 96,000
Variable × $4
$384,000
Fixed — 100,000 × $3 = 300,000 684,000
$ 1,500 U

Overhead volume variance


Budgeted overhead (see above) $684,000
Overhead applied (96,000 × $7) 672,000
$ 12,000 U
PART IV — RATIO ANALYSIS (14 points)
(a) $240,000
Current ratio = ———— = 2.40:1.
$100,000

(b) $450,000
Debt to total assets ratio = ————— = 45%.
$1,000,000

(c) $300,000
Times interest earned = ———— = 6 times.
$50,000

(d) $960,000
Inventory turnover = ————— = 8 times.
$120,000

(e) $150,000
Profit margin = ————— = 7.5%.
$2,000,000

(f) $150,000
Return on stockholders' equity = ———— = 30%.
$500,000

(g) $150,000
Return on assets = ———— = 16.7%.
$900,000

PART V — MISCELLANEOUS MANAGERIAL MINI-PROBLEMS (14 points)


(a) Units transferred out (20,000 + 2,000 – 1,000)............................................................................ 21,000
Ending work in process (1,000 × 60%)........................................................................................ 600
Equivalent units for conversion costs................................................................................. 21,600

PART II — MATCHING (18 points)


PART III — VARIANCE ANALYSIS (19 points)
(b) $800,000
(1) Break-even point = ———— = $2,000,000.
.4

$800,000 + $400,000
(2) Desired sales = —————————— = 6,000 units.
200

$500,000
(3) Margin of safety = —————— = 20%.
$2,500,000
CHAPTER 9: RESPONSIBILITY ACCOUNTING

Multiple Choice
b 8. A balanced scorecard approach to performance
c 1. Goal congruence exists when measurement
a. the goals of the company harmonize with each other. a. can only be used in profit or investment centers.
b. the company's managers are pursuing their own goals b. balances financial measures with nonfinancial
effectively. measures.
c. the company's managers are pursuing the goals of the c. uses only qualitative data to evaluate performance.
company. d. uses budgeted data rather than historical data.
d. all of the above are true.

c 2. Goal congruence is most likely to result when b 9. If a company has a favorable sales volume variance, its
a. reports to managers include all costs. a. sales price variance is also favorable.
b. managers' behavior is affected by the criteria used to b. total contribution margin might be less than planned.
judge their performances. c. total contribution margin will be more than planned.
c. performance evaluation criteria encourage behavior in d. income will be positive.
the company's best interests as well as in the
manager's best interests. c 10. Transfer prices
d. a manager knows the criteria used to judge his or her a. reduce employee turnover.
performance. b. are necessary for investment centers.
c. should encourage the kinds of behavior that upper-level
d 3. In responsibility accounting the most relevant management wants.
classification of costs is d. are not used for departments with high amounts of fixed
a. fixed and variable. costs.
b. incremental and nonincremental.
c. discretionary and committed. b 11. A transfer price is
d. controllable and noncontrollable. a. an accounting device to turn profit centers into
investment centers.
c 4. Which of the following is critically important for a b. the price charged by one segment of the company for
responsibility accounting system to be effective? goods or services provided to another segment.
a. Each employee should receive a separate performance c. only useful in a segment that deals with outsiders as well
report. as with other segments of the same company.
b. Service department costs should be allocated to the d. the amount charged by a cost center for a service
operating departments that use the service. performed for a profit center.
c. Each manager should know the criteria used for
evaluating his or her performance. c 12. The cost allocation policy most likely to encourage use of
d. The details on the performance reports for individual a service is based on
managers should add up to the totals on the a. budgeted total costs of the service department.
report to their supervisor. b. actual total costs of the service department.
c. budgeted variable costs for the service department.
c 5. Which of the following items is LEAST likely to appear on d. actual variable costs for the service department.
the performance report of the manager of a product
line? c 13. Which of the following statements is true?
a. Variable manufacturing costs for products in the line. a. A company changes its total income when it changes the
b. Selling expenses for the line. bases used to allocate indirect costs.
c. A share of company-wide advertising. b. A company should select an allocation basis so as to
d. Revenues from the line. raise or lower reported income on given products.
c. A company's total income will remain unchanged no
b 6. The sequence that reflects increasing breadth of matter how indirect costs are allocated.
responsibility is d. Costs should be allocated on an "ability-to-bear" basis.
a. cost center, investment center, profit center.
b. cost center, profit center, investment center. a 14. If a company allocates costs of a service department to
c. profit center, cost center, investment center. other departments, it should
d. investment center, cost center, profit center. a. consider the likely effects of the allocations on the use of
the services.
a 7. The criteria used for evaluating performance b. use the method that best reflects the relative sizes of the
a. should be designed to help achieve goal congruence. departments.
b. can be used only with profit centers and investment c. turn the service department into an investment center.
centers. d. allocate only the fixed costs of the service department.
c. should be used to compare past performance with
current performance. a 15. If a computer department does work for other
d. motivate people to work in the company's best interests. departments, charging a flat price per hour, the
computer department is d 23. Which of the following is NOT a good reason for
a. an artificial profit center. allocating indirect costs to operating departments?
b. a cost center. a. To remind managers of the need to cover indirect costs.
c. an investment center. b. So that operating managers will encourage service
d. none of the above. department managers to keep costs down.
c. To encourage managers to use services wisely.
a 16. The WORST method of allocating service department d. To determine the true costs of operating departments.
costs is
a. to allocate total actual costs based on actual use of the b 24. An artificial profit center
service. a. has no investment.
b. to allocate total budgeted costs based on long-term b. does not provide its goods or services outside the entity.
expected use of the service. c. cannot control its costs.
c. to allocate total budgeted costs based on actual use of d. could not be operated as a cost center.
the service.
d. none of the above, because all the above are equally c 25. A responsibility center is
undesirable. a. any department.
b. any manager.
b 17. As a general rule, the best transfer price to use to c. any area of activity for which a manager is responsible.
transfer the costs of a service center to an operating d. only large departments.
department is
a. the price charged by an outside company for the same a 26. ABC's actual selling price was less than planned and
service. actual unit volume more than planned. Therefore,
b. the price that encourages goal congruence. a. ABC had a favorable sales volume variance.
c. one that is based on budgeted variable cost. b. ABC's total contribution margin was more than planned.
d. one that is based on budgeted total cost. c. ABC had a favorable sales price variance.
d. ABC's actual total sales equaled planned total sales.
b 18. Which of the following costs is LEAST likely to appear on
the performance report for the foreman of a b 27. The term "dual rates" refers to
production department? a. allocating costs to several operating departments.
a. Wages of direct laborers. b. allocating fixed costs based on capacity requirements
b. Rent on machinery used in department. and variable costs based on use.
c. Repairs to machinery used in department. c. allocating both actual costs and budgeted costs.
d. Cost of materials used. d. using the budgeted rate to allocate some costs, the
actual rate to allocate others.
d 19. ABC Company operates a factory that makes
components for other ABC factories to assemble. The a 28. Which of the following methods of allocating the costs of
factory could be treated as service departments provides the broadest
a. a cost center. recognition of departments served?
b. an artificial profit center. a. Reciprocal allocation.
c. an investment center. b. Step-down allocation.
d. any of the above. c. Direct allocation.
d. Arbitrary allocation.
d 20. For reports to follow the principles of responsibility
accounting, which of the following must be true? d 29. Which of the following is a good reason for allocating
a. Each segment of the entity is an artificial profit center. indirect costs to operating departments?
b. The company is decentralized. a. The company could lose money if the operating
c. The company uses transfer prices. departments do not pay for the services they use.
d. The reports show controllable costs separately from b. To remind managers of the need to cover indirect costs.
noncontrollable costs. c. To encourage managers to use more services.
d. To determine the true costs of operating departments.
c 21. The effective use of responsibility accounting requires
that performance reports for cost centers b 30. When a manager takes an action that benefits his or her
a. show only variable costs. responsibility center, but not the company as a whole,
b. show a fair share of allocated costs. a. it is a non-controllable action.
c. distinguish between controllable and noncontrollable b. there is a lack of goal congruence.
costs. c. the center must be an artificial profit center.
d. show a fair share of revenues attributable to the center. d. the manager should be fired.

b 22. Criteria for evaluating performance should be carefully d 31. Which of the following is a good reason for NOT
selected because allocating indirect costs to operating departments?
a. they must be approved by the IRS. a. The company saves money if the operating departments
b. a manager's behavior can be affected by the criteria do not pay for the services they use.
used to judge his or her performance. b. To remind managers of the need to cover indirect costs.
c. managers may find out what they are. c. To encourage managers to use more services.
d. stockholders inquire about them at annual meetings. d. The costs are not controllable by the operating
departments.
controllable
d 32. Which of the following is a good reason for NOT a. varies from company to company.
allocating indirect costs to operating departments? b. depends on whether the cost is fixed or variable.
a. To remind managers that revenues must cover indirect c. depends on whether the cost is direct or indirect.
costs. d. is irrelevant to the preparation of performance reports.
b. To recognize that operating departments benefit from
the services. d 36. If at all possible, a manager's performance report should
c. To encourage managers to use services wisely. a. consider the results that the manager can control.
d. Because allocating them might prompt operating b. consider only the results that the manager can control.
managers to use nonincremental costs in making c. not be influenced by the results of decisions made by
decisions. other managers.
d. reflect all of the above characteristics.
b 33. A profit center is a responsibility center
a. that sells its output outside the company. d 37. Comparing budgeted and actual amounts is important in
b. whose manager is responsible for both revenues and evaluating the performance of
costs. a. the manager of a cost center.
c. that provides a service to other responsibility centers. b. the manager of a profit center.
d. within an investment center. c. the manager of an investment center.
d. any manager.
d 34. An investment center is
a. larger than a cost center. c 38. Direct, step-down, and reciprocal are names for
b. larger than a profit center. a. the allocation methods most likely to produce goal
c. seldom the responsibility of a single manager. congruence.
d. not truthfully characterized in any of the above b. transfer-pricing methods.
statements. c. methods for allocating costs of service departments to
operating departments.
a 35. The managerial level at which a particular cost is d. alternative organizational structures.

b 39. Cascade Company had the following results in June.

Planned Actual
------- -------
Sales $80,000 $78,900
Variable costs 50,000 48,500
------- -------
Contribution margin $30,000 $30,400
======= =======

Planned sales were 10,000 units; actual sales were 9,700 units. The sales price variance is
a. $1,100 U.
b. $1,000 F.
c. $900 U.
d. $400 F.

c 40. Cascade Company had the following results in June.

Planned Actual
------- -------
Sales $80,000 $78,900
Variable costs 50,000 48,500
------- -------
Contribution margin $30,000 $30,400
======= =======

Planned sales were 10,000 units, actual sales were 9,700 units. The sales volume variance is
a. $1,100 U.
b. $1,000 F.
c. $900 U.
d. $400 F.

b 41. Certainty Stores has three stores and one service center. The percentage of services used in the current year are Store X,
35%; Store Y, 40%; and Store Z, 25%. The service center costs were budgeted at $160,000 fixed and $240,000 variable.
Actual fixed costs were $140,000 and actual variable costs were $270,000. Actual service center costs are allocated to the
stores based on actual usage of the service center. Service center costs allocated to Store Y are
a. $64,000.
b. $164,000.
c. $410,000.
d. some other number.

c 42. Certainty Stores has three stores and one service center. The percentage of services used in the current year are Store X,
35%; Store Y, 40%; and Store Z, 25%. The service center costs were budgeted at $350,000 fixed and $250,000 variable.
Actual fixed costs were $370,000 and actual variable costs were $280,000. Budgeted service center costs are allocated to the
stores based on actual usage of the service center. Service center costs allocated to Store Y are
a. $140,000.
b. $148,000.
c. $240,000.
d. $260,000.

c 43. Wabasha Co. has two service departments (A and B) and two producing departments (X and Y). Data provided are as follows:

Service Depts. Operating Depts.


-------------- ---------------
A B X Y
------- ------ ------ ------
Direct costs $240 $400
Services performed by Dept. A 40% 40% 20%
Services performed by Dept. B. 20% 70% 10%

Wabasha uses the direct method to allocate service department costs. The service department cost allocated to Department Y
is
a. $88.
b. $96.
c. $130.
d. $240.
c 44. Wabasha Co. has two service departments (A and B) and two producing departments (X and Y). Data provided are as follows:

Service Depts. Operating Depts.


-------------- ---------------
A B X Y
------- ------ ------ ------
Direct costs $250 $400
Services performed by Dept. A 40% 40% 20%
Services performed by Dept. B. 20% 70% 10%

Wabasha uses the step-down method to allocate service department costs. Department A costs are allocated first. The service
department cost allocated to Department Y is
a. $90.
b. $97.50.
c. $112.50.
d. $130.

c 45. Wabasha Co. has two service departments (A and B) and two producing departments (X and Y). Data provided are as follows:

Service Depts. Operating Depts.


-------------- ---------------
A B X Y
------- ------ ------ ------
Direct costs $150 $300
Services performed by Dept. A 40% 40% 20%
Services performed by Dept. B. 20% 70% 10%

Wabasha uses the reciprocal method to allocate service department costs. The service department cost allocated to
Department Y is
a. $60.
b. $75.
c. $85.
d. $135.

d 46. Olson Stores has three stores and one service center. The percentage of services used in the current year are Store A, 40%;
Store B, 25%; and Store C, 45%. The expected long-term budgeted usages are Store A, 30%; Store B, 30%; and Store C,
40%. The service center costs were budgeted at $450,000 fixed and $550,000 variable. Actual fixed costs were $430,000 and
actual variable costs were $570,000. Olson allocates the budgeted variable costs of the central purchasing unit based on
actual use of the unit's services, and allocates budgeted fixed costs based on expected long-term use of the unit's services.
Service center costs allocated to Store A are
a. $135,000.
b. $220,000.
c. $300,000.
d. $355,000.

b 47. Olson Stores has three stores and one service center. The percentage of services used in the current year are Store A, 45%;
Store B, 35%; and Store C, 20%. The expected long-term budgeted usages are Store A, 30%; Store B, 40%; and Store C,
30%. The service center costs were budgeted at $450,000 fixed and $550,000 variable. Actual fixed costs were $430,000 and
actual variable costs were $570,000. Olson allocates the budgeted variable costs of the central purchasing unit based on
actual use of the unit's services, and allocates budgeted fixed costs based on expected long-term use of the unit's services.
Service center costs allocated to Store B are
a. $350,000.
b. $372,500.
c. $400,000.
d. $550,000.

d 48. Basin Co. has two service departments (A and B) and two producing departments (X and Y). Data provided are as follows:

Service Depts. Operating Depts.


A B X Y
------- ------ ------ ------
Direct costs $200 $400
Services performed by Dept. A 20% 40% 40%
Services performed by Dept. B. 30% 60% 10%

Basin uses the direct method to allocate service department costs. The service department cost allocated to Department X is
a. $280.
b. $300.
c. $320.
d. $443.

a 49. Basin Co. has two service departments (A and B) and two producing departments (X and Y). Data provided are as follows:

Service Depts. Operating Depts.


A B X Y
------- ------ ------ ------
Direct costs $200 $400
Services performed by Dept. A 20% 40% 40%
Services performed by Dept. B. 30% 60% 10%

Basin uses the step-down method to allocate service department costs. Department A costs are allocated first. The service
department cost allocated to Department X is
a. $457.
b. $443.
c. $320.
d. $300.

c 50. Basin Co. has two service departments (A and B) and two producing departments (X and Y). Data provided are as follows:

Service Depts. Operating Depts.


A B X Y
------- ------ ------ ------
Direct costs $200 $400
Services performed by Dept. A 20% 40% 40%
Services performed by Dept. B. 30% 60% 10%

Basin uses the reciprocal method to allocate service department costs. The service department cost allocated to Department
X is
a. $300.
b. $340.
c. $417.
d. $468.

True-False

F 1. All responsibility centers are either natural or artificial.

F 2. The sales volume variance is the difference between actual and planned unit sales multiplied by the actual contribution margin per
unit.

F 3. The principle of controllability is less important to the internal reporting for a centralized company than for a decentralized one.

F 4. Allocated costs are less important to the internal reporting for a centralized company than for a decentralized company.

F 5. Achieving goal congruence is less important in a centralized organization than in a decentralized one.

T 6. It is not always possible to separate the variable and fixed components of actual costs.

F 7. A profit center will always have sales to outside customers.

T 8. The sales price variance is the difference between the actual selling price and the planned selling price multiplied by actual units
sold.

T 9. The direct method of allocating service department costs ignores all of the interactions between service departments.

F 10. The reciprocal method of allocating service department costs considers only the usage by the producing departments in
determining the allocations.

Problems

1. The following data are for Billings Stores, which has two stores and one service center.
Helena Butte
------- -----
Percentage of services used in current year 20% 80%
Expected long-term use of services 30% 70%

Budgeted central purchasing costs were $225,000 fixed and $125,000 variable. Actual fixed costs were $240,000 and actual
variable costs were $115,000. The managers wish to allocate the actual central purchasing costs to the stores based on actual
use of the central purchasing service.

a. Compute the allocation to the Helena store.

b. Compute the allocation to the Butte store.

SOLUTION:

a. To Helena: $71,000 [20% x ($240,000 + $115,000)]

b. To Butte: $284,000 [80% x ($240,000 + $115,000)]

2. The following data are for Billings Stores, which has two stores and one service center.
Helena Butte
------- -----
Percentage of services used in current year 20% 80%
Expected long-term use of services 30% 70%
Budgeted central purchasing costs were $225,000 fixed and $125,000 variable. Actual fixed costs were $240,000 and actual
variable costs were $115,000. The company wishes to allocate the budgeted variable costs of the central purchasing unit based
on actual use of the unit's services and to allocate budgeted fixed costs based on expected long-term use of the unit's services.

a. Compute the total cost allocated to the Helena store for the services of the central purchasing unit.

b. Compute the total cost allocated to the Butte store for the services of the central purchasing unit.

SOLUTION:

a. To Helena: $92,500 ($125,000 x 20% + $225,000 x 30%)

b. To Butte: $257,500 ($125,000 x 80% + $225,000 x 70%)

3. Following are data about Alphabet Co.'s two service departments and two operating departments.
Service Depts. Operating Depts.
-------------- ---------------
A B X Y
------- ------ ------ ------
Direct costs $200 $500 $1,500 $2,000
Services performed by Dept. A 20% 40% 40%
Services performed by Dept. B. 10% 90% -

a. Alphabet allocates costs of its service departments using the direct method of allocation. Find the total cost that will be
allocated to Dept. X.

b. Alphabet allocates the costs of its service departments using the step-down method, beginning with Dept. A. Find the total
amount of cost that will be allocated to Dept. X.

SOLUTION:

a. Allocated to X: $600 [($200 x 40/(40 + 40)] + [$500 x (90/90)]

b. Allocated to X: $620
A B X Y
---- ---- ---- ----
A's direct cost $200
A's cost allocated (200) $ 40 $80 $80
B's direct cost 500
-----
Total for allocating $540
B's costs allocated (540) 540 0
---- ---
Allocated to X $620
Allocated to Y $80

4. Following are data about Alphabet Co.'s two service departments and two operating departments.
Service Depts. Operating Depts.
-------------- ---------------
A B X Y
------- ------ ------ ------
Direct costs $400 $1,000 $3,000 $4,000
Services performed by Dept. A 20% 40% 40%
Services performed by Dept. B. 10% 90% -

Alphabet allocates costs of its service departments using the reciprocal method of allocation. Find the total cost that will be
allocated to Dept. X.

SOLUTION:

Allocated to X: $1,195.92
A = $400 + .1B A = 510.20
B = $1,000 + .2A B = 1,102.04

A B X Y
------- ------- ------- -------
Direct costs $400.00 $1,000.00
A's cost allocated (510.20) 102.04 $204.08 $204.08
B's costs allocated 110.20 (1,102.04) 991.84 0
------- -------
Allocated to X $1,195.92
Allocated to Y $204.08

5. The following data are for Lexington Stores, which has two stores and one service center.
Concord Graham
------- ------
Percentage of services used in current year 40% 60%
Expected long-term use of services 30% 70%

Budgeted central purchasing costs were $100,000 fixed and $75,000 variable. Actual fixed costs were $140,000 and actual
variable costs were $105,000. The managers wish to allocate the actual central purchasing costs to the stores based on actual
use of the central purchasing service.

a. Compute the allocation to the Concord store.

b. Compute the allocation to the Graham store.

SOLUTION:

a. To Concord: $98,000 [40% x ($140,000 + $105,000)]

b. To Graham: $147,000 [60% x ($140,000 + $105,000)]

6. The following data are for Lexington Stores, which has two stores and one service center.
Concord Graham
------- ------
Percentage of services used in current year 40% 60%
Expected long-term use of services 30% 70%

Budgeted central purchasing costs were $100,000 fixed and $75,000 variable. Actual fixed costs were $140,000 and actual
variable costs were $105,000. The company wishes to allocate the budgeted variable costs of the central purchasing unit based
on actual use of the unit's services and to allocate budgeted fixed costs based on expected long-term use of the unit's services.

a. Compute the total cost allocated to the Concord store for the services of the central purchasing unit.

b. Compute the total cost allocated to the Graham store for the services of the central purchasing unit.

SOLUTION:

a. To Concord: $60,000 ($75,000 x 40% + $100,000 x 30%)

b. To Graham: $115,000 ($75,000 x 60% + $100,000 x 70%)

7. Following are data about Hamilton Co.'s two service departments and two operating departments.
Service Depts. Operating Depts.
-------------- ---------------
A B X Y
------- ------ ------ ------
Direct costs $400 $600 $2,000 $3,000
Services performed by Dept. A 30% 30% 40%
Services performed by Dept. B. 20% 70% 10%
a. Hamilton allocates costs of its service departments using the direct method of allocation. Find the total cost that will be
allocated to each of the operating departments.

b. Hamilton allocates the costs of its service departments using the step-down method, beginning with Dept. A. Find the total
amount of cost that will be allocated to each of the operating departments.

c. Hamilton allocates costs of its service departments using the reciprocal method of allocation. Find the total cost that will be
allocated to each of the operating departments.

SOLUTION:

a. Allocated to X: $696.43 {$400 x [30/(30 + 40)] + $600 x [70/(70 + 10)]}


Allocated to Y: $303.57 {$400 x [40/(30 + 40)] + $600 x [10/(70 + 10)]}

b. Allocated to X: $750.00, Allocated to Y: $250.00


A B X Y
---- ---- ------- -------
A's direct cost $400
A's cost allocated (400) $120 $120.00 $160.00
B's direct cost 600
----
Total for allocating $720
B's costs allocated (720) 630.00 90.00
------- ------
Allocated to X $750.00
Allocated to Y $250.00

c. Allocated to X: $702.13, Allocated to Y: $297.87


A = $400 + .2B A = 553.19
B = $600 + .3A B = 765.96

A B X Y
------- ------- ------- -------
Direct costs $400.00 $600.00
A's cost allocated (553.19) 165.96 $165.96 $221.27
B's costs allocated 153.19 (765.96) 536.17 76.60
------- -------
Allocated to X $702.13
Allocated to Y $297.87

8. Following are data about Hawley Co.'s two service departments and three operating departments.
Service Depts. Operating Depts.
-------------- ----------------------
A B X Y Z
------- ------ ------ ------ ------
Direct costs $400 $600
Services performed by Dept. A 30% 40% 20% 10%
Services performed by Dept. B. 40% 20% 20% 20%

Hawley allocates costs of its service departments using the reciprocal method of allocation. Find the total costs that will be
allocated to each of the operating departments.

SOLUTION:

Allocated to x: $454.55, allocated to Y: $309.09, Allocated to Z: $236.36


A = $400 + .4B A = 727.27
B = $600 + .3A B = 818.18

A B X Y Z
------- ------- ------- ------- -------
Direct costs $400.00 $600.00
A's cost allocated (727.27) 218.18 $290.91 $145.45 $ 72.72
B's costs allocated 327.27 (818.18) 163.64 163.64 163.64
------- ------- -------
Allocated to X $454.55
Allocated to Y $309.09
Allocated to Z $236.36

9. Following are data about Augusta Co.'s three service departments and two operating departments.
Service Depts. Operating Depts.
--------------------- ----------------
A B C X Y
------- ------ ------ ------ ------
Direct costs $150 $300 $350
Services performed by Dept. A 20% 30% 40% 10%
Services performed by Dept. B. 10% 20% 50% 20%
Services performed by Dept. C 30% 40% 15% 15%

a. Augusta allocates costs of its service departments using the direct method of allocation. Find the total cost that will be
allocated to Dept. X.

b. Augusta allocates the costs of its service departments using the step-down method, beginning with Dept. A followed by Dept.
B. Find the total amount of cost that will be allocated to Dept. X.

SOLUTION:

a. Allocated to X: $509.29 {$150 x [40/(40 + 10)] + $300 x [50/(50 + 20)] + $350 x [15/(15 + 15)]}

b. Allocated to X: $477.50
A B C X Y
---- ---- ------- ------- -------
A's direct cost $150
A's cost allocated (150) $ 30 $ 45.00 $ 60.00 $ 15.00
B's direct cost 300
----
Total for allocating $330
B's costs allocated (330) 73.33 183.34 73.33
C's direct cost 350.00
-------
Total for allocating $468.33
C's costs allocated (468.33) 234.16 234.16
------ ------
Allocated to X $477.50
Allocated to Y $322.50

10. Osseo Company had the following results in June.

Planned Actual
-------- --------
Sales $160,000 $162,500
Variable costs at $5 per unit 100,000 102,500
-------- --------
Contribution margin $ 60,000 $ 60,000
======== ========

Planned sales were 20,000 units, actual sales were 20,500 units.

a. Find the sales price variance. Indicate F or U

b. Find the sales volume variance. Indicate F or U

SOLUTION:

a. $1,500 U {20,500 x [($162,500/20,500) - $8]}

b. $1,500 F [$3 x (20,500 - 20,000)]


CHAPTER 10

BUDGETARY CONTROL AND


RESPONSIBILITY ACCOUNTING

Summary of Questions by STUDY Objectives and Bloom’s Taxonomy

True-False Statements
sg
1. 1 K 9. 3 C 17. 3 K 25. 5 K 33. 3 K
sg
2. 1 C 10. 3 K 18. 3 K 26. 6 K 34. 4 C
sg
3. 1 K 11. 3 K 19. 4 C 27. 7 K 35. 5 K
sg
4. 2 K 12. 3 C 20. 4 C 28. 7 K 36. 7 K
a a,sg
5. 2 C 13. 3 C 21. 4 C 29. 8 C 37. 8 K
a
6. 2 C 14. 3 C 22. 4 C 30. 8 C
sg
7. 2 K 15. 3 K 23. 4 C 31. 1 K
sg
8. 2 C 16. 3 K 24. 5 C 32. 2 K
Multiple Choice Questions
38. 1 K 62. 3 C 86. 4 K 110. 6 C 134. 7 AP
39. 1 C 63. 3 K 87. 4 C 111. 6 C 135. 7 C
40. 1 K 64. 3 C 88. 4 C 112. 6 AP 136. 7 AP
41. 1 C 65. 3 K 89. 4 C 113. 7 AN 137. 7 C
a
42. 1 K 66. 3 C 90. 4 C 114. 7 AP 138. 8 AP
a
43. 1 K 67. 3 C 91. 4 K 115. 7 AN 139. 8 K
a
44. 1 K 68. 3 K 92. 4 C 116. 7 AP 140. 8 C
a
45. 2 C 69. 3 K 93. 4 C 117. 7 AP 141. 8 C
a
46. 2 C 70. 3 AP 94. 4 C 118. 7 AP 142. 8 C
a
47. 2 C 71. 3 C 95. 4 C 119. 7 AP 143. 8 AP
a
48. 2 C 72. 3 C 96. 4 K 120. 7 AP 144. 8 AP
a
49. 2 C 73. 3 C 97. 5 C 121. 7 AP 145. 8 AN
sg
50. 2 C 74. 3 AP 98. 5 C 122. 7 AP 146. 1 C
sg
51. 2 C 75. 3 AP 99. 5 C 123. 7 AN 147. 2 K
st
52. 2 C 76. 3 AP 100. 5 C 124. 7 AP 148. 2 K
sg
53. 2,3 C 77. 3 AP 101. 5 C 125. 7 AP 149. 3 AP
st
54. 3 C 78. 3 AP 102. 6 K 126. 7 C 150. 3 K
sg
55. 3 AP 79. 3 AP 103. 6 C 127. 7 C 151. 3 K
st
56. 3 AP 80. 3 AP 104. 6 K 128. 7 K 152. 3 K
sg
57. 3 AP 81. 4 K 105. 6 C 129. 7 C 153. 4 K
st
58. 3 C 82. 4 AP 106. 6 C 130. 7 K 154. 4 K
sg
59. 3 C 83. 4 C 107. 6 AP 131. 7 AP 155. 6 K
st
60. 3 K 84. 4 C 108. 6 C 132. 7 K 156. 7 K
sg
61. 3 C 85. 4 C 109. 6 AP 133. 7 K 157. 7 AP
sg
Summary of Questions by STUDY Objectives and Bloom’s Taxonomy

Brief Exercises
158. 3 AP 161. 3 AP 164. 7 AP 167. 7 AN
a
159. 3 AP 162. 4 AP 165. 7 AP 168. 8 AP
a
160. 3 AP 163. 6 AP 166. 7 AP 169. 8 AP
Exercises
170. 2 AP 174. 3 AP 178. 3 AP 182. 5 AN 186. 7 AN
171. 2,3 AP 175. 3 AP 179. 3 AP 183. 5 AP 187. 7 AN
172. 3 AP 176. 3 AP 180. 3,6 AP 184. 6 AN 188. 7 AN
173. 3 AP 177. 3 AP 181. 4,5 AP 185. 7 AP 189. 7 AN
Completion Statements
190. 1 K 193. 3 K 196. 4 K 199. 7 K
191. 1 K 194. 3 K 197. 4 K 200. 7 K
192. 1 K 195. 4 K 198. 4 K 201. 7 K

SUMMARY OF STUDY OBJECTIVES BY QUESTION TYPE

Ite Typ Ite Typ Ite Typ Ite Typ Ite Typ Ite Typ Ite Type
Study Objective 1
1. TF 31. TF 40. MC 43. MC 190. C
2. TF 38. MC 41. MC 44. MC 191. C
3. TF 39. MC 42. MC 146. MC 192. C

Study Objective 2
4. TF 7. TF 45. MC 48. MC 51. MC 147. MC 171. Ex
5. TF 8. TF 46. MC 49. MC 52. MC 148. MC
6. TF 32. TF 47. MC 50. MC 53. MC 170. Ex

Study Objective 3
9. TF 18. TF 60. MC 69. MC 78. MC 160. BE 178. Ex
10. TF 33. TF 61. MC 70. MC 79. MC 161. BE 179. Ex
11. TF 53. MC 62. MC 71. MC 80. MC 171. Ex 180. Ex
12. TF 54. MC 63. MC 72. MC 149. MC 172. Ex 193. C
13. TF 55. MC 64. MC 73. MC 150. MC 173. Ex 194. C
14. TF 56. MC 65. MC 74. MC 151. MC 174. Ex
15. TF 57. MC 66. MC 75. MC 152. MC 175. Ex
16. TF 58. MC 67. MC 76. MC 158. BE 176. Ex
17. TF 59. MC 68. MC 77. MC 159. BE 177. Ex

Study Objective 4
19. TF 34. TF 85. MC 90. MC 95. MC 181. Ex
20. TF 81. MC 86. MC 91. MC 96. MC 195. C
21. TF 82. MC 87. MC 92. MC 153. MC 196. C
22. TF 83. MC 88. MC 93. MC 154. MC 197. C
23. TF 84. MC 89. MC 94. MC 162. BE 198. C

Study Objective 5
24. TF 35. TF 98. MC 100. MC 181. Ex 183. Ex
25. TF 97. MC 99. MC 101. MC 182. Ex

CHAPTER STUDY OBJECTIVES reports that compare actual results with planned objectives, (b)
analyzing the differences to determine their causes, (c) taking
1. Describe the concept of budgetary control. appropriate corrective action, and (d) modifying future plans, if
Budgetary control consists of (a) preparing periodic budget necessary.
2. Evaluate the usefulness of static budget reports. 3. Certain budget reports are prepared monthly,
Static budget reports are useful in evaluating the progress whereas others are prepared more frequently depending on
toward planned sales and profit goals. They are also the activities being monitored.
appropriate in assessing a manager's effectiveness in
controlling costs when (a) actual activity closely approximates 4. The master budget is not used in the budgetary
the master budget activity level, and/or (b) the behavior of the control process.
costs in response to changes in activity is fixed.
5. A master budget is most useful in evaluating a
3. Explain the development of flexible budgets and manager's performance in controlling costs.
the usefulness of flexible budget reports. To develop the
flexible budget it is necessary to: (a) Identify the activity index 6. A static budget is one that is geared to one level of
and the relevant range of activity; (b) Identify the variable activity.
costs, and determine the budgeted variable cost per unit of
activity for each cost; (c) Identify the fixed costs, and determine 7. A static budget is changed only when actual activity is
the budgeted amount for each cost; (d) Prepare the budget for different from the level of activity expected.
selected increments of activity within the relevant range.
Flexible budget reports permit an evaluation of a 8. A static budget is most useful for evaluating a
manager's performance in controlling production and costs. manager's performance in controlling variable costs.

4 Describe the concept of responsibility 9. A flexible budget can be prepared for each of the
accounting. Responsibility accounting involves accumulating types of budgets included in the master budget.
and reporting revenues and costs on the basis of the individual
manager who has the authority to make the day-to-day 10. A flexible budget is a series of static budgets at
decisions about the items. The evaluation of a manager's different levels of activities.
performance is based on the matters directly under the
manager's control. In responsibility accounting, it is necessary 11. Flexible budgeting relies on the assumption that unit
to distinguish between controllable and noncontrollable fixed variable costs will remain constant within the relevant range of
costs and to identify three types of responsibility centers: cost, activity.
profit, and investment. 12. Total budgeted fixed costs appearing on a flexible
5. Indicate the features of responsibility reports for budget will be the same amount as total fixed costs on the
cost centers. Responsibility reports for cost centers compare master budget.
actual costs with flexible budget data. The reports show only
controllable costs, and no distinction is made between variable 13. A flexible budget is prepared before the master
and fixed costs. budget.

6. Identify the content of responsibility reports for 14. The activity index used in preparing a flexible budget
profit centers. Responsibility reports show contribution should not influence the variable costs that are being
margin, controllable fixed costs, and controllable margin for budgeted.
each profit center.
15. A formula used in developing a flexible budget is:
7. Explain the basis and formula used in evaluating Total budgeted cost = fixed cost + (total variable cost per unit ×
performance in investment centers. The primary basis for activity level).
evaluating performance in investment centers is return on
investment (ROI). The formula for computing ROI for 16. Flexible budgets are widely used in production and
investment centers is: Controllable margin ÷ Average operating service departments.
assets.
17. A flexible budget report will show both actual and
a
8. Explain the difference between ROI and residual budget cost based on the actual activity level achieved.
income. ROI is controllable margin divided by average total
assets. Residual income is the income that remains after 18. Management by exception means that management
subtracting the minimum rate of return on a company’s will investigate areas where actual results differ from planned
average operating assets. ROI sometimes provides misleading results if the items are material and controllable.
results because profitable investments are often rejected when
the investment reduces ROI but increases overall profitability. 19. Policies regarding when a difference between actual
and planned results should be investigated are generally more
restrictive for noncontrollable items than for controllable items.

TRUE-FALSE STATEMENTS 20. A distinction should be made between controllable


and noncontrollable costs when reporting information under
1. Budget reports comparing actual results with planned responsibility accounting.
objectives should be prepared only once a year.
21. Cost centers, profit centers, and investment centers
2. If actual results are different from planned results, the can all be classified as responsibility centers.
difference must always be investigated by management to
achieve effective budgetary control.
a
22. More costs become controllable as one moves down 30. Residual income generates a dollar amount which
to each lower level of managerial responsibility. represents the increase in value to the company beyond the
cost necessary to pay for the financing of assets.
23. In a responsibility accounting reporting system, as
one moves up each level of responsibility in an organization, Additional True-False Questions
the responsibility reports become more summarized and show
less detailed information. 31. Budget reports provide the feedback needed by
management to see whether actual operations are on course.
24. A cost center incurs costs and generates revenues
and cost center managers are evaluated on the profitability of 32. A static budget is an effective means to evaluate a
their centers. manager's ability to control costs, regardless of the actual
activity level.
25. The terms "direct fixed costs" and "indirect fixed
costs" are synonymous with "traceable costs" and "common 33. The flexible budget report evaluates a manager's
costs," respectively. performance in two areas: (1) pro-duction and (2) costs.

26. Controllable margin is subtracted from controllable 34. The terms controllable costs and noncontrollable
fixed costs to get net income for a profit center. costs are synonymous with variable costs and fixed costs,
respectively.
27. The denominator in the formula for calculating the
return on investment includes operating and nonoperating 35. Most direct fixed costs are not controllable by the
assets. profit center manager.

28. The formula for computing return on investment is 36. The manager of an investment center can improve
controllable margin divided by average operating assets. ROI by reducing average operating assets.
a a
29.When evaluating residual income, the calculation tells 37. Residual income and ROI are used as performance
management what percentage return was generated by the evaluation methods for profit center performance.
particular division being evaluated.

Answers to True-False Statements

Item
1. Ans.
F Item
7. Ans.
F Item
13. Ans.
F Item
19. Ans.
F Item
25. Ans.
T Item
31. Ans.
T Item
a
37. Ans.
F
2. F 8. F 14. F 20. T 26. F 32. F
3. T 9. T 15. T 21. T 27. F 33. T
4. F 10. T 16. T 22. F 28. T 34. F
a
5. F 11. T 17. T 23. T 29. F 35. F
a
6. T 12. T 18. T 24. F 30. T 36. T

MULTIPLE CHOICE QUESTIONS b. management may take corrective action.


c. management may modify the future plans.
38. What is budgetary control? d. all of these.
a. Another name for a flexible budget
b. The degree to which the CFO controls the budget 42. The purpose of the departmental overhead cost report
c. The use of budgets in controlling operations is to
d. The process of providing information on budget a. control indirect labor costs.
differences to lower level managers b. control selling expense.
c. determine the efficient use of materials.
39. A major element in budgetary control is d. control overhead costs.
a. the preparation of long-term plans.
b. the comparison of actual results with planned 43. The purpose of the sales budget report is to
objectives. a. control selling expenses.
c. the valuation of inventories. b. determine whether income objectives are being met.
d. approval of the budget by the stockholders. c. determine whether sales goals are being met.
40. Budget reports should be prepared d. control sales commissions.
a. daily.
b. monthly. 44. The comparison of differences between actual and
c. weekly. planned results
d. as frequently as needed. a. is done by the external auditors.
b. appears on the company's external financial
41. On the basis of the budget reports, statements.
a. management analyzes differences between actual c. is usually done orally in departmental meetings.
and planned results.
d. appears on periodic budget reports. a. mixed.
b. flexible.
45. A static budget c. variable.
a. should not be prepared in a company. d. fixed.
b. is useful in evaluating a manager's performance by
comparing actual variable costs and planned variable costs. 51. Assume that actual sales results exceed the planned
c. shows planned results at the original budgeted activity results for the second quarter. This favorable difference is
level. greater than the unfavorable difference reported for the first
d. is changed only if the actual level of activity is quarter sales. Which of the following statements about the
different than originally budgeted. sales budget report on June 30 is true?
a. The year-to-date results will show a favorable
difference.
46. A static budget report b. The year-to-date results will show an unfavorable
a. shows costs at only 2 or 3 different levels of activity. difference.
b. is appropriate in evaluating a manager's effectiveness c. The difference for the first quarter can be ignored.
in controlling variable costs. d. The sales report is not useful if it shows a favorable
c. should be used when the actual level of activity is and unfavorable difference for the two quarters.
materially different from the master budget activity level.
d. may be appropriate in evaluating a manager's 52. A static budget is appropriate for
effectiveness in controlling costs when the behavior of the a. variable overhead costs.
costs in response to changes in activity is fixed. b. direct materials costs.
c. fixed overhead costs.
47. A static budget is appropriate in evaluating a d. none of these.
manager's performance if
a. actual activity closely approximates the master budget 53. What is the primary difference between a static
activity. budget and a flexible budget?
b. actual activity is less than the master budget activity. a. The static budget contains only fixed costs, while the
c. the company prepares reports on an annual basis. flexible budget contains only variable costs.
d. the company is a not-for-profit organization. b. The static budget is prepared for a single level of
activity, while a flexible budget is adjusted for different activity
48. When budgeted and actual results are not the same levels.
amount, there is a budget c. The static budget is constructed using input from only
a. error. upper level management, while a flexible budget obtains input
b. difference. from all levels of management.
c. anomaly. d. The static budget is prepared only for units produced,
d. by-product. while a flexible budget reflects the number of units sold.
49. Top management's reaction to a difference between 54. A flexible budget
budgeted and actual sales often depends on a. is prepared when management cannot agree on
a. whether the difference is favorable or unfavorable. objectives for the company.
b. whether management anticipated the difference. b. projects budget data for various levels of activity.
c. the materiality of the difference. c. is only useful in controlling fixed costs.
d. the personality of the top managers. d. cannot be used for evaluation purposes because
budgeted data are adjusted to reflect actual results.
50. If costs are not responsive to changes in activity level,
then these costs can be best described as
55.The master budget of Benedict Company shows that the planned activity level for next year is expected to be 50,000 machine
hours. At this level of activity, the following manufacturing overhead costs are expected:
Indirect labor $240,000
Machine supplies 60,000
Indirect materials 70,000
Depreciation on factory building 50,000
Total manufacturing overhead $420,000
A flexible budget for a level of activity of 60,000 machine hours would show total manufacturing overhead costs of
a. $494,000.
b. $420,000.
c. $504,000.
d. $454,000
. b. The actual results for 65,000 units with a new budget
for 65,000 units.
56. Rickets Crickets prepared a 2008 budget for 60,000 c. The actual results for 65,000 units with last year's
units of product. Actual production in 2008 was 65,000 units. actual results for 67,000 units
To be most useful, what amounts should a performance report d. It doesn't matter. All of these choices are equally
for this company compare? useful.
a. The actual results for 65,000 units with the original
budget for 60,000 units
57. A department has budgeted monthly manufacturing 65. Within the relevant range of activity, the behavior of
overhead cost of $270,000 plus $3 per direct labor hour. If a total costs is assumed to be
flexible budget report reflects $522,000 for total budgeted a. linear and upward sloping.
manu-facturing cost for the month, the actual level of activity b. linear and downward sloping.
achieved during the month was c. curvilinear and upward sloping.
a. 264,000 direct labor hours. d. linear to a point and then level off.
b. 84,000 direct labor hours.
c. 174,000 direct labor hours. 66. Sales results that are evaluated by a static budget
d. Cannot be determined from the information provided. might show
1. favorable differences that are not
58. Which one of the following would be the same total justified.
amount on a flexible budget and a static budget if the activity 2. unfavorable differences that are not
level is different for the two types of budgets? justified.
a. Direct materials cost a. 1
b. Direct labor cost b. 2
c. Variable manufacturing overhead c. both 1 and 2.
d. Fixed manufacturing overhead d. neither 1 nor 2.

59. In developing a flexible budget within a relevant range 67. The selection of levels of activity to depict a flexible
of activity, budget
a. only fixed costs are included. 1.will be within the relevant range.
b. it is necessary to relate variable cost data to the 2.is largely a matter of expediency.
activity index chosen. 3.is governed by generally accepted accounting principles.
c. it is necessary to prepare a budget at 1,000 unit a. 1
increments. b. 2
d. variable and fixed costs are combined and are c. 3
reported as a total cost. d. 1 and 2

60. What budgeted amounts appear on the flexible 68. Management by exception
budget? a. causes managers to be buried under voluminous
a. Original budgeted amounts at the static budget paperwork.
activity level b. means that all differences will be investigated.
b. Actual costs for the budgeted activity level c. means that only unfavorable differences will be
c. Budgeted amounts for the actual activity level investigated.
achieved d. means that material differences will be investigated.
d. Actual costs for the estimated activity level
69. Under management by exception, which differences
61.The flexible budget between planned and actual results should be investigated?
a. is prepared before the master budget. a. Material and noncontrollable
b. is relevant both within and outside the relevant range. b. Controllable and noncontrollable
c. eliminates the need for a master budget. c. Material and controllable
d. is a series of static budgets at different levels of d. All differences should be investigated
activity.
70. Romano Roofing's budgeted manufacturing costs for
62. A flexible budget can be prepared for which of the 25,000 squares of shingles are:
following budgets comprising the master budget? Fixed manufacturing costs $15,000
a. Sales Variable manufacturing costs $20.00 per square
b. Overhead Romano produced 20,000 squares of shingles during March.
c. Direct materials How much are budgeted total manufacturing costs in March?
d. All of these a. $400,000
b. $515,000
63. Another name for the static budget is c. $500,000
a. master budget. d. $415,000
b. overhead budget.
c. permanent budget. 71. A flexible budget depicted graphically
d. flexible budget. a. is identical to a CVP graph.
b. differs from a CVP graph in the way that fixed costs
64. If a company plans to sell 24,000 units of product but are shown.
sells 30,000, the most appropriate comparison of the cost data c. differs from a CVP graph in the way that variable
associated with the sales will be by a budget based on costs are shown.
a. the original planned level of activity. d. differs from a CVP graph in that sales revenue is not
b. 27,000 units of activity. shown.
c. 30,000 units of activity.
d. 24,000 units of activity. 72. The activity index used in preparing the flexible
budget
a. is prescribed by generally accepted accounting 73. A static budget is not appropriate in evaluating a
principles. manager's effectiveness if a company has
b. is only applicable to fixed manufacturing costs. a. substantial fixed costs.
c. is the same for all departments. b. substantial variable costs.
d. should significantly influence the costs that are being c. planned activity levels that match actual activity
budgeted. levels.
d. no variable costs.

74. Trepid Manufacturing Company prepared a static budget of 40,000 direct labor hours, with estimated overhead costs of
$200,000 for variable overhead and $60,000 for fixed overhead. Trepid then prepared a flexible budget at 38,000 labor hours. How
much is total overhead costs at this level of activity?
a. $190,000
b. $250,000
c. $247,000
d. $260,000

75. For June, Mark Manufacturing estimated sales revenue at $200,000. It pays sales commissions that are 4% of sales. The
sales manager's salary is $95,000, estimated shipping expenses total 1% of sales, and miscellaneous selling expenses are $5,000.
How much are budgeted selling expenses for the month of July if sales are expected to be $180,000?
a. $14,000
b. $109,000
c. $9,000
d. $110,000

76. Ziglar’s Sipit Company budgeted manufacturing costs for 25,000 sipits are:
Fixed manufacturing costs $25,000 per month
Variable manufacturing costs $12.00 per sipit
Ziglar’s produced 20,000 sipits during March. How much is the flexible budget for total manufacturing costs for March?
a. $260,000
b. $325,000
c. $240,000
d. $265,000

77. True Masons budgeted costs for 25,000 linear feet of block are:
Fixed manufacturing costs $12,000 per month
Variable manufacturing costs $16.00 per linear
True Masons installed 20,000 linear feet of block during March. How much is budgeted total manufacturing costs in March?
a. $320,000
b. $412,000
c. $400,000
d. $332,000

78. In the Klugman Company, indirect labor is budgeted for $36,000 and factory supervision is budgeted for $12,000 at
normal capacity of 80,000 direct labor hours. If 90,000 direct labor hours are worked, flexible budget total for these costs is
a. $48,000.
b. $54,000.
c. $52,500.
d. $49,500.

79. Wayman Company uses flexible budgets. At normal capacity of 8,000 units, budgeted manufacturing overhead is:
$48,000 variable and $135,000 fixed. If Wayman had actual overhead costs of $187,500 for 9,000 units produced, what is the
difference between actual and budgeted costs?
a. $1,500 unfavorable
b. $1,500 favorable
c. $4,500 unfavorable
d. $6,000 favorable
Indirect labor 200,000 Taxe
10,000
80. A company's planned activity level for next year is Factory supplies 20,000
expected to be 100,000 machine hours. At this level of activity, Supervision 50,00
the company budgeted the following manufacturing overhead A flexible budget prepared at the 80,000 machine hours level
costs: of activity would show total manufacturing overhead costs of
Variable a. $288,000.
Fixed b. $360,000.
Indirect materials $140,000 c. $384,000.
Depreciation d. $408,000.
c. Fewer costs are controllable as one moves up to each
81. The accumulation of accounting data on the basis of higher level of managerial responsibility.
the individual manager who has the authority to make day-to- d. The term segment is sometimes used to identify
day decisions about activities in an area is called areas of responsibility in decentralized operations.
a. static reporting.
b. flexible accounting. 89. Costs incurred indirectly and allocated to a
c. responsibility accounting. responsibility level are considered to be
d. master budgeting. a. nonmaterial.
b. mixed.
82. Cart Company recorded operating data for its shoe c. controllable.
division for the year. d. noncontrollable.
Sales $750,000
Contribution margin 150,000 90. Management by exception
Controllable fixed costs 90,000a. is most effective at top levels of management.
Average total operating assets 300,000b. can be implemented at each level of responsibility
How much is controllable margin for the year? within an organization.
a. 20% c. can only be applied when comparing actual results
b. 50% with the master budget.
c. $150,000 d. is the opposite of goal congruence.
d. $60,000
91. Which responsibility centers generate both revenues
83. A cost is considered controllable at a given level of and costs?
managerial responsibility if a. Investment and profit centers
a. the manager has the power to incur the cost within a b. Profit and cost centers
given time period. c. Cost and investment centers
b. the cost has not exceeded the budget amount in the d. Only profit centers
master budget.
c. it is a variable cost, but it is uncontrollable if it is a 92. The linens department of a large department store is
fixed cost. a. not a responsibility center.
d. it changes in magnitude in a flexible budget. b. a profit center.
c. a cost center.
84. As one moves up to each higher level of managerial d. an investment center.
responsibility,
a. fewer costs are controllable. 93. The foreign subsidiary of a large corporation is
b. the responsibility for cost incurrence diminishes. a. not a responsibility center.
c. a greater number of costs are controllable. b. a profit center.
d. performance evaluation becomes less important. c. a cost center.
d. an investment center.
85. A responsibility report should
a. be prepared in accordance with generally accepted 94.The maintenance department of a manufacturing company
accounting principles. is a(n)
b. show only those costs that a manager can control. a. segment.
c. only show variable costs. b. profit center.
d. only be prepared at the highest level of managerial c. cost center.
responsibility. d. investment center.

86. Top management can control 95. Which of the following is not a correct match?
a. only controllable costs. 1.Incurs costs
b. only noncontrollable costs. 2.Generates revenue
c. all costs. 3.Controls investment funds
d. some noncontrollable costs and all controllable costs. a.Investment Center 1, 2, 3
b.Cost Center 1
87. Not-for-profit entities c.Profit Center 1, 2, 3
a. do not use responsibility accounting. d.All are correct matches.
b. utilize responsibility accounting in trying to maximize
net income. 96. A cost center
c. utilize responsibility accounting in trying to minimize a. only incurs costs and does not directly generate
the cost of providing services. revenues.
d. have only noncontrollable costs. b. incurs costs and generates revenues.
c. is a responsibility center of a company which incurs
88. Which of the following is not a true statement? losses.
a. All costs are controllable at some level within a d. is a responsibility center which generates profits and
company. evaluates the investment cost of earning the profit.
b. Responsibility accounting applies to both profit and
not-for-profit entities. 97. A manager of a cost center is evaluated mainly on
a. the profit that the center generates.
b. his or her ability to control costs. b. success in meeting budgeted goals for controllable
c. the amount of investment it takes to support the cost costs.
center. c. amount of controllable margin generated by the profit
d. the amount of revenue that can be generated. center.
d. amount of contribution margin generated by the profit
98. Performance reports for cost centers compare actual center.
a. total costs with static budget data.
b. total costs with flexible budget data. 104.Controllable margin is defined as
c. controllable costs with static budget data. a. sales minus variable costs.
d. controllable costs with flexible budget data. b. sales minus contribution margin.
c. contribution margin less controllable fixed costs.
99. In the performance report for cost centers, d. contribution margin less noncontrollable fixed costs.
a. controllable and noncontrollable costs are reported.
b. fixed costs are not reported. 105. Controllable margin is most useful for
c. no distinction is made between fixed and variable a. external financial reporting.
costs. b. preparing the master budget.
d. only materials and controllable costs are reported. c. performance evaluation of profit centers.
d. break-even analysis.
100. Of the following choices, which contain both a
traceable fixed cost and a common fixed cost? 106. Which of the following will not result in an unfavorable
a. Profit center manager's salary and timekeeping costs controllable margin difference?
for a responsibility center's employees. a. Sales exceeding budget; costs under budget
b. Company president's salary and company personnel b. Sales exceeding budget; costs over budget
department costs. c. Sales under budget; costs under budget
c. Company personnel department costs and d. Sales under budget; costs over budget
timekeeping costs for a responsibility center's employees.
d. Depreciation on a responsibility center's equipment
and supervisory salaries for the center.

101.Which of the following is not an indirect fixed cost?


a. Company president's salary
b. Depreciation on the company building housing
several profit centers
c. Company personnel department costs
d. Profit center supervisory salaries

102. A profit center is


a. a responsibility center that always reports a profit.
b. a responsibility center that incurs costs and generates
revenues.
c. evaluated by the rate of return earned on the
investment allocated to the center.
d. referred to as a loss center when operations do not
meet the company's objectives.

103. The best measure of the performance of the manager


of a profit center is the
a. rate of return on investment.

107. Given below is an excerpt from a management performance report:


Budget Actual Difference
Contribution margin $1,000,000 $1,050,000 $50,000
Controllable fixed costs $ 500,000 $ 450,000 $50,000
The manager's overall performance
a. is 20% below expectations.
b. is 20% above expectations.
c. is equal to expectations.
d. cannot be determined from information given.

108.Which of the following are financial measures of performance?


1. Controllable margin
2. Product quality
3. Labor productivity
a. 1
b. 2
c. 3
d. 1 and 3
109.Given below is an excerpt from a management performance report:
Budget Actual Difference
Contribution margin$600,000$580,000$20,000 U
Controllable fixed costs$200,000$220,000$20,000 U
The manager's overall performance
a. is 10% above expectations.
b. is 10% below expectations.
c. is equal to expectations.
d. cannot be determined from the information provided.

110.A responsibility report for a profit center will


a. not show controllable fixed costs.
b. not show indirect fixed costs.
c. show noncontrollable fixed costs.
d. not show cumulative year-to-date results.

111. The dollar amount of the controllable margin


a. is usually higher than the contribution margin.
b. is usually lower than the contribution margin.
c. is always equal to the contribution margin.
d. cannot be a negative figure.

112. Garrison Company recorded operating data for its shoe division for the year. The company’s desired return is 5%.
Sales $500,000
Contribution margin 100,000
Total direct fixed costs 60,000
Average total operating assets 200,000
Which one of the following reflects the controllable margin for the year?
a. 20%
b. 50%
c. $30,000
d. $40,000

113. The area manager of the Steak House Restaurants is considering two possible expansion alternatives. The required
investments, expected controllable margins, and the ROIs of each are as follows:
Project Investment Controllable Margin ROI
Charlotte $120,000 $30,000 25%
Richmond $540,000 $50,000 9.25%
The Steak House segment has currently $2,000,000 in invested capital and a controllable margin of $250,000. Which one of following
projects will increase the Steak House division’s ROI?
a. Both the Charlotte and Richmond options
b. Only the Charlotte option
c. Only the Richmond option
d. Neither the Charlotte nor the Richmond options

114. Timex Corporation recorded operating data for its Cheap division for the year. Timex requires its return to be 10%.
Sales $ 700,000
Controllable margin 80,000
Total average assets 2,000,000
Fixed costs 50,000
What is the ROI for the year?
a. 4%
b. 35%
c. –6%
d. 1.5%

115.Halpern Division’s operating results include: controllable margin of $150,000, sales totaling $1,200,000, and average operating
assets of $500,000. Halpern is considering a project with sales of $100,000, expenses of $86,000, and an investment of average
operating assets of $200,000. Halpern’s required rate of return is 9%. Should Halpern accept this project?
a. Yes, ROI will drop by 6.6% which is still above the required rate of return.
b. No, the return is less than the required rate of 9%.
c. Yes, ROI still exceeds the cost of capital.
d. No, ROI will decrease to 7%.
1 16. Perot Manufacturing reported the following items for 2008:
Income tax expense $ 30,000
Contribution margin 100,000
Controllable fixed costs 40,000
Interest expense 20,000
Total operating assets 325,000
How much is controllable margin?
a. $100,000
b. $60,000
c. $30,000
d. $10,000

117.Merck Pharmaceuticals is evaluating its Vioxx division, an investment center. The division has a $45,000 controllable margin and
$300,000 of sales. How much will Merck’s average operating assets be when its return on investment is 10%?
a. $450,000
b. $495,000
c. $300,000
d. $255,000

118.An investment center generated a contribution margin of $200,000, fixed costs of $100,000 and sales of $1,000,000. The center’s
average operating assets were $400,000. How much is the return on investment?
a. 25%
b. 175%
c. 50%
d. 75%

119. Safety Seats Company recorded operating data for its auto accessories division for the year.
Sales $375,000
Contribution margin 75,000
Total direct fixed costs 45,000
Average total operating assets 200,000
How much is ROI for the year if management is able to identify a way to improve the contribution margin by $15,000, assuming fixed
costs are held constant?
a. 45.0%
b. 22.5%
c. 15.0%
d. 12.0%

120. The current controllable margin for Claremont Division is $62,000. Its current operating assets are $200,000. The division is
considering purchasing equipment for $60,000 that will increase annual controllable margin by an estimated $10,000. If the equipment
is purchased, what will happen to the return on investment for Claremont Division?
a. An increase of 16.1%
b. A decrease of 13.3%
c. A decrease of 3.3%
d. A decrease of 7.2%

121. CinRich Corporation recorded operating data for its Waterhole division for the year. CinRich requires its return to be 9%.
Sales $500,000
Controllable margin 90,000
Total average assets 300,000
Fixed costs 30,000
How much is ROI for the year?
a. 10%
b. 16.7%
c. 20%
d. 30%

122. Lou Alabassi is the North Division manager and his performance is evaluated by executive management based on Division
ROI. The current controllable margin for North Division is $46,000. Its current operating assets total $210,000. The division is
considering purchasing equipment for $40,000 that will increase sales by an estimated $10,000, with annual depreciation of $10,000. If
the equipment is purchased, what will happen to the return on investment for the division?
a. An increase of 0.5%
b. A decrease of 0.5%
c. A decrease of 3.5%
d. It will remain unchanged.
123. Cruise Division of Harrah’s Company’s operating results include: controllable margin, $200,000; sales $2,200,000; and
operating assets, $800,000. The Cruise Division’s ROI is 25%. Management is considering a project with sales of $100,000, variable
expenses of $60,000, fixed costs of $40,000; and an asset investment of $150,000. Should management accept this new project?
a. No, since ROI will be lowered.
b. Yes, since ROI will increase.
c. Yes, since additional sales always mean more customers.
d. No, since a loss will be incurred.

124. The Eastern Division of Flint Corp. had an ROI of 25% when sales were $1 million and controllable margin was $200,000.
What were the average operating assets?
a. $50,000
b. $250,000
c. $800,000
d. $4,000

125.Cart Company recorded operating data for its shoe division for the year.
Sales$500,000
Contribution margin 90,000
Total fixed costs 60,000
Average total operating assets 200,000
How much is ROI for the year if management is able to identify a way to improve the contribution margin by $20,000, assuming fixed
costs are held constant?
a. 25%
b. 18%
c. 45%
d. 12%

126.A distinguishing characteristic of an investment center is that


a. revenues are generated by selling and buying stocks and bonds.
b. interest revenue is the major source of revenues.
c. the profitability of the center is related to the funds invested in the center.
d. it is a responsibility center which only generates revenues.

127. A measure frequently used to evaluate the performance of the manager of an investment center is
a. the amount of profit generated.
b. the rate of return on funds invested in the center.
c. the percentage increase in profit over the previous year.
d. departmental gross profit.

128. Return on investment is calculated by dividing


a. contribution margin by sales.
b. controllable margin by sales.
c. contribution margin by average operating assets.
d. controllable margin by average operating assets.
b. An investment center is only responsible for its
investments.
129.Which one of the following will not increase return on c. An investment center is only responsible for revenues
investment? and expenses.
a. Variable costs are increased d. A profit center is evaluated using contribution margin,
b. An increase in sales while an investment center is evaluated using ROI.
c. Average operating assets are decreased
d. Variable costs are decreased 132. The denominator in the formula for return on
investment calculation is
130. If an investment center has generated a controllable a. investment center controllable margin.
margin of $75,000 and sales of $300,000, what is the return on b. dependent on the specific type of profit center.
investment for the investment center if average operating c. average investment center operating assets.
assets were $500,000 during the period? d. sales for the period.
a. 15%
b. 25% 133. In the formula for ROI, idle plant assets are
c. 45% a. included in the calculation of controllable margin.
d. 60% b. included in the calculation of operating assets.
c. excluded in the calculation of operating assets.
131. Which statement is true? d. excluded from total assets.
a. An investment center is responsible for revenues and
expenses, as well as earning a return on assets. 134.In computing ROI, land held for future use
a. will hurt the performance measurement of an c. is included in the calculation of operating assets.
investment center's manager. d. is considered a nonoperating asset.
b. is important in evaluating the performance of a profit
center manager.
135. Dodge City Parts has a current return on investment of 10% and the company has established an 8% minimum rate of return
for the division. The division manager has two investment projects available, for which the following estimates have been made:
Project A - Annual controllable margin = $24,000, operating assets = $400,000
Project B - Annual controllable margin = $60,000, operating assets = $550,000
Which project should be funded?
a. Both projects
b. Project A
c. Project B
d. Neither project

136. If an investment center has a $45,000 controllable margin and $600,000 of sales, what average operating assets are needed
to have a return on investment of 10%?
a. $60,000
b. $105,000
c. $450,000
d. $600,000

137. Which of the following valuations of operating assets is not readily available from the accounting records?
a. Cost
b. Book value
c. Market value
d. Both cost and market value
a
138. The following information is available for Aggie Auto Sales:
Average operating assets $800,000
Controllable margin 80,000
Contribution margin 200,000
Minimum rate of return 8%
How much is Aggie Auto’s residual income?
a. $136,000
b. $720,000
c. $16,000
d. $64,000
a. To determine whether decentralization is possible or
a
139. What is the goal of residual income? not
a. To maximize the amount of costs which are b. To motivate managers through possible termination
controllable c. To evaluate management performance
b. To maximize profits d. To measure company profits
c. To maximize the total amount of residual income
a
d. To maximize controllable margin 143. Niceville Company had sales of $400,000, variable
costs of $200,000, and direct fixed costs totaling $100,000.
a
140. Which one of the following is a correct statement The company’s operating assets total $800,000, and its
about residual income? required return is 10%. How much is the residual income?
a. Its goal is to maximize profits of an investment center. a. $120,000
b. It is less effective for evaluating investment centers b. $20,000
than ROI. c. $80,000
c. It is the ratio of controllable margin to the minimum d. $320,000
rate of return on average operating assets.
a
d. It evaluates performance by comparing the return of 144. Oxford Company earned controllable margin of
an investment center with the company’s minimum rate of $125,000 on sales of $1,600,000. The division had average
return. operating assets of $1,300,000. The company requires a return
on investment of at least 8%. How much is residual income?
a
141. Which one of the following does not impact the a. $104,000
amount of residual income? b. $21,000
a. Contribution margin c. $146,000
b. Net income d. $128,000
c. Sales
a
d. Controllable costs 145. The performance of the manager of Purina Division is
measured by residual income. Which of the following would
a
142. For what purpose do companies calculate residual decrease the manager’s performance measure?
income? a. Decrease in required rate of return
b. Increase in amount of return on investment desired
c. Increase in sales a. fixed manufacturing costs.
d. Increase in contribution margin b. fixed selling and administrative expenses.
c. variable selling and administrative expenses.
Additional Multiple Choice Questions d. both fixed manufacturing costs and fixed selling and
administrative expenses.
146. Which of the following would not be considered an
aspect of budgetary control? 149. Weiser Company uses flexible budgets. At
a. It assists in the determination of differences between normal capacity of 8,000 units, budgeted manufacturing
actual and planned results. overhead is $64,000 variable and $180,000 fixed. If Weiser
b. It provides feedback value needed by management to had actual overhead costs of $250,000 for 9,000 units
see whether actual operations are on course. produced, what is the difference between actual and budgeted
c. It assists management in controlling operations. costs?
d. It provides a guarantee for favorable results. a. $2,000 unfavorable
b. $2,000 favorable
147. A static budget is usually appropriate in c. $6,000 unfavorable
evaluating a manager's effectiveness in controlling d. $8,000 favorable
a. fixed manufacturing costs and fixed selling and 150. To develop the flexible budget, management
administrative expenses. takes all of the following steps except identify the
b. variable manufacturing costs and variable selling and a. activity index and the relevant range of activity.
administrative expenses. b. variable costs and determine the budgeted variable
c. fixed manufacturing costs and variable selling and cost per unit.
administrative expenses. c. fixed costs and determine the budgeted fixed cost per
d. variable manufacturing costs and fixed selling and unit.
administrative expenses. d. All of these options are steps in developing the
flexible budget.
148. A static budget report is appropriate for
151. A flexible budget is appropriate for
Direct Labor Costs Manufacturing Overhead Costs
a. No No
b. Yes Yes
c. Yes No
d. No Yes
c. An increase in sales
152.All of the following statements are correct about d. An increase in controllable fixed costs
management by exception except it
a. enables top management to focus on problem areas 156. Costs that relate specifically to one center and
that need attention. are incurred for the sole benefit of that center are
b. means that management has to investigate every a. common fixed costs.
budget difference. b. direct fixed costs.
c. requires that there must be some guidelines for c. indirect fixed costs.
identifying an exception. d. noncontrollable fixed costs.
d. means that top management's review of a budget
report is focused primarily on differences between actual 157. If controllable margin is $300,000 and the
results and planned objectives. average investment center operating assets are $1,000,000,
the return on investment is
153. Controllable costs for responsibility accounting a. .33%.
purposes are those costs that are directly influenced by b. 3.33%.
a. a given manager within a given period of time. c. 10%.
b. a change in activity. d. 30%.
c. production volume.
d. sales volume.

154. All of the following statements are correct about


controllable costs except
a. all costs are controllable at some level of
responsibility within a company.
b. all costs are controllable by top management.
c. fewer costs are controllable as one moves up to each
higher level of managerial responsibility.
d. costs incurred directly by a level of responsibility are
controllable at that level.

155. Which of the following will cause an increase in


ROI?
a. An increase in variable costs
b. An increase in average operating assets
Answers to Multiple Choice Questions

Ite
38. Ans
c Ite
56. Ans
b Ite
74. Ans
b Ite
92. Ans
b Ite
110. Ans
b Item
128. Ans
d Ite
146. Ans.
d
39. b 57. b 75. b 93. d 111. b 129. a 147. a
40. d 58. d 76. d 94. c 112. d 130. a 148. d
41. d 59. b 77. d 95. c 113. b 131. a 149. b
42. d 60. c 78. c 96. a 114. a 132. c 150 c
43. c 61. d 79. b 97. b 115. b 133. c .
151 b
44. d 62. d 80. d 98. d 116. b 134. d .
152 b
45. c 63. a 81. c 99. c 117. a 135. c .
153 a
46. d 64. c 82. d 100. c 118. a 136. c .
154 c
47. a 65. a 83. a 101. d 119. b 137. c .
155 c
48. b 66. c 84. c 102. b 120. c a
138. c .
156 b
49. c 67. d 85. b 103. c 121. d a
139. c .
157 d
50. d 68. d 86. c 104. c 122. c a
140. d .
a
51. a 69. c 87. c 105. c 123. a 141. b
a
52. c 70. d 88. c 106. a 124. c 142. c
a
53. b 71. d 89. d 107. b 125. a 143. b
a
54. b 72. d 90. b 108. a 126. c 144. b
a
55. a 73. b 91. a 109. b 127. b 145. b

BRIEF EXERCISES

BE 158
Shirk Productions makes a single product. Expected manufacturing costs are as follows:
Variable costs
Direct materials $6.50 per unit
Direct labor 2.40 per unit
Manufacturing overhead 1.10 per unit
Fixed costs per month
Supervisory salaries $12,600
Depreciation 3,500
Other fixed costs 2,200

Instructions
Determine the amount of manufacturing costs for a flexible budget level of 3,200 units per month.

Solution 158 (4 min.)


[3,200 × ($6.50 + $2.40 + $1.10)] + ($12,600 + $3,500 + $2,200) = $50,300

BE 159
Sekine Company uses flexible budgets. Items from the budget for March in which 2,000 units were produced and sold appear below:
Direct materials $18,000
Indirect materials - variable 2,000
Supervisor salaries 15,000
Depreciation on factory equipment 4,000
Direct labor 10,000
Property taxes on factory 1,000
Instructions
If Sekine prepares a flexible budget at 3,000 units, compute its total variable cost.

Solution 159 (4 min.)


Variable cost per unit: ($18,000 + $2,000 + $10,000) ÷ 2,000 = $15 per unit
Variable cost at 3,000 units: $15 × 3,000 = $45,000
BE 160
SugarTown’s manufacturing costs for August when production was 1,000 units appear below:
Direct material $12 per unit
Direct labor $6,500
Variable overhead 5,000
Factory depreciation 9,000
Factory supervisory salaries 7,800
Other fixed factory costs 2,500
Instructions
Compute the flexible budget manufacturing cost amount for a month when 800 units are produced.

Solution 160 (5 min.)


Direct material ($12 × 800) $ 9,600
Direct labor [($6,500 ÷ 1,000) × 800] 5,200
Variable overhead [($5,000 ÷ 1,000) × 800] 4,000
Factory depreciation—fixed 9,000
Factory supervisory salaries—fixed 7,800
Other fixed factory costs—fixed 2,500
Total $38,100

BE 161
Butterfly World’s budgeted sales for April were estimated at $500,000, sales commissions at 4% of sales, and the sales manager's
salary at $80,000. Shipping expenses were estimated at 1% of sales and miscellaneous selling expenses were estimated at $1,000,
plus 0.5% of sales.

Instructions
Determine the budgeted selling expenses on a flexible budget for April.
Solution 161 (5 min.)
Sales commissions (4% × $500,000) $ 20,000
Sales manager’s salary 80,000
Shipping expenses (1% × $500,000) 5,000
Miscellaneous selling: Fixed portion 1,000
Variable: (0.5% × $500,000) 2,500
Budgeted selling expenses $108,500

BE 162
Ranger Company produces men’s shirts. The following budgeted and actual amounts are for 2008:
Cost Budget at 2,500 units Actual Amounts at 2,900 units
Direct materials $55,000 $65,500
Direct labor 70,000 81,000
Fixed overhead 35,000 34,500

Instructions
Prepare a performance report for Ranger Company for the year.

Solution 162 (5 min.)


RANGER COMPANY
Manufacturing Performance Budget Report
For the Year Ended December 31, 2008

Budget Actual Differences


Direct materials $ 63,800 $ 65,500 $1,700 U
Direct labor 81,200 81,000 200 F
Fixed overhead 35,000 34,500 500 F
Total costs $180,000 $181,000 $1,000 U

BE 163
Lincoln Inc. reported the following items for 2008:
Controllable fixed costs $ 77,000
Contribution margin 142,000
Interest expense 20,000
Variable costs 80,000
Total assets $925,000

Instructions
Compute the controllable margin for 2008.

Solution 163 (2 min.)


$142,000 – $77,000 = $65,000

BE 164
The data for an investment center is given below.
January 1, 2008 December 31, 2008
Current Assets $ 400,000 $ 800,000
Plant Assets 3,000,000 4,000,000

The controllable margin is $615,000.

Instructions
Compute the return on investment for the center for 2008.

Solution 164 (4 min.)


Average current assets ($400,000 + $800,000) ÷ 2 = $600,000
Plant assets ($3,000,000 + $4,000,000) ÷ 2 = $3,500,000
ROI = Controllable Margin ÷ Average Operating Assets = $615,000 ÷ $4,100,000 = 15%

BE 165
Data for the Electric Division of Bowden Baseball Company which is operated as an investment center follows:
Sales $6,000,000
Contribution Margin 800,000
Controllable Fixed Costs 500,000
Return on Investment 12%

Instructions
Calculate controllable margin and average operating assets.

Solution 165 (3 min.)


Controllable Margin ($800,000 – $500,000) = $300,000
Average Operating Assets ($300,000 ÷ .12) = $2,500,000

BE 166
Wimmer Division’s operating results include:
 Controllable margin, $150,000
 Sales revenue, $1,200,000
 Operating assets, $500,000

Wimmer is considering a project with sales of $120,000, expenses of $84,000, and an investment of $180,000. Wimmer’s required rate
of return is 15%.

Instructions
Determine whether Wimmer should accept this project.

Solution 166 (5 min.)


Current ROI = $150,000 ÷ $500,000 = 30%
ROI of new project = $36,000 ÷ $180,000 = 20%
New ROI with project = [$150,000 + $36,000] ÷ [$500,000 + $180,000] = 27.4%

While ROI decreases, that does not make this a bad investment, since many projects cause total ROI to fall even though they increase
value of the division. The determination is based on how the ROI of the project compares to the required rate of return. The company is
not willing to accept any projects with an investment less than 15%, so the 20% project should be accepted.

BE 167
An investment center manager is considering three possible investments. The company’s required return is 10%. The required asset
investment, controllable margins, and the ROIs of each investment are as follows:
Project Average Investment Controllable Margin ROI
Bud $160,000 $32,000 20.0%
Wise 140,000 16,000 11.4%
Er 220,000 66,000 30%

The investment center is currently generating an ROI of 25% based on $1,200,000 in operating assets and a controllable margin of
$300,000.

Instructions
If the manager can select only one project, determine which one is the best choice to increase the investment center's ROI. Compute
how much the investment center’s ROI will be if the manager selects your recommendation.

Solution 167 (4 min.)


Er is the best choice because it increases the ROI (30% is greater than 25%).
Project New ROI
Bud ($300,000 + $32,000) ÷ ($1,200,000 + $160,000) = 24.4%
Wise ($300,000 + $16,000) ÷ ($1,200,000 + $140,000) = 23.6%
Er ($300,000 + $66,000) ÷ ($1,200,000 + $220,000) = 25.8%

a
BE 168
The owner of Shrek Toys has recently expanded his business in order to add an additional product line. In addition to toys, the
company now sells shirts. The company has a minimum rate of return of 12%.
Toys Shirts
Sales $600,000 $200,000
Controllable margin 120,000 10,000
Average operating assets 900,000 200,000

Instructions
Compute the residual income for both investment centers.
Solution 168 (5 min.)
Toys Shirts
Controllable margin $120,000 $ 10,000
Average assets × 12% 108,000 24,000
Residual income $ 12,000 $(14,000)

a
BE 169
A & B Flooring has 4 divisions. Its hardwood flooring division’s information follows for 2008:
Sales $4,000,000
Controllable margin 250,000
Variable costs 60,000
Average operating assets 1,800,000

Instructions
A & B’s required rate of return is 9%. How much is residual income?

Solution 169 (4 min.)


$250,000 – (9% × $1,800,000) = $88,000
EXERCISES

Ex. 170
Doonan Company's master budget reflects budgeted sales information for the month of June, 2008, as follows:
Budgeted Quantity Budgeted Unit Sales Price
Product A 20,000 $7
Product B 24,000 $9
During June, the company actually sold 19,500 units of Product A at an average unit price of $7.10 and 24,800 units of Product B at an
average unit price of $8.90.

Instructions
Prepare a Sales Budget Report for the month of June for Doonan Company which shows whether the company achieved its planned
objectives.

Solution 170 (10–15 min.)


DOONAN COMPANY
Sales Budget Report
For the Month Ended June 30, 2008
Product Line Budget Actual Difference
Product A $140,000 $138,450 $1,550 U
Product B 216,000 220,720 4,720 F
Total sales $356,000 $359,170 $3,170 F
Ex. 171
Colaw Manufacturing Co.'s static budget at 6,000 units of production includes $36,000 for direct labor and $6,000 for direct materials.
Total fixed costs are $24,000.
Instructions
a. Determine how much would appear on Colaw's flexible budget for 2008 if 9,000 units are produced and sold.
b. How would this comparison differ if a static budget were used instead of a flexible budget for performance evaluation?

Solution 171 (8–10 min.)


a. 6,000 Units Unit Variable Cost
9,000 Units
Variable costs:
Direct labor $36,000 $6.00 $54,000
Direct materials 6,000 1.00 9,000
42,000 63,000
Fixed costs 24,000 24,000
Total costs $66,000 $87,000

b. If a static budget were used, budgeted variable costs would be only $42,000 because they would be based on the static
budget level of 6,000 units. The company would appear way over budget since the costs incurred would be related to a higher level of
activity.

Ex. 172
Jenner Company developed its annual manufacturing overhead budget for its master budget for 2008 as follows:
Expected annual operating capacity 120,000 Direct Labor Hours
Variable overhead costs
Indirect labor $420,000
Indirect materials 90,000
Factory supplies 30,000
Total variable 540,000
Fixed overhead costs
Depreciation 180,000
Supervision 120,000
Property taxes 96,000
Total fixed 396,000
Total costs $936,000

The relevant range for monthly activity is expected to be between 8,000 and 12,000 direct labor hours.
Instructions
Prepare a flexible budget for a monthly activity level of 8,000 and 9,000 direct labor hours.
Solution 172 (15–20 min.)
JENNER COMPANY
Monthly Flexible Manufacturing Overhead Budget

Activity level
Direct labor hours 8,000 9,000
Variable costs
Indirect labor $28,000 $31,500
Indirect materials 6,000 6,750
Factory supplies 2,000 2,250
Total variable 36,000 40,500
Fixed costs
Depreciation 15,000 15,000
Supervision 10,000 10,000
Property taxes 8,000 8,000
Total fixed 33,000 33,000
Total costs $69,000 $73,500

Ex. 173
Dailey Company has prepared the following monthly flexible manufacturing overhead budget for its Mixing Department:
DAILEY COMPANY
Monthly Flexible Manufacturing Overhead Budget
Mixing Department
Activity level
Direct labor hours 3,000 4,000
Variable costs
Indirect materials $ 1,500 $ 2,000
Indirect labor 15,000 20,000
Factory supplies 4,500 6,000
Total variable 21,000 28,000
Fixed costs
Depreciation 20,000 20,000
Supervision 10,000 10,000
Property taxes 15,000 15,000
Total fixed 45,000 45,000
Total costs $66,000 $73,000

Instructions
Prepare a flexible budget at the 5,000 direct labor hours of activity.

Solution 173 (15–20 min.)


DAILEY COMPANY
Monthly Flexible Manufacturing Overhead Budget
Mixing Department

Activity level
Direct labor hours 5,000
Variable costs
Indirect materials $ 2,500
Indirect labor 25,000
Factory supplies 7,500
Total variable 35,000
Fixed costs
Depreciation 20,000
Supervision 10,000
Property taxes 15,000
Total fixed 45,000
Total costs $80,000
Ex. 174
Fagan Company uses a flexible budget for manufacturing overhead based on machine hours. Variable manufacturing overhead costs
per machine hour are as follows:

Indirect labor $5.00


Indirect materials 2.50
Maintenance .50
Utilities .30

Fixed overhead costs per month are:


Supervision $600
Insurance 200
Property taxes 300
Depreciation 900

The company believes it will normally operate in a range of 2,000 to 4,000 machine hours per month.

Instructions
Prepare a flexible manufacturing overhead budget for the expected range of activity, using increments of 1,000 machine hours.

Solution 174 (15–20 min.)


FAGAN COMPANY
Monthly Flexible Manufacturing Overhead Budget
Activity level
Machine hours 2,000 3,000 4,000
Variable costs
Indirect labor $10,000 $15,000 $20,000
Indirect materials 5,000 7,500 10,000
Maintenance 1,000 1,500 2,000
Utilities 600 900 1,200
Total variable 16,600 24,900 33,200
Fixed costs
Supervision 600 600 600
Insurance 200 200 200
Property taxes 300 300 300
Depreciation 900 900 900
Total fixed 2,000 2,000 2,000
Total costs $18,600 $26,900 $35,200

Ex. 175
Dashboard Corporation's manufacturing costs for July when production was 1,000 units appears below:
Direct materials $10 per unit
Factory depreciation $8,000
Variable overhead 5,000
Direct labor 2,000
Factory supervisory salaries 5,800
Other fixed factory costs 1,500

Instructions
How much is the flexible budget manufacturing cost amount for a month when 1,100 units are produced?

Solution 175 (8–10 min.)


Direct materials ($10 × 1,100) $11,000
Direct labor [($2,000 ÷ 1,000) × 1,100] 2,200
Variable overhead [($5,000 ÷ 1,000) × 1,100] 5,500
Factory depreciation—fixed 8,000
Factory supervisory salaries—fixed 5,800
Other fixed factory costs 1,500
Total $34,000
Ex. 176
Fagan Company uses a flexible budget for manufacturing overhead based on machine hours. Variable manufacturing overhead costs
per machine hour are as follows:
Indirect labor $5.00
Indirect materials 2.50
Maintenance .50
Utilities .30
Fixed overhead costs per month are:
Supervision $600
Insurance 200
Property taxes 300
Depreciation 900
The company believes it will normally operate in a range of 2,000 to 4,000 machine hours per month. During the month of August,
2008, the company incurs the following manufacturing overhead costs:
Indirect labor $14,000
Indirect materials 8,100
Maintenance 1,400
Utilities 950
Supervision 720
Insurance 200
Property taxes 300
Depreciation 930

Instructions
Prepare a flexible budget report, assuming that the company used 3,000 machine hours during August.

Solution 176 (20–25 min.)


FAGAN COMPANY
Manufacturing Overhead Budget Report (Flexible)
For the Month Ended August 31, 2008
Difference
Budget atActual at Favorable F
3,000 hrs. 3,000 hrs.Unfavorable U
Variable costs
Indirect labor $15,000 $14,000 $1,000 F
Indirect materials 7,500 8,100 600 U
Maintenance 1,500 1,400 100 F
Utilities 900 950 50 U
Total variable 24,900 24,450 450 F
Fixed Costs
Supervision 600 720 120 U
Insurance 200 200 —
Property taxes 300 300 —
Depreciation 900 930 30 U
Total fixed 2,000 2,150 150 U
Total costs $26,900 $26,600 $ 300 F
Ex. 177
Molle Company uses flexible budgets to control its selling expenses. Monthly sales are expected to be from $200,000 to $240,000.
Variable costs and their percentage relationships to sales are:

Sales commissions 6%
Advertising 4%
Traveling 5%
Delivery 1%

Fixed selling expenses consist of sales salaries $40,000 and depreciation on delivery equipment $10,000.

Instructions
Prepare a flexible budget for increments of $20,000 of sales within the relevant range.

Solution 177 (17–22 min.)


MOLLE COMPANY
Monthly Flexible Selling Expense Budget
Activity level
Sales $200,000 $220,000 $240,000

Variable expenses
Sales commissions $12,000 $13,200 $14,400
Advertising 8,000 8,800 9,600
Traveling 10,000 11,000 12,000
Delivery 2,000 2,200 2,400
Total variable 32,000 35,200 38,400
Fixed expenses
Sales salaries 40,000 40,000 40,000
Depreciation 10,000 10,000 10,000
Total fixed 50,000 50,000 50,000
Total costs $82,000 $85,200 $88,400

Ex. 178
Molle Company uses flexible budgets to control its selling expenses. Monthly sales are expected to be from $200,000 to $240,000.
Variable costs and their percentage relationships to sales are:

Sales commissions 6%
Advertising 4%
Traveling 5%
Delivery 1%

Fixed selling expenses consist of sales salaries $40,000 and depreciation on delivery equipment $10,000.
Ex. 178 (cont.)
The actual selling expenses incurred in February, 2008, by Molle Company are as follows:

Sales commissions $13,700


Advertising 8,000
Traveling 11,300
Delivery 1,600

Fixed selling expenses consist of sales salaries $41,000 and depreciation on delivery equipment $10,000.

Instructions
Prepare a flexible budget performance report, assuming that February sales were $220,000.

Solution 178 (17–22 min.)


MOLLE COMPANY
Selling Expense Budget Report (Flexible)
For the Month Ended February 29, 2008

Difference
Favorable F
Budget Actual Unfavorable U
$220,000 $220,000
Variable expenses
Sales commissions $13,200 $13,700 $ 500 U
Advertising 8,800 8,000 800 F
Traveling 11,000 11,300 300 U
Delivery 2,200 1,600 600 F
Total variable 35,200 34,600 600 F
Fixed expenses
Sales salaries 40,000 41,000 1,000 U
Depreciation 10,000 10,000 —
Total fixed 50,000 51,000 1,000 U
Total expenses $85,200 $85,600 $ 400 U
Ex. 179
A flexible budget graph for the Assembly Department shows the following:
1. At zero direct labor hours, the total budgeted cost line intersects the vertical axis at $60,000.
2. At normal capacity of 50,000 direct labor hours, the line drawn from the total budgeted cost line intersects the vertical axis at
$180,000.

Instructions
Develop the budgeted cost formula for the Assembly Department and identify the fixed and variable costs.

Solution 179 (5 min.)


Budgeted Costs:
Assembly $60,000 + $2.40.

Fixed costs are $60,000.


Variable costs are $2.40 per labor hour.
($180,000 – $60,000) ÷ 50,000.

Ex. 180
Pele Clothing Company's static budget at 2,000 units of production includes $8,000 for direct labor, $2,000 for utilities (variable), and
total fixed costs of $16,000. Actual production and sales for the year was 6,000 units, with an actual cost of $47,200.

Instructions
Determine if Pele Clothing is over or under budget.

Solution 180 (8–10 min.)


2,000 Units Unit Variable Cost 6,000 Units
Variable costs:
Direct labor $ 8,000 $4.00 $24,000
Utilities 2,000 1.00 6,000
10,000 30,000
Fixed costs 16,000 16,000
Total costs $26,000 $46,000

The company is over budget by $1,200. The flexible budget amount allowed was $46,000, and the company incurred $47,200 of actual
costs.

Ex. 181
Colter Company produces men's ties. The following budgeted and actual amounts are for 2008:
Cost Budget at 5,000 Units Actual Amounts at 5,800 Units
Direct materials $60,000 $71,000
Direct labor 75,000 86,500
Equipment depreciation 5,000 5,000
Indirect labor 7,500 8,600
Indirect materials 9,000 9,600
Rent and insurance 12,000 13,000
Instructions
Prepare a performance budget report for Colter Company for the year.

Solution 181 (8–10 min.)


COLTER COMPANY
Manufacturing Performance Budget Report
For the Year Ended December 31, 2008
Budget Actual Differences
Direct materials $ 69,600 $ 71,000 $1,400 U
Direct labor 87,000 86,500 500 F
Equipment depreciation 5,000 5,000 0
Indirect labor 8,700 8,600 100 F
Indirect materials 10,440 9,600 840 F
Rent and insurance 12,000 13,000 1,000 U
Total costs $192,740 $193,700 $ 960 U

Ex. 182
Data concerning manufacturing overhead for Friendly Company are presented below. The Mixing Department is a cost center.

An analysis of the overhead costs reveals that all variable costs are controllable by the manager of the Mixing Department and that
50% of supervisory costs are controllable at the department level.

The flexible budget formula and the cost and activity for the months of July and August are as follows:
Flexible Budget Per
Direct Labor Hour Actual Costs and Activity
July August
Direct labor hours 6,000 7,000
Overhead costs
Variable
Indirect materials $3.50 $ 20,500 $ 25,100
Indirect labor 6.00 39,500 40,700
Factory supplies 1.00 7,600 8,200
Fixed
Depreciation $20,000 15,000 15,000
Supervision 25,000 23,000 26,000
Property taxes 10,000 12,000 12,000
Total costs $117,600 $127,000

Instructions
(a) Prepare the responsibility reports for the Mixing Department for each month.
(b) Comment on the manager's performance in controlling costs during the two month period.

Solution 182 (20–25 min.)


(a) FRIENDLY COMPANY
Mixing Department
Manufacturing Overhead Cost Responsibility Report
For the Months of July and August

July August
Controllable Cost Budget Actual Difference Budget Actual Difference
Indirect materials 21,000 20,500 500 F 24,500 25,100 600 U
Indirect labor 36,000 39,500 3,500 U 42,000 40,700 1,300 F
Factory supplies 6,000 7,600 1,600 U 7,000 8,200 1,200 U
Supervision 12,500 11,500 1,000 F 12,500 13,000 500 U
Total costs 75,500 79,100 3,600 U 86,000 87,000 1,000 U

(b) The manager did a better job of controlling costs in August ($1,000 U) than in July ($3,600 U).

Ex. 183
Gentry Company's manufacturing overhead budget for the first quarter of 2008 contained the following data:
Variable Costs
Indirect materials $20,000
Indirect labor 12,000
Utilities 10,000
Maintenance 6,000
Fixed Costs
Supervisor's salary $40,000
Depreciation 8,000
Property taxes 4,500
Actual variable costs for the first quarter were:
Indirect materials $18,600
Indirect labor 13,200
Utilities 10,500
Maintenance 5,300
Actual fixed costs were as expected except for property taxes which were $4,500. All costs are considered controllable by the
department manager except for the supervisor's salary.

Instructions
Prepare a manufacturing overhead responsibility performance report for the first quarter.

Solution 183 (15–20 min.)


GENTRY COMPANY
Manufacturing Overhead Cost Responsibility Report
For the Quarter Ended March 31, 2008

Controllable Costs Budget Actual Difference


Indirect materials $20,000 $18,600 $1,400 F
Indirect labor 12,000 13,200 1,200 U
Utilities 10,000 10,500 500 U
Maintenance 6,000 5,300 700 F
Depreciation 8,000 8,000 —
Property taxes 4,000 4,500 500 U
Total costs $60,000 $60,100 $ 100 U

Ex. 184
The Ace Division, a profit center of Berek Engineering Company, reported the following data for the first quarter of 2008:
Sales $6,000,000
Variable costs 4,200,000
Controllable direct fixed costs 800,000
Noncontrollable direct fixed costs 530,000
Indirect fixed costs 200,000

Instructions
(a) Prepare a performance report for the manager of the Ace Division.
(b) What is the best measure of the manager's performance? Why?
(c) How would the responsibility report differ if the division was an investment center?

Solution 184 (15–20 min.)


(a) BEREK ENGINEERING COMPANY
Ace Division
Management Performance Report
For the Quarter Ended March 31, 2008
Sales..................................................................................................................... $6,000,000
Variable costs....................................................................................................... 4,200,000
Contribution margin.............................................................................................. 1,800,000
Controllable fixed costs........................................................................................ 800,000
Controllable margin.............................................................................................. $1,000,000

(b) Controllable margin is the best measure of the manager's performance because this amount equals the excess of controllable
revenues over controllable costs.

(c) For an investment center, the responsibility report would also show the return on investment for the period.

Ex. 185
RTO Rental Company reported the following:
Beginning of year operating assets $2,200,000
End of year operating assets 2,000,000
Contribution margin 1,000,000
Sales 5,000,000
Controllable fixed costs 643,000
Its required return is 10%.

Instructions
Compute the company’s ROI.

Solution 185 (3 min.)


($1,000,000 – $643,000) ÷ [($2,200,000 + $2,000,000) ÷ 2] = 17%

Ex. 186
Reese Company has two investment centers and has developed the following information:
Department A Department B
Departmental controllable margin $120,000 ?
Average operating assets ? $400,000
Sales 800,000 250,000
ROI 10% 12%

Instructions
Answer the following questions about Department A and Department B.
1. What was the amount of Department A's average operating assets? $____________.
2. What was the amount of Department B's controllable margin? $____________.
3. If Department B is able to reduce its operating assets by $100,000, Department B's new ROI would be ____________.
4. If Department A is able to increase its controllable margin by $60,000 as a result of reducing variable costs, Department A's new
ROI would be _________________.

Solution 186 (8–12 min.)


1. $1,200,000 ($120,000 ÷ .10)
2. $48,000 ($400,000 × .12)
3. 16% [$48,000 ÷ ($400,000 – $100,000)]
4. 15% [($120,000 + $60,000) ÷ $1,200,000]

Ex. 187
The Appliance Division of Malone Manufacturing Company reported the following results for 2008:
Sales $4,000,000
Variable costs 3,200,000
Controllable fixed costs 300,000
Average operating assets 2,000,000

Management is considering the following independent alternative courses of action in 2009 in order to maximize the return on
investment for the division.

1. Reduce controllable fixed costs by 20% with no change in sales or variable costs.
2. Reduce average operating assets by 20% with no change in controllable margin.
3. Increase sales $400,000 with no change in the contribution margin percentage.

Instructions
(a) Compute the return on investment for 2008.
(b) Compute the expected return on investment for each of the alternative courses of action.

Solution 187 (15–20 min.)


(a) Controllable margin
Return on investment = ————————————
Average operating assets

$500,000
2008 ROI = —————— = 25%
$2,000,000

$560,000 (a)
(b) 1. ——————— = 28%
$2,000,000

$500,000
2. ———————— = 31.3%
$1,600,000 (b)

$580,000 (c)
3. ——————— = 29%
$2,000,000

(a) $500,000 + ($300,000 × 20%) = $560,000.

(b) $2,000,000 – ($2,000,000 × .20) = $1,600,000.

$4,000,000 – $3,200,000
(c) Contribution margin 20% (————————————);
$4,000,000

$500,000 + ($400,000 × 20%) = $580,000.

Ex. 188
Data for the following subsidiaries of Timmons Company, which are operated as investment centers, are as follows:
Black Company Greer Company
Sales $3,000,000 $2,000,000
Controllable margin (1) (3)
Average operating assets (2) 4,000,000
Contribution margin 1,200,000 800,000
Controllable fixed costs 500,000 200,000
Return on Investment 10% (4)

Instructions
Compute the missing amounts using the ROI formula.

Solution 188 (9–14 min.)


(1) Controllable margin ($1,200,000 – $500,000) = $700,000
(2) Average operating assets ($700,000 ÷ .10) = $7,000,000
(3) Controllable margin ($800,000 – $200,000) = $600,000
(4) ROI ($600,000 ÷ $4,000,000) = 15%

Ex. 189
The data for an investment center is given below.
1/1/08 12/31/08
Current assets $ 300,000 $ 500,000
Plant assets 3,000,000 4,000,000
Idle plant assets 250,000 330,000
Land held for future use 1,200,000 1,200,000

The controllable margin is $780,000.

Instructions
What is the return on investment for the center for 2008?

Solution 189 (4–5 min.)


ROI = Controllable margin ÷ Average operating assets

Plant assets ($3,000,000 + $4,000,000) ÷ 2 = $3,500,000


Average current assets ($300,000 + $500,000) ÷ 2 = 400,000
$3,900,000
Note: Idle plant assets and land held for future use are not included in average operating assets.

ROI = $780,000 ÷ $3,900,000 = 20%


COMPLETION STATEMENTS

190. The use of budgets in controlling operations is known as ________________.


191. A major aspect of budgeting control is the use of budget reports that compare _____________________ with
_______________________.
192. In analyzing differences from planned objectives, management may take ___________________, or it could decide to modify
___________________.
193. The master budget is a __________________ budget which is based on operating at one budgeted activity level.
194. A __________________ budget projects budget data for various levels of activity.
195. Total ________________ costs will be the same on the master budget and on a flexible budget which reflects the actual level
of activity.
196. Under ___________________ accounting, the evaluation of a manager's performance is based on the costs and revenues
directly under that manager's control.
197. A cost is __________________ at a given level of managerial responsibility if a manager has the authority to incur the cost in
a given time period.
198. In general, costs ____________________ directly by the level of responsibility are _______________, whereas costs that are
____________________ to the responsibility level are __________________.
199. Responsibility centers may be classified into three types: (1)____________________, (2)___________________ and,
(3)____________________.
200. The primary basis for evaluating the performance of a manager of an investment center is _________________.
201. Return on investment is calculated by dividing _________________________ by ________________________.

Answers to Completion Statements


190. budgetary control 197. controllable
191. actual results, planned objectives 198. incurred, controllable, allocated,
192. corrective action, future plans noncontrollable
193. static 199. cost centers, profit centers, investment centers
194. flexible 200. return on investment (ROI)
195. fixed 201. controllable margin, average operating assets
196. responsibility

MATCHING

202. Match the items below by entering the appropriate code letter in the space provided.

A. Budgetary control G. Responsibility reporting system


B. Static budget H. Return on Investment
C. Flexible budget I. Profit center
D. Responsibility accounting J. Investment center
E. Controllable costs K. Indirect fixed costs
F. Management by exception L. Direct fixed costs

____ 1. The review of budget reports by top management directed entirely or primarily to differences between actual results and
planned objectives.

____ 2. A part of management accounting that involves accumulating and reporting revenues and costs on the basis of the
individual manager who has the authority to make the day-to-day decisions about the items.

____ 3. The preparation of reports for each level of responsibility shown in the company's organization chart.
____ 4. A projection of budget data at one level of activity.

____ 5. Costs that a manager has the authority to incur within a given period of time.

____ 6. The use of budgets to control operations.

____ 7. A projection of budget data for various levels of activity.

____ 8. A responsibility center that incurs costs, generates revenues, and has control over the investment funds available for use.

____ 9. Costs that relate specifically to a responsibility center and are incurred for the sole benefit of the center.

____ 10. A responsibility center that incurs costs and also generates revenues.

____ 11. Costs which are incurred for the benefit of more than one profit center.

____ 12. A measure of the profitability of an investment center computed by dividing controllable margin (in dollars) by average
operating assets.

Answers to Matching
1. F 7. C
2. D 8. J
3. G 9. L
4. B 10. I
5. E 11. K
6. A 12. H

CHAPTER 18
RESPONSIBILITY ACCOUNTING AND TRANSFER PRICING ANSWER: b EASY
IN DECENTRALIZED ORGANIZATIONS
4. In a decentralized company in which
MULTIPLE CHOICE divisions may buy goods from one another, the transfer
pricing system should be designed primarily to
1. Which of the following is more characteristic
of a decentralized than a centralized business structure?
a. increase the consolidated value of inventory.
b. allow division managers to buy from
a. The firm’s environment is stable. outsiders.
b. There is little confidence in lower-level c. minimize the degree of autonomy of division
management to make decisions. managers.
c. The firm grows very quickly. d. aid in the appraisal and motivation of
d. The firm is relatively small. managerial performance.

ANSWER: c EASY ANSWER: d EASY


5. When the majority of authority is maintained by top
2. Costs of decentralization include all of the following management personnel, the organization is said to be
except
a. centralized.
a. more elaborate accounting control systems. b. decentralized.
b. potential costs of poor decisions. c. composed of cost centers.
c. additional training costs. d. engaged in transfer pricing activities.
d. slow response time to changes in local
conditions. ANSWER: a EASY

ANSWER: d EASY 6. What term identifies an accounting system in


which the operations of the business are broken down into
3. Transfer pricing is primarily incurred in reportable segments, and the control function of a
foreperson, sales manager, or supervisor is emphasized?
a. foreign corporations exporting their products.
b. decentralized organizations.
c. multinational corporations domiciled in the a. responsibility accounting
U.S. b. operations-research accounting
d. closely held corporations. c. control accounting
d. budgetary accounting c. in or out at cost-based transfer price.
d. to other divisions in the same company.
ANSWER: a EASY
ANSWER: b MEDIUM
7. In a responsibility accounting system, costs are an organization
classified into categories on the basis of
a. affect the motivation of subunit managers to
a. fixed and variable costs. transact with one another.
b. prime and overhead costs. b. always promote goal congruence.
c. administrative and nonadministrative costs. c. are less motivating to managers than overall
d. controllable and noncontrollable costs. organizational goals.
d. must be the same for all managers to
ANSWER: d EASY eliminate suboptimization.

8. When used for performance evaluation, periodic ANSWER: a MEDIUM


internal reports based on a responsibility accounting
system should not 14. A management decision may be beneficial for a given
profit center, but not for the entire company. From
a. be related to the organization chart. the overall company viewpoint, this decision would
b. include allocated fixed overhead. lead to
c. include variances between actual and
budgeted controllable costs. a. goal congruence.
d. distinguish between controllable and b. centralization.
noncontrollable costs. c. suboptimization.
d. maximization.
ANSWER: b EASY
ANSWER: c EASY
9. A ___________ is a document that reflects
the revenues and/or costs that are under the control of a 15. A major benefit of cost-based transfers is that
particular manager.
a. it is easy to agree on a definition of cost.
b. costs can be measured accurately.
a. quality audit report c. opportunity costs can be included.
b. responsibility report d. they provide incentives to control costs.
c. performance evaluation report
d. project report ANSWER: c MEDIUM
ANSWER: b EASY 16. An internal reconciliation account is not required for
internal transfers based on
10. The cost object under the control of a manager is
called a(n) __________________ center. a. market value.
b. dual prices.
a. cost c. negotiated prices.
b. revenue d. cost.
c. responsibility
d. investment ANSWER: d MEDIUM
ANSWER: c EASY 13. Performance evaluation measures in
17. The most valid reason for using something
11. In evaluating the performance of a profit center other than a full-cost-based transfer price between units of
manager, he/she should be evaluated on a company is because a full-cost price
a. all revenues and costs that can be traced
directly to the unit. a. is typically more costly to implement.
b. all revenues and costs under his/her control. b. does not ensure the control of costs of a
c. the variable costs and the revenues of the supplying unit.
unit. c. is not available unless market-based prices
d. the same costs and revenues on which the are available.
unit is evaluated. d. does not reflect the excess capacity of the
supplying unit.
ANSWER: b EASY
ANSWER: b MEDIUM
12. If a division is set up as an autonomous profit center,
then goods should not be transferred 18. To avoid waste and maximize efficiency
when transferring products among divisions in a
a. in at a cost-based transfer price. competitive economy, a large diversified corporation
b. out at a cost-based transfer price. should base transfer prices on
24. The minimum potential transfer price is determined by
a. variable cost.
b. market price. a. incremental costs in the selling division.
c. full cost. b. the lowest outside price for the good.
d. production cost. c. the extent of idle capacity in the buying
division.
ANSWER: b MEDIUM d. negotiations between the buying and selling
division.
19. A transfer pricing system is also known as
ANSWER: a EASY
a. investment center accounting.
b. a revenue allocation system. 25. As the internal transfer price is increased,
c. responsibility accounting.
d. a charge-back system. a. overall corporate profits increase.
b. profits in the buying division increase.
ANSWER: d EASY c. profits in the selling division increase.
d. profits in the selling division and the overall
20. The maximum of the transfer price negotiation range corporation increase.
is
ANSWER: c EASY
a. determined by the buying division.
b. set by the selling division. 26. In an internal transfer, the selling division records the
c. influenced only by internal cost factors. event by crediting
d. negotiated by the buying and selling division.
a. accounts receivable and CGS.
ANSWER: a EASY b. CGS and finished goods.
c. finished goods and accounts receivable.
21. The presence of idle capacity in the selling division d. finished goods and intracompany sales.
may increase
ANSWER: d EASY
a. the incremental costs of production in the
selling division. 27. In an internal transfer, the buying division records the
b. the market price for the good. transaction by
c. the price that a buying division is willing to
pay on an internal transfer. a. debiting accounts receivable.
d. a negotiated transfer price. b. crediting accounts payable.
c. debiting intracompany CGS.
ANSWER: a MEDIUM d. crediting inventory.

22. Which of the following is a consistently ANSWER: b EASY


desirable characteristic in a transfer pricing system?
28. Top management can preserve the
autonomy of division managers and encourage an optimal
a. system is very complex to be the most fair to level of internal transactions by
the buying and selling units
b. effect on subunit performance measures is
not easily determined a. selecting performance evaluation measures
c. system should reflect organizational goals that are consistent with the achievement of
d. transfer price remains constant for a period overall corporate goals.
of at least two years b. selecting division managers who are most
concerned about their individual
ANSWER: c MEDIUM performance.
c. prescribing transfer prices between
23. With two autonomous division managers, the segments.
price of goods transferred between the divisions needs to d. setting up all organizational units as revenue
be approved by centers.

ANSWER: a MEDIUM
a. corporate management.
b. both divisional managers. individual departments, interdepartmental transfers of a
c. both divisional managers and corporate product should preferably be made at prices
management.
d. corporate management and the manager of
the buying division. a. equal to the market price of the product.
b. set by the receiving department.
ANSWER: b EASY c. equal to fully-allocated costs of the
producing department.
d. equal to variable costs to the producing c. goal congruence.
department. d. operational auditing techniques.

ANSWER: a EASY ANSWER: d MEDIUM

30. Allocating service department costs to revenue-


producing departments is an alternative to Use the following information for questions 36–40.

a. responsibility accounting. Office Products Inc. manufactures and sells various high-tech
b. the use of profit centers. office automation products. Two divisions of Office Products
c. the use of cost centers. Inc. are the Computer Chip Division and the Computer
d. a transfer pricing system. Division. The Computer Chip Division manufactures one
product, a “super chip,” that can be used by both the Computer
ANSWER: d MEDIUM Division and other external customers. The following
information is available on this month’s operations in the
31. External factors considered in setting transfer prices Computer Chip Division:
in multinational firms typically do not include
SELLING PRICE PER CHIP $50
a. the corporate income tax rates in host Variable costs per chip $20
countries of foreign subsidiaries. Fixed production costs $60,000
b. foreign monetary exchange risks. Fixed SG&A costs $90,000
c. environmental policies of the host countries Monthly capacity 10,000
of foreign subsidiaries. chips
d. actions of competitors of foreign External sales 6,000
subsidiaries. chips
Internal sales 0
ANSWER: c MEDIUM chips

32. Corporate taxes and tariffs are particular transfer- Presently the Computer Division purchases no chips from the
pricing concerns of Computer Chips Division, but instead pays $45 to an external
supplier for the 4,000 chips it needs each month.
a. investment centers.
b. multinational corporations. 36. Assume that next month’s costs and levels
c. division managers. of operations in the Computer and Computer Chip
d. domestic corporations involved in importing Divisions are similar to this month. What is the minimum
foreign goods. of the transfer price range for a possible transfer of the
super chip from one division to the other?
ANSWER: b EASY

29. To evaluate the performance of a. $50


b. $45
33. When managers attempt to cause actual c. $20
results to conform to planned results, this is known as d. $35

ANSWER: c MEDIUM
a. efficiency.
b. effectiveness. 37. Assume that next month’s costs and levels
c. conformity. of operations in the Computer and Computer Chip
d. goal congruence. Divisions are similar to this month. What is the maximum
of the transfer price range for a possible transfer of the
ANSWER: b EASY chip from one division to the other?
34. Which of the following would not be considered a
critical success factor? a. $50
b. $45
a. quality c. $35
b. cost control d. $30
c. customer service
d. all of the above are critical success factors ANSWER: b MEDIUM
38. Two possible transfer prices (for 4,000 units)
ANSWER: d EASY are under consideration by the two divisions: $35 and $40.
Corporate profits would be ___________ if $35 is selected
35. The costs of service departments can be assigned to as the transfer price rather than $40.
other divisions through the use of

a. cost centers. a. $20,000 larger


b. transfer prices. b. $40,000 larger
c. $20,000 smaller
d. the same ANSWER: b MEDIUM

ANSWER: d MEDIUM 42. What is the minimum of the transfer price range for a
transfer between the two divisions?
39. If a transfer between the two divisions is
a. $96
arranged next period at a price (on 4,000 units of b. $90
c. $70
super chips) of $40, total profits in the Computer Chip d. $106

division will ANSWER: a MEDIUM

43. If the two divisions agree to transact with one another,


corporate profits will
a. rise by $20,000 compared to the prior period.
b. drop by $40,000 compared to the prior a. drop by $30,000 per month.
period. b. rise by $20,000 per month.
c. drop by $20,000 compared to the prior c. rise by $50,000 per month.
period. d. rise or fall by an amount that depends on the
d. rise by $80,000 compared to the prior period. level of the transfer price.
ANSWER: d MEDIUM ANSWER: c MEDIUM
40. Assume, for this question only, that the 44–47.
Computer Chip Division is selling all that it can produce to
external buyers for $50 per unit. How would overall Bigole Corp. produces various products used in the
corporate profits be affected if it sells 4,000 units to the construction industry. The Plumbing Division produces and
Computer Division at $45? (Assume that the Computer sells 100,000 copper fittings each month. Relevant information
Division can purchase the super chip from an outside for last month follows:
supplier for $45.)
Total sales (all external) $250,000
a. no effect Expenses (all on a unit base):
b. $20,000 increase Variable manufacturing $0.50
c. $20,000 decrease Fixed manufacturing .25
d. $90,000 increase Variable selling .30
Fixed selling .40
ANSWER: c MEDIUM Variable G&A .15
Fixed G&A .50
Use the following information for questions 41–43. Total $2.10

The Motor Division of Super Truck Co. uses 5,000 carburetors Top-level managers are trying to determine how a transfer
per month in its production of automotive engines. It presently price can be set on a transfer of 10,000 of the copper fittings
buys all of the carburetors it needs from two outside suppliers from the Plumbing Division to the Bathroom Products Division.
at an average cost of $100. The Carburetor Division of Super
Truck Co. manufactures the exact type of carburetor that the 44. A transfer price based on variable cost will be set at
Motor Division requires. The Carburetor Division is presently ___________ per unit.
operating at its capacity of 15,000 units per month and sells all
of its output to a foreign car manufacturer at $106 per unit. Its a. $0.50
cost structure (on 15,000 units) is: b. $0.80
c. $0.95
VARIABLE PRODUCTION COSTS $70 d. $0.75

Variable selling costs 10 ANSWER: c MEDIUM


All fixed costs 10
45. A transfer price based on full production cost would
Assume that the Carburetor Division would not incur any be set at ___________ per unit.
variable selling costs on units that are transferred internally.
a. $0.75
41. What is the maximum of the transfer price range for a b. $2.10
transfer between the two divisions? c. $1.45
d. $1.60
a. $106
b. $100 ANSWER: a MEDIUM
c. $90
d. $70 46. A transfer price based on market price would be set at
___________ per unit.
d. compare alternatives for decision making.
a. $2.10
b. $2.50 ANSWER: c MEDIUM
c. $1.60
d. $2.25 51. A service department provides specific functional
tasks for other internal units. Which of the following
ANSWER: b MEDIUM activities would not be engaged in by a service
Use the following information for questions department?
47. If the Plumbing Division is operated as an
autonomous investment center and its capacity is a. purchasing
100,000 fittings per month, the per-unit transfer price b. warehousing
is not likely to be below c. distributing
d. manufacturing
a. $0.75.
b. $1.60. ANSWER: d EASY
c. $2.10.
d. $2.50. 52. All of the following objectives are reasons to allocate
service department costs to compute full cost except
ANSWER: d MEDIUM to

48. A company has two divisions, A and B, each a. provide information on cost recovery.
operated as a profit center. A charges B $35 per unit for b. abide by regulations that may require full
each unit transferred to B. Other data follow: costing in some instances.
c. provide information on controllable costs.
d. reflect production’s “fair share” of costs.
A’s variable cost per unit $30
A’s fixed costs $10,000 ANSWER: c MEDIUM
A’s annual sales to B 5,000
units 53. All of the following objectives are reasons that service
A’s annual sales to outsiders 50,000 department allocations can motivate managers
units except to
A is planning to raise its transfer price to $50 per unit. a. instill a consideration of support costs in
Division B can purchase units at $40 each from outsiders, production managers.
but doing so would idle A’s facilities now committed to b. encourage production managers to help
producing units for B. Division A cannot increase its sales service departments control costs.
to outsiders. From the perspective of the company as a c. encourage the usage of certain services.
whole, from whom should Division B acquire the units, d. determine divisional profitability.
assuming B’s market is unaffected?
ANSWER: d MEDIUM
a. outside vendors 54. Which of the following is a reason for
b. Division A, but only at the variable cost per allocating service department costs and thereby motivating
unit management?
c. Division A, but only until fixed costs are
covered, then should purchase from outside a. provides for cost recovery
vendors b. provides relevant information in determining
d. Division A, in spite of the increased transfer corporatewide profits generated by
price alternative actions
c. meets regulations in some pricing instances
ANSWER: d MEDIUM d. reflects usage of services on a fair and
equitable basis
49. A service department includes which of the following?
ANSWER: d MEDIUM
Payroll Production
a. yes no 55. Service departments provide functional tasks for
b. yes yes which of the following?
c. no yes
d. no no Internal units External units
a. no no
ANSWER: a EASY b. yes no
50. Indirect costs should be allocated for all of the c. no yes
following reasons except to d. yes yes
a. motivate managers. ANSWER: b EASY
b. determine the full cost of a product.
c. motivate general administration.
56. After service department costs have been b. direct method.
allocated, what is the final step in determining full product c. reciprocal method.
cost? d. none of the above.

ANSWER: b EASY
a. determine direct material cost
b. determine overhead application rates for 62. Which service department cost allocation
revenue-producing areas method assigns indirect costs to cost objects after
c. determine direct labor cost considering some of the interrelationships of the cost
d. determine total service department costs objects?
ANSWER: b EASY
a. step method
57. Which of the following is not an objective for b. indirect method
computing full cost? c. algebraic method
d. direct method
a. to reflect production’s “fair share” of costs
b. to instill a consideration of support costs ANSWER: a EASY
c. to reflect usage of services on a fair and
equitable basis 63. Which service department cost allocation method
d. to provide for cost recovery utilizes a “benefits-provided” ranking?

ANSWER: c MEDIUM a. algebraic method


b. indirect method
58. A rational and systematic allocation base for service c. step method
department costs should reflect the cost accountant’s d. direct method
consideration of all of the following except
ANSWER: c EASY
a. the ability of revenue-producing departments
to bear the allocated costs. 64. Which service department cost allocation
b. the benefits received by the revenue- method assigns indirect costs to cost objects after
producing department from the service considering interrelationships of the cost objects?
department.
c. a causal relationship between factors in the
revenue-producing department and costs Algebraic method Step method
incurred in the service department. a. no no
d. all of the above are considerations. b. no yes
c. yes yes
ANSWER: d MEDIUM d. yes no

59. Which of the following is not a method for allocating ANSWER: c EASY
service department costs?
65. Which of the following methods of assigning
a. step method indirect service department costs recognizes on a partial
b. indirect method basis the reciprocal relationships among the departments?
c. direct method
d. algebraic method a. step method
b. direct method
ANSWER: b EASY c. indirect method
d. algebraic method
60. Which service department cost allocation
method assigns costs directly to revenue-producing areas ANSWER: a EASY
with no other intermediate cost pools or allocations?
66. The most accurate method for allocating service
a. step method department costs is the
b. indirect method
c. algebraic method a. step method.
d. direct method b. direct method.
c. algebraic method.
ANSWER: d EASY d. none of the above.

61. The overhead allocation method that ANSWER: c EASY


allocates service department costs without consideration
of services rendered to other service departments is the 67. The criteria that are most often used to decide on
allocation bases are?

a. step method.
Benefits received Fairness
Causal relationships a. step method
a. yes yes b. indirect method
b. yes yes c. algebraic method
c. no yes d. direct method
d. no no
ANSWER: c EASY
ANSWER: b MEDIUM
73. An automotive company has three divisions.
68. To identify costs that relate to a specific product, an One division manufactures new replacements parts for
allocation base should be chosen that automobiles, another rebuilds engines, and the third does
repair and overhaul work on a line of trucks. All three
a. does not have a cause-and-effect divisions use the services of a central payroll department.
relationship. The best method of allocating the cost of the payroll
b. has a cause-and-effect relationship. department to the various operating divisions is
c. considers variable costs but not fixed costs.
d. considers direct material and direct labor but
not manufacturing overhead. a. total labor hours incurred in the divisions.
b. value of production in the divisions.
ANSWER: b EASY c. direct labor costs incurred in the divisions.
d. machine hours used in the divisions.
69. The fixed costs of service departments
should be allocated to production departments based on ANSWER: a MEDIUM
74. The allocation of general overhead control costs to
operating departments can be least justified in
a. actual short-run utilization based on determining
predetermined rates.
b. actual short-run units based on actual rates. a. income of a product or functional unit.
c. the service department’s expected costs b. costs for making management’s decisions.
based on expected long-run use of capacity. c. costs of products sold.
d. the service department’s actual costs based d. costs for government’s “cost-plus” contracts.
on actual utilization of services.
ANSWER: b MEDIUM
ANSWER: d MEDIUM
Use the following information for questions 75–84.

70. Which service department cost allocation Gates Co. has three production departments A, B, and C.
method provides for reciprocal allocation of service costs Gates also has two service departments, Administration and
among the service department as well as to the revenue Personnel. Administration costs are allocated based on value
producing departments? of assets employed, and Personnel costs are allocated based
on number of employees. Assume that Administration
provides more service to the other departments than does the
a. algebraic method Personnel Department.
b. indirect method
c. step method Dept. Direct Costs Employees
d. direct method Asset Value
Admin. $900,000 25
ANSWER: a EASY $450,000
Personnel 350,000 10
71. The algebraic method 600,000
A 700,000 15
a. considers all interrelationships of the 300,000
departments and reflects these relationships B 200,000 5
in equations. 150,000
b. does not consider interrelationships of the C 250,000 10
departments nor reflect these relationships in 800,000
equations.
c. is also referred to as the “benefits-provided” 75. Using the direct method, what amount of
ranking method. Administration costs is allocated to A (round to the nearest
d. is not a service department cost allocation dollar)?
method.

ANSWER: a EASY a. $216,000


b. $150,000
72. Which service department cost allocation c. $288,000
method considers all interrelationships of the departments d. $54,000
and reflects these relationships in equations?
ANSWER: a MEDIUM c. $291,892
d. $72,973
76. Using the direct method, what amount of
Personnel costs is allocated to B (round to the nearest ANSWER: a MEDIUM
dollar)?
82. Assume that Administration costs have been
allocated and the balance in Personnel is $860,000. What
a. $50,000 amount is allocated to A (round to the nearest dollar)?
b. $43,750
c. $26,923
d. $58,333 a. $213,964
b. $106,982
ANSWER: d MEDIUM c. $430,000
d. $0
77. Using the direct method, what amount of ANSWER: c MEDIUM
Administration costs is allocated to C (round to the
nearest dollar)? 83. Assume that Administration costs have been
allocated and the balance in Personnel is $860,000. What
a. $576,000 amount is allocated to B (round to the nearest dollar)?
b. $54,000
c. $108,000
d. $150,000 a. $213,964
b. $430,000
ANSWER: a MEDIUM c. $106,982
d. $143,333
78. Using the step method, what amount of
Administration costs is allocated to Personnel (round to ANSWER: d MEDIUM
the nearest dollar)?
84. Assume that Administration costs have been
allocated and the balance in Personnel is $860,000. What
a. $72,973 amount is allocated to C (round to the nearest dollar)?
b. $291,892
c. $145,946
d. $389,189 a. $213,964
b. $430,000
ANSWER: b MEDIUM c. $286,667
d. $143,333
79. Using the step method, what amount of
Administration costs is allocated to A (round to the nearest ANSWER: c MEDIUM
dollar)?
Use the following information for questions 85–90.
a. $72,973
b. $291,892 Brooks Co. has two service departments: Data Processing and
c. $145,946 Administration/Personnel. The company also has three
d. $389,189 divisions: X, Y, and Z. Data Processing costs are allocated
based on hours of use and Administration/Personnel costs are
ANSWER: c MEDIUM allocated based on number of employees.

80. Using the step method, what amount of Direct costs Employees
Administration costs is allocated to B (round to the nearest Hours of use
dollar)? Admin/Per. $400,000 10
3,300
Data Pro. 850,000 5
a. $72,973 1,100
b. $291,892 X 450,000 30
c. $145,946 1,800
d. $389,189 Y 300,000 15
2,200
ANSWER: a MEDIUM Z 550,000 25
4,500
81. Using the step method, what amount of
Administration costs is allocated to C (round to the nearest Assume that Data Processing provides more service than
dollar)? Administration/Personnel.

a. $389,189
b. $145,946
85. Using the direct method, what amount of 90. Assume that Data Processing costs have
Data Processing costs is allocated to X (round to the been allocated and the balance in Administration is
nearest dollar)? $600,000. Using the step method, what amount is
allocated to Z?
a. $180,000
b. $129,661 a. $200,000
c. $0 b. $112,500
d. $84,706 c. $214,286
d. $225,000
ANSWER: a MEDIUM
ANSWER: c MEDIUM
86. Using the direct method, what amount of
Data Processing costs is allocated to Y (round to the Use the following information for questions 91 and 92.
nearest dollar)?
Blake Company has two service departments: Data Processing
and Personnel. Data Processing provides more service than
a. $158,475 does Personnel. Blake also has two production departments:
b. $0 A and B. Data Processing costs are allocated on the basis of
c. $220,000 assets used while Personnel costs are allocated based on the
d. $103,529 number of employees.
ANSWER: c MEDIUM Direct costs Employees
Assets used
87. Using the direct method, what amount of Data Pro. $1,000,000 15
Data Processing costs is allocated to Z (round to the $700,000
nearest dollar)? Pers. 300,000 8
230,000
a. $211,765 A 500,000 12
b. $0 125,000
c. $152,542 B 330,000 20
d. $450,000 220,000

ANSWER: d MEDIUM 91. Using the direct method, what amount of


88. Assume that Data Processing costs have been Data Processing costs is allocated to A (round to the
allocated and the balance in Administration is $600,000. Using nearest dollar)?
the step method, what amount is allocated to X?
a. $362,319
a. $257,143 b. $637,681
b. $112,500 c. $253,623
c. $200,000 d. $446,377
d. $187,500
ANSWER: a MEDIUM
ANSWER: a MEDIUM
92. Using the direct method, what amount of
89. Assume that Data Processing costs have Personnel costs is allocated to B (round to the nearest
been allocated and the balance in Administration is dollar)?
$600,000. Using the step method, what amount is
allocated to Y?
a. $123,750
b. $206,250
a. $225,000 c. $112,500
b. $128,571 d. $187,500
c. $187,500
d. $200,000 ANSWER: d MEDIUM
ANSWER: b MEDIUM

Use the following information for questions 93–96.

Hartwell Company distributes its service department overhead costs directly to producing departments without allocation to the other
service departments. Information for January is presented here.

Maintenance Utilities
Overhead costs incurred $18,700 $9,000
Service provided to:
Maintenance Dept. 10%
Utilities Dept. 20%
Producing Dept. A 40% 30%
Producing Dept. B
40% 60% d. None of the above.

93. The amount of Utilities Department costs ANSWER: b MEDIUM


distributed to Dept. B for January should be (rounded to
the nearest dollar) 95. Using the step method, how much of
Hartwell’s Utilities Department cost is allocated between
Departments A and B?
a. $3,600.
b. $4,500.
c. $5,400. a. $9,900
d. $6,000. b. $10,800
c. $12,740
ANSWER: d MEDIUM d. $27,700

94. Assume instead Hartwell Company distributes the ANSWER: c MEDIUM


service department’s overhead costs based on the
step method. Maintenance provides more service 96. Assume that Hartwell Company distributes
than does Utilities. Which of the following is true? service department overhead costs based on the algebraic
method. What would be the formula to determine the total
a. Allocate maintenance expense to maintenance costs?
Departments A and B.
b. Allocate maintenance expense to
Departments A and B and the Utilities a. M = $18,700 + .10U
Department. b. M = $9,000 + .20U
c. Allocate utilities expense to the Maintenance c. M = $18,700 + .30U + .40A + .40B
Department and Departments A d. M = $27,700 + .40A + .40B
and B.
ANSWER: a MEDIUM
Use the following information for questions 10–14.

Wire Division of XS Steel Corporation produces “bales” of steel wire that are used in various commercial applications. The bales sell
for an average of $20 each and Wire Division has the capacity to produce 10,000 bales per month. Consumer Products Division of XS
Steel uses approximately 2,000 bales of steel wire each month in its production of various appliances. The operating information for
Wire Division at its present level of operations (8,000 bales per month) follows:

Sales (all external) $160,000


Variable costs per bale:
Production $5
Selling 2
G&A 3
Fixed costs per bale (based on a 10,000 unit capacity):
Production $2
Selling 3
G&A 4

Consumer Products Division currently pays $15 per bale for wire obtained from its external supplier.

10. If 2,000 bales are transferred in one month to Consumer Products Division at $10 per bale, what would be the
profit/loss of Wire Products Division?

ANSWER: The $10 per unit would equal the Division’s variable costs ($5 + 2 + 3 = $10), so the contribution margin per
unit is zero. Thus, only the 8,000 units of external sales would generate a contribution margin of $80,000 (8,000 × $10) to
cover fixed costs of $90,000 (10,000 × $9). So the Division would show a $10,000 loss.

MEDIUM

11. For the Wire Products Division to operate at break-even level, what would it need to charge for the production and
transfer of 2,000 bales to the Consumer Products Division? Assume all variable costs indicated will be incurred by the Wire Division.

ANSWER:
Total fixed costs to Wire are:
Production $2 × 10,000 = $20,000
Selling $3 × 10,000 = 30,000
G&A $4 × 10,000 = 40,000
Total $90,000

Less: Contrib.Margin on Regular Business


[$20 – (5 + 2 + 3)] × 8,000 (80,000 )
Unrecovered Fixed Costs $10,000

which must be covered by CM of inside sales =


Trans.Price × Vol. = SP – [(5 + 2 + 3) × 2,000]
SP = $15

MEDIUM

12. If Wire Products Division transferred 2,000 wire bales to the Consumer Products Division at 200 percent of full
absorption cost, what would be the transfer price?

ANSWER:
Full absorption cost: Variable Production Cost = $ 5
Fixed Production Cost = 2
Total full absorption cost $ 7
Doubled × 2
Transfer price $ 14

MEDIUM

13. If Consumer Products Division agrees to pay Wire Products Division $16 for 2,000 bales this month, what would be
Consumer’s change in total profits?

ANSWER:
Proposed transfer price per unit $ 16
Consumer’s current market purchase price per unit 15
Increase in cost per unit of wire to Consumer’s $ 1
Times units purchased × 2,000
Decrease in profit due to increased costs $2,000

MEDIUM
14. Assuming, for this question only, that Wire Products Division would not incur any variable G&A costs on internal sales, what is
the minimum price that it would consider accepting for sales of bales to Consumer Products Division?

ANSWER: Wire Division must cover its out of pocket costs or the relevant variable costs; the fixed costs are irrelevant
since they will be incurred regardless of this extra inside business. Thus, the total cost to be covered is $7 (production, $5;
selling, $2).

MEDIUM

Use the following information for questions 15–19.

Carpet Division of Building Products Inc. manufactures a single grade of residential grade carpeting. The division has the capacity to
produce 500,000 square yards of carpet each year. Its current costs and revenues are shown here:

Sales (400,000 square yards) $2,000,000


Variable costs per square yard:
Production $2.00
SG&A 1.00
Fixed costs per square yard (based on 500,000 yard capacity)
Production $0.50
SG&A 1.00

The Housing Division currently purchases 40,000 yards of carpeting (of the grade produced by the Carpet Division) each year at a cost
of $6.50 per square yard from an outside vendor.

15. If the autonomous Housing and Carpet Divisions enter negotiations on the internal transfer of 40,000 square yards of
carpeting, what is the maximum price that will be considered?

ANSWER: The maximum price or ceiling is the current purchase price of the buying division or $6.50 per yard.
MEDIUM

16. If the autonomous Housing and Carpet Divisions enter negotiations on the internal transfer of 40,000 square yards of
carpeting, what is the Carpet Division’s minimum price?

ANSWER: The minimum price acceptable to Carpet is its incremental cost of $3


($2 + $1) per square yard.

MEDIUM

17. If the Housing and Carpet Divisions agree on the internal transfer of 40,000 square yards of carpet at a price of $4.50
per square yard, how will the profits of the Housing Division be affected?

ANSWER:
Current external purchase price $6.50
Proposed transfer price 4.50
Reduction in purchase price per yard $2.00
Times yards acquired ×40,000
Increase in profits $80,000

MEDIUM

18. If the Housing and Carpet Divisions agree on the internal transfer of 40,000 square yards of carpet at a price of $4.00
per square yard, how will overall corporate profits be affected?

ANSWER:
Current outside purchase price per square yard $6.50
Carpet’s variable cost per square yard 3.00
Savings per square yard to Housing Division
& corporate $3.50
Times number square yards bought × 40,000
Savings to corporate and increase in profits $140,000

MEDIUM

19. Assume, for this question only, that Carpet Division is producing and selling 500,000 square yards of carpet to
external buyers at a price of $5 per square yard. What would be the effect on overall corporate profits if Carpet Division reduces
external sales of carpet by 40,000 square yards and transfers the 40,000 square yards of carpet to the Housing Division?

ANSWER: Since Carpet is operating at full capacity, it would lose the contribution margin on the 40,000 square yards.
However, the Housing Division would not have to buy externally. Thus,

Lost CM ($2 × 40,000 yd) = $(80,000 )


Gained CM ($3.50 × 40,000 yd) = 140,000
Net increase in corporate profits $ 60,000

MEDIUM

Use the following information for questions 20 and 21.

XY Corporation is comprised of two divisions: X and Y. X currently produces and sells a gear assembly used by the automotive
industry in electric window assemblies. X is currently selling all of the units it can produce (25,000 per year) to external customers for
$25 per unit. At this level of activity, X’s per unit costs are:

Variable:
Production $7
SG&A 2
Fixed:
Production 6
SG&A 5
Y Division wants to purchase 5,000 gear assemblies per year from X Division. Y Division currently purchases these units from an
outside vendor at $22 each.

20. What is the minimum price per unit that X Division could accept from Y Division for 5,000 units of the gear assembly
and be no worse off than currently?

ANSWER: X Division is operating and selling outside at full capacity so minimum price is equal to the variable cost to
make and sell plus the lost contribution margin from outside sales:

VC: Production $7
SGA 2 $ 9
Contribution margin 16
Selling price $25

MEDIUM

21. What will be the effect on overall corporate profits if the two divisions agree to an internal transfer of 5,000 units?

ANSWER: Corporate profits will decrease by forcing the transfer.

CM per units earned by X is from external sales $25 – [$7 + $2] $16
Times units to be sold ×5,000
Decrease in CM to X and XY Corp. $ 80,000
Net savings to buy internally
rather than externally [$22 – $9] $13
Times units to be purchased × 5,000
Savings by buying internally $ 65,000
Net effect on XY Corp. profits $ (15,000 )

MEDIUM

Use the following information for questions 22 and 23. Savings 5/26 × 530,000 = 101,923

Third Savings and Loan of Dallas has three departments that MEDIUM
generate revenue: loans, checking accounts, and savings
accounts. Third S & L has two service departments: 23. Using the step method, compute the amount
Administration/Personnel and Maintenance. The service allocated to each department from Maintenance.
departments provide service in the order of their listing. The
following information is available for direct costs.
Administration/ Personnel costs are best allocated based on ANSWER:
number of employees while Maintenance costs are best To allocate Admin./Pers. to Maintenance
allocated based on square footage occupied. 8/34 × $530,000 = $124,706(rounded)

Department Direct costs Employees Then, Maintenance balance is $450,000 + $124,706 =


Footage $574,706
Admin./Pers. $530,000 10
30,000 Then, allocate Maintenance :
Maintenance 450,000 8
16,500 Loans 45/97 × $574,706 = $266,616
Loans 900,000 15 Checking 10/97 × 574,706 = 59,248
45,000 Savings 42/97 × 574,706 = 248,842
Checking 600,000 6
10,000 MEDIUM
Savings 240,500 5
42,000

22. Using the direct method, compute the


amount allocated to each department from
Administration/Personnel.

ANSWER:
Loans 15/26 × $530,000 = $305,769
Checking 6/26 × 530,000 = 122,308
6. The Telemarketing Department of a residential
remodeling company would most likely be evaluated
as a:
A. cost center.
MULTIPLE CHOICE QUESTIONS B. revenue center.
C. profit center.
1. When managers of subunits throughout an D. investment center.
organization strive to achieve the goals set by top E. contribution center.
management, the result is:
A. goal congruence. Answer: B LO: 2 Type: RC
B. planning and control.
C. responsibility accounting. 7. If the head of a hotel's food and beverage operation is
D. delegation of decision making. held accountable for revenues and costs, the food
E. strategic control. and beverage operation would be considered a(n):
A. cost center.
Answer: A LO: 1 Type: RC B. revenue center.
C. profit center.
2. Which of the following is not an example of a D. investment center.
responsibility center? E. contribution center.
A. Cost center.
B. Revenue center. Answer: C LO: 2 Type: RC
C. Profit center.
D. Investment center. 8. Which of the following would have a low likelihood of
E. Contribution center. being organized as a profit center?
A. A movie theater of a company that operates a
Answer: E LO: 2 Type: RC chain of theaters.
B. A maintenance department that charges users
3. A manufacturer's raw-material purchasing department for its services.
would likely be classified as a: C. The billing department of an Internet Services
A. cost center. Provider (ISP).
B. revenue center. D. The mayor's office in a large city.
C. profit center. E. Both "C" and "D" above.
D. investment center.
E. contribution center. Answer: E LO: 2 Type: N

Answer: A LO: 2 Type: N 9. Easy-to-Use Software operates stores within five regions.
Regional managers are held accountable for marketing,
4. Hitchcock Corporation is in the process of overhauling advertising, and sales decisions, and all costs incurred
the performance evaluation system for its Los within their region. In addition, regional managers decide
Angeles manufacturing division, which produces and whether new stores will open, where the stores will be
sells parts that are popular in the aerospace industry. located, and whether the stores will lease or purchase the
Which of the following is least likely to be chosen to facilities. Store managers, in contrast, are accountable for
evaluate the overall operations of the Los Angeles marketing, advertising, and sales decisions, and costs
division? incurred within their stores. Ideally, on the basis of this
A. Cost center. information, what type of responsibility center should the
B. Responsibility center. software company use to evaluate its regions and stores?
C. Profit center. Regions Stores
D. Investment center. A. Profit center Profit center
E. The profit center and investment center are B. Profit center Cost center
equally unlikely to be chosen. C. Profit center Revenue center
D. Investment center Profit center
Answer: A LO: 2 Type: N E. Investment center Cost center

5. A cost center manager: Answer: D LO: 2 Type: N


A. does not have the ability to produce revenue.
B. may be involved with the sale of new marketing 10. Decentralized firms can delegate authority by
programs to clients. structuring an organization into responsibility centers.
C. would normally be held accountable for Which of the following organizational segments is
producing an adequate return on invested most like a totally independent, standalone business
capital. where managers are expected to "make it on their
D. often oversees divisional operations. own"?
E. may be the manager who oversees the A. Cost center.
operations of a retail store. B. Revenue center.
C. Profit center.
Answer: A LO: 2 Type: N D. Investment center.
E. Contribution center.
given to a department manager versus that reported to a
Answer: D LO: 2 Type: N company vice-president?
Department Manager Company Vice-President
11. A responsibility center in which the manager is held A. Somewhat detailed Somewhat detailed
accountable for the profitable use of assets and B. Somewhat detailed Somewhat summarized
capital is commonly known as a(n): C. Somewhat summarized Somewhat detailed
A. cost center. D. Somewhat summarized Somewhat summarized
B. revenue center. E. None of the above because department managers do not
C. profit center.
D. investment center. Answer: B LO: 3 Type: N
E. contribution center.
16. Leisure Time owns six hotels in Hawaii, collectively
Answer: D LO: 2 Type: RC known as the Hawaiian Division. The various hotels,
including the Surf & Sun, have operating departments
12. The Asian Division of a multinational manufacturing (such as Maintenance, Housekeeping, and Food and
organization would likely be classified as a: Beverage) that are evaluated as either cost centers or
A. cost center. profit centers. The Food and Beverage Department,
B. revenue center. for example, is a profit center, with activities divided
C. profit center. into three segments: Banquets and Catering,
D. investment center. Restaurants, and Kitchen. If Leisure Time uses a
E. contribution center. performance-reporting system that is based on
responsibility accounting, which of the following
Answer: D LO: 2 Type: N disclosures is likely to occur?
A. The detailed operating costs of the Surf & Sun's
13. Performance reports help managers: Kitchen Department will appear on the Hawaiian
A. use management by exception and effectively Division's performance report.
control operations. B. The Food and Beverage Department's profit will
B. decide whether a cost, profit, or investment appear on Kitchen's performance report.
center framework is appropriate. C. The profit of the Surf & Sun hotel will appear on
C. design their organizational hierarchy. the Hawaiian Division's performance report.
D. pinpoint trouble spots. D. The Food and Beverage profit at the Surf & Sun
E. by assisting with functions "A" and "D." will appear on Leisure Time's performance report.
E. The profit of the Surf & Sun hotel will appear on
Answer: E LO: 3 Type: RC Food and Beverage's performance report.

14. Consider the following statements about performance Answer: C LO: 3 Type: N
reports:
17. A cost pool is:
I. Performance reports provide feedback to A. a collection of homogeneous costs to be
managers and allow them to better control assigned.
operations. B. the combined result of decisions made by
II. Many performance reports have budget, different responsibility center managers.
actual, and variance data. C. the primary function of a responsibility
III. Performance reports are often structured accounting system.
around a firm's organizational hierarchy—that D. the amount of cost that has been allocated, say,
is, data relating to lower-level units (e.g., 10%, to a user department.
departments) are combined and flow into E. the tool used to allocate cost dollars to user
higher-level units (e.g., stores). departments.

Which of the above statements is (are) true? Answer: A LO: 4 Type: RC


A. I only.
B. I and II. 18. A cost object is:
C. I and III. A. a collection of costs to be assigned.
D. II and III. B. a responsibility center, product, or service to
E. I, II, and III. which cost is to be assigned.
C. the tool used to charge cost dollars to user
Answer: E LO: 3 Type: RC departments.
D. the primary function of a responsibility
15. Aloha Hotels owns numerous hotels on each of the accounting system.
Hawaiian Islands. The company's performance reporting E. a common cost.
system is structured around the firm's organizational
structure, with information flowing from operating Answer: B LO: 4 Type: RC
departments at a particular property and later respectively
grouped by individual hotel, island operation (i.e., 19. Kelly Corporation, with operations throughout the
division), and the company as a whole. Which of the country, will soon allocate corporate overhead to the
following best depicts the detail level of the information firm's various responsibility centers. Which of the
following is definitely not a cost object in this Answer: B LO: 4 Type: RC
situation?
A. The maintenance department. Use the following to answer questions 24-25:
B. Product no. 675.
C. Kelly Corporation. Management of Children Are Precious (CAP), an operator of
D. The Midwest division. day-care facilities, wants the firm's profit to be subdivided by
E. The telemarketing center. center. The firm's accountant has provided the following data:

Answer: C LO: 4 Type: N Actua


20. An allocation base for a cost pool should ideally be: Actual Budgeted Direc
A. machine hours. Center Revenue Revenue Costs
B. a cost object. Downtown $ 340,200 $ 320,000 $ 300,0
C. a common cost. Irvine 534,600 560,000 440,
D. a cost driver. H Beach 745,200 720,000 740,
E. direct labor, either cost or hours. Totals $1,620,000 $1,600,000 $1,480,

Answer: D LO: 4 Type: RC CAP's advertising, which is handled by the home office, is not
reflected in the preceding figures and amounted to $60,000.
21. Which of the following is an appropriate base to
distribute the cost of building depreciation to 24. If advertising expense were allocated to centers
responsibility centers? based on actual center profitability, how much
A. Number of employees in the responsibility advertising would be allocated to Irvine?
centers. A. $19,800.
B. Budgeted sales dollars of the responsibility B. $21,000.
centers. C. $30,000.
C. Square feet occupied by the responsibility D. $40,543.
centers. E. Some other amount.
D. Budgeted net income of the responsibility
centers. Answer: D LO: 4 Type: A
E. Total budgeted direct operating costs of the
responsibility centers. 25. Assume that management used the allocation base
that is most influenced by advertising effort and
Answer: C LO: 4 Type: N consistent with sound managerial accounting
practices. How much advertising would be allocated
22. David Corporation is in the process of selecting allocation to Irvine?
bases so that selected costs can be charged to responsibility A. $17,838.
centers. Would the number of employees likely be a good B. $19,800.
base to use to allocate the costs of Human Resources, C. $20,000.
Building and Grounds, and Repairs and Maintenance to D. $20,400.
user centers? E. $21,000.
Human Buildings and Repairs and
Resources Grounds Maintenance Answer: E LO: 4 Type: A, N
A. Yes Yes Yes
B. Yes No Yes 26. Responsibility accounting systems strive to:
C. Yes No No A. place blame on guilty individuals.
D. No Yes Yes B. provide information to managers.
E. No Yes No C. hold managers accountable for both controllable
and noncontrollable costs.
Answer: C LO: 4 Type: N D. identify unfavorable variances.
E. provide information so that managers can make
23. Cost pools should be charged to responsibility centers decisions that are in the best interest of their
by using: individual centers rather than in the best interests
A. budgeted amounts of allocation bases because of the firm as a whole.
the cost allocation to one responsibility center
should influence the allocations to others. Answer: B LO: 4 Type: RC
B. budgeted amounts of allocation bases because
the cost allocation to one responsibility center 27. Controllable costs, as used in a responsibility
should not influence the allocations to others. accounting system, consist of:
C. actual amounts of allocation bases because the A. only fixed costs.
cost allocation to one responsibility center should B. only direct materials and direct labor.
influence the allocations to others. C. those costs that a manager can influence in the
D. actual amounts of allocation bases because the time period under review.
cost allocation to one responsibility center should D. those costs about which a manager has some
not influence the allocations to others. knowledge.
E. some other approach. E. those costs that are influenced by parties
external to the organization.
E. Items "C" and "D" above.
Answer: C LO: 4 Type: RC
Answer: D LO: 5 Type: N
28. For a company that uses responsibility accounting,
which of the following costs is least likely to appear on 32. The difference between the profit margin controllable
a performance report of an assembly-line supervisor? by a segment manager and the segment profit margin
A. Direct materials used. is caused by:
B. Departmental supplies. A. variable operating expenses.
C. Assembly-line labor. B. allocated common expenses.
D. Repairs and maintenance. C. fixed expenses controllable by the segment
E. Assembly-line facilities depreciation. manager.
D. fixed expenses traceable to the segment but
Answer: E LO: 4 Type: N controllable by others.
E. other revenue.
29. Common costs:
A. are not easily related to a segment's activities. Answer: D LO: 5 Type: RC
B. are easily related to a segment's activities.
C. are charged to the operating segments of a 33. The profit margin controllable by the segment
company. manager would not include:
D. are not charged to the operating segments of a A. variable operating expenses.
company. B. fixed expenses controllable by the segment
E. are best described by characteristics "A" and "D" manager.
above. C. a share of the company's common fixed
expenses.
Answer: E LO: 5 Type: RC D. income tax expense.
E. items "C" and "D" above.
30. Harris Company is preparing a segmented income
statement, subdivided into departments (billing, Answer: E LO: 5 Type: RC
purchasing, and telemarketing). Which of the
following choices correctly describes the accounting 34. A segment contribution margin would reflect the
treatment of the firm's compensation cost for key impact of:
executives (president and vice-presidents)? A. variable operating expenses.
A. The cost is charged to the departments. B. fixed expenses controllable by the segment
B. The cost is not charged to the departments manager.
because, although easily traceable to the C. fixed expenses traceable to the segment but
departments, it is not controllable at the controllable by others.
departmental level. D. common fixed expenses.
C. The cost is not charged to the departments E. items "A," "B," and "C" above.
because, although controllable at the
departmental level, it is not easily traceable to Answer: A LO: 5 Type: RC
the departments.
D. The cost is not charged to the departments 35. Gathersburg Retail has three stores in Maryland.
because it is both easily traceable to the Which of the following costs would likely be excluded
departments and controllable by the when computing the profit margin controllable by
departments. store no. 3's manager?
E. The cost is not charged to the departments A. Hourly labor costs incurred by personnel at store
because it is neither easily traceable to the no. 3.
departments nor controllable by the departments. B. Property taxes attributable to store no. 3.
C. The salary of Gathersburg's president.
Answer: E LO: 5 Type: N D. The salary of store no. 3's manager.
E. Items "B," "C," and "D" above.
31. West Coast Electronics (WCE) operates 87 stores
and has three divisions: California, Oregon, and Answer: E LO: 5 Type: N
Washington. Which of the following costs would not
appear on Oregon's portion of WCE's segmented 36. Which of the following measures would reflect the variable
income statement? costs incurred by a business segment?
A. Costs related to statewide advertising Segment Profit Margin Segment
contracts, negotiated by Oregon's divisional Contribution Controllable Profit
manager. Margin by Segment Margin
B. Variable sales commissions paid to Oregon's Manager
salespeople. A. Yes No No
C. Compensation paid to Oregon's chief B. Yes No Yes
operating officer, as determined by WCE's C. Yes Yes No
management. D. Yes Yes Yes
D. Oregon's allocated share of general WCE E. No Yes Yes
corporate overhead.
Answer: D LO: 5 Type: RC Variable operating expenses
Fixed expenses:
37. Which of the following measures would reflect the fixed Traceable to A and controllable by A
costs controllable by a segment manager? Traceable to A and controllable by others
Segment Profit Margin Segment
Contribution Controllable Profit A's segment profit margin is:
Margin by Segment Margin A. $105,000.
Manager B. $225,000.
A. Yes No No C. $380,000.
B. Yes No Yes D. $500,000.
C. Yes Yes No E. $505,000.
D. Yes Yes Yes
E. No Yes Yes Answer: A LO: 5 Type: A

Answer: E LO: 5 Type: RC 41. The following data relate to Department no. 3 of Tsay
Corporation:
38. Which of the following would be the best measure on
which to base a segment manager's performance Segment contribution margin $540,000
evaluation for purposes of granting a bonus? Profit margin controllable by the 310,000
A. Segment sales revenue. segment manager
B. Segment contribution margin. Segment profit margin 150,000
C. Profit margin controllable by the segment
manager. On the basis of this information, Department no. 3's
D. Segment profit margin. variable operating expenses are:
E. Segment net income. A. $80,000.
B. $160,000.
Answer: C LO: 5 Type: N C. $230,000.
D. $390,000.
39. Sands Corporation operates two stores: J and K. The E. not determinable.
following information relates to store J:
Answer: E LO: 5 Type: A
Sales revenue
Variable operating expenses 42. The following data relate to Department no. 2 of
Fixed expenses: Young Corporation:
Traceable to J and controllable by J
Traceable to J and controllable by others Segment contribution margin $480,000
Profit margin controllable by the 230,000
J's segment contribution margin is: segment manager
A. $345,000. Segment profit margin 110,000
B. $425,000.
C. $620,000. On the basis of this information, fixed costs traceable
D. $700,000. to Department no. 2 but controllable by others are:
E. $745,000. A. $120,000.
B. $140,000.
Answer: D LO: 5 Type: A C. $250,000.
D. $370,000.
40. Thompson Corporation operates two stores: A and B. E. not determinable.
The following information relates to store A:
Answer: A LO: 5 Type: A
Sales revenue
Use the following to answer questions 43-47:

The following information was taken from the segmented income statement of Restin, Inc., and the company's three divisions:
Los Bay Central
Restin, Angeles Area Valley
Inc. Division Division Division
Revenues $750,000 $200,000 $235,000 $325,000
Variable operating expenses 410,000 110,000 120,000 180,000
Controllable fixed expenses 210,000 65,000 75,000 70,000
Noncontrollable fixed expenses 60,000 15,000 20,000 25,000

In addition, the company incurred common fixed costs of B. $18,000.


$18,000. C. $20,000.
D. $40,000.
43. Bay Area's segment profit margin is: E. $115,000.
A. $14,000.
Answer: C LO: 5 Type: A D. Prevention cost.
E. Appraisal cost.
44. The profit margin controllable by the Central Valley
segment manager is: Answer: C LO: 6 Type: RC
A. $32,000.
B. $44,000. 50. Which of the following costs is often considered the
C. $50,000. hardest to measure?
D. $75,000. A. Prevention costs.
E. $145,000. B. Appraisal costs.
C. Internal failure costs.
Answer: D LO: 5 Type: A D. External failure costs.
E. The cost of lost sales.
45. Assuming use of a responsibility accounting system,
which of the following amounts should be used to Answer: E LO: 6 Type: RC
evaluate the performance of the Los Angeles division
manager? 51. Which of the following costs would be classified as a
A. $4,000. prevention cost on a quality report?
B. $8,000. A. Reliability engineering.
C. $10,000. B. Materials inspection.
D. $25,000. C. Rework.
E. $90,000. D. Warranty repairs.
E. Out-of-court liability settlements.
Answer: D LO: 5 Type: A, N
Answer: A LO: 6 Type: RC
46. Which of the following amounts should be used to
evaluate whether Restin, Inc., should continue to 52. Which of the following costs would be classified as an
invest company resources in the Los Angeles appraisal cost on a quality report?
division? A. Reliability engineering.
A. $4,000. B. Materials inspection.
B. $8,000. C. Rework.
C. $10,000. D. Warranty repairs.
D. $25,000. E. Out-of-court liability settlements.
E. $90,000.
Answer: B LO: 6 Type: RC
Answer: C LO: 5 Type: A, N
53. If goods are inspected and found to be defective, any
47. Assume that the Los Angeles division increases its rework costs related to these units before the units
promotion expense, a controllable fixed cost, by are transferred to the finished-goods warehouse
$10,000. As a result, revenues increase by $50,000. would be classified as a(n):
If variable expenses are tied directly to revenues, the A. external failure cost.
new Los Angeles segment profit margin is: B. internal failure cost.
A. $12,500. C. production inefficiency cost.
B. $22,500. D. prevention cost.
C. $32,500. E. appraisal cost.
D. $50,000.
E. $60,000. Answer: B LO: 6 Type: RC

Answer: B LO: 5 Type: A 54. Which of the following costs would be classified as an
internal failure cost on a quality report?
48. Quality of conformance refers to: A. Reliability engineering.
A. the extent to which a product meets the B. Materials inspection.
specifications of its design. C. Rework.
B. the extent to which a product adds value to a D. Warranty repairs.
firm's product line. E. Out-of-court liability settlements.
C. the extent to which a product is designed for its
intended use. Answer: C LO: 6 Type: RC
D. the extent to which a product maximizes non-
value-added activities in the production process. 55. The cost of servicing a unit under a warranty
E. a cost control that is achievable. agreement is known as a(n):
A. external failure cost.
Answer: A LO: 6 Type: RC B. internal failure cost.
C. production inefficiency cost.
49. Which of the following is not a cost of quality? D. prevention cost.
A. External failure cost. E. appraisal cost.
B. Internal failure cost.
C. Production inefficiency cost. Answer: A LO: 6 Type: RC
D. Warranty repairs.
56. Which of the following costs would be classified as an E. Pilot studies/focus-group sessions.
external failure cost on a quality report?
A. Reliability engineering. Answer: D LO: 6 Type: RC
B. Materials inspection.
C. Rework.
57. Which of the following choices correctly depicts a prevention cost and an external failure cost?
Prevention Cost External Failure Cost
A. Inspection of work in process Warranty costs
B. Quality training Product liability lawsuits
C. In-house rework of defective units Transportation costs to customer sites
D. Customer complaints Reliability engineering
E. Choices "A" and "B" above.

Answer: B LO: 6 Type: RC


machine breakdowns
58. Elizabeth, Inc., was having significant quality
problems in its manufacturing plant. To remedy The sum of Lexington’s prevention and external failure costs
the situation, management implemented various up- is:
front procedures and programs that were expected A. $40,000.
to reduce the production of bad units to acceptable B. $49,000.
(normal) levels and benefit the firm financially. If C. $59,000.
the procedures and programs functioned as D. $63,100.
intended, what is likely true about the amounts the E. some other amount.
company incurred for prevention cost, internal
failure cost, and external failure cost? Answer: D LO: 6 Type: A
Internal External
Prevention Failure Failure 61. Under the contemporary view of product quality, companies
Cost Cost Cost should strive to:
A. Increase Increase Increase A. balance failure costs with the sum of prevention and
B. Increase Increase Decrease appraisal costs.
C. Increase Decrease Increase B. increase total quality costs.
D. Increase Decrease Decrease C. achieve zero defects in manufacturing.
E. Decrease Decrease Decrease D. inspect after-the-fact rather than install a series of
preventative manufacturing controls.
Answer: D LO: 6 Type: N E. operate at the top of the total quality cost curve.

59. The costs that follow appeared on Omaha's Answer: C LO: 7 Type: RC
quality cost report:
62. Which of the following is a helpful tool in identifying the
Warranty costs $15,000 frequency of quality-control problems?
Raw-materials 10,000 A. Decision trees.
inspection B. Scatter diagrams.
Quality training 31,000 C. Pareto diagrams.
Customer complaints 5,500 D. Flowcharts.
Rework of defective 12,800 E. Decision tables.
units
Answer: C LO: 7 Type: RC
The sum of Omaha's appraisal and internal
failure costs is: 63. Many companies (especially those in Europe) now require
A. $10,000. their suppliers to meet specified quality guidelines issued by
B. $12,800. the:
C. $22,800. A. International Standards Organization (ISO).
D. $68,800. B. Quality Assurance Institute (QAI).
E. some other amount. C. Taguchi Standards Association (TSA).
D. Pareto Standards Institute (PSI).
Answer: C LO: 6 Type: A E. an organization other than those mentioned above.

60. The costs that follow appeared on Lexington’s Answer: A LO: 7 Type: RC
quality cost report:
64. All of the following concepts are related to environmental
Warranty costs $19,000 management (cost and otherwise) except:
Raw-materials inspection 9,000 A. dynamic programming efforts.
Quality training 40,000 B. sustainable development.
Customer complaints 4,100 C. monitoring costs.
Production stoppages from 7,800 D. abatement costs.
E. remediation costs. 2. What practice is present when divisional managers
throughout an organization work together in an effort to
Answer: A LO: 8 Type: RC achieve the organization's goals?
A. Participatory management.
65. Costs incurred to reduce or eliminate pollution B. Goal attainment.
are commonly known as: C. Goal congruence.
A. monitoring costs. D. Centralization of objectives.
B. abatement costs. E. Negotiation by subordinates.
C. on-site remediation costs.
D. off-site remediation costs. Answer: C LO: 1 Type: RC
E. hidden costs.
3. Consider the following statements about goal congruence:
Answer: B LO: 8 Type: RC
I. Goal congruence is obtained when managers of
66. Clean-up costs are commonly classified as: subunits throughout an organization strive to achieve
A. monitoring costs. the goals set by top management.
B. abatement costs. II. Managers are often more concerned about the
C. remediation costs. performance of their own subunits rather than the
D. internal failure costs. performance of the entire organization.
E. external failure costs. III. Achieving goal congruence in most organizations is
relatively straightforward and easy to accomplish.
Answer: C LO: 8 Type: N
Which of the above statements is (are) true?
A. I only.
67. Which of the following fail to be captured and B. II only.
reported by a company's accounting system C. I and II.
as an environmental cost? D. II and III.
A. Monitoring costs. E. I, II, and III.
B. Abatement costs.
C. Hidden costs. Answer: C LO: 1 Type: RC
D. On-site remediation costs.
E. Off-site remediation costs. 4. Which of the following performance measures is (are) used
to evaluate the financial success or failure of investment
Answer: C LO: 8 Type: RC centers?
A. Residual income.
68. A company that strives to maximize the value B. Return on investment.
of its pollution-related activities would follow C. Number of suppliers.
a(n): D. Economic value added.
A. process improvement strategy. E. All of the above measures are used except "C."
B. prevention strategy.
C. end-of-pipe strategy. Answer: E LO: 1 Type: RC
D. visible cost strategy.
E. matrix strategy. 5. ROI is most appropriately used to evaluate the performance
of:
Answer: B LO: 8 Type: RC A. cost center managers.
B. revenue center managers.
C. profit center managers.
D. investment center managers.
MULTIPLE CHOICE QUESTIONS E. both profit center managers and investment center
managers.
1. The biggest challenge in making a
decentralized organization function effectively Answer: D LO: 2 Type: RC
is:
A. earning maximum profits through fair 6. Which of the following is not considered in the calculation of
practices. divisional ROI?
B. minimizing losses. A. Divisional income.
C. taking advantage of the specialized B. Earnings velocity.
knowledge and skills of highly talented C. Capital turnover.
managers. D. Sales margin.
D. obtaining goal congruence among E. Sales revenue.
division managers.
E. developing an adequate budgetary Answer: B LO: 2 Type: RC
control system.
7. Which of the following is the correct mathematical expression
Answer: D LO: 1 Type: RC for return on investment?
A. Sales margin ÷ capital turnover.
B. Sales margin + capital turnover. A. 13.33%.
C. Sales margin - capital turnover. B. 83.33%.
D. Sales margin x capital turnover. C. 120.00%.
E. Capital turnover ÷ sales margin. D. 750.00%.
E. some other figure.
Answer: D LO: 2 Type: RC
Answer: A LO: 2 Type: A
8. The ROI calculation will indicate:
A. the percentage of each sales dollar that is 13. Zang Enterprises had a sales margin of 7%, sales of
invested in assets. $5,000,000, and invested capital of $4,000,000. The
B. the sales dollars generated from each company's ROI was:
dollar of income. A. 5.60%.
C. how effectively a company used its B. 8.75%.
invested capital. C. 11.43%.
D. the invested capital generated from each D. 17.86%.
dollar of income. E. some other figure.
E. the overall quality of a company's
earnings. Answer: B LO: 2 Type: A

Answer: C LO: 2 Type: RC 14. Mission, Inc., reported a return on investment of 12%, a
capital turnover of 5, and income of $180,000. On the basis
9. A company's sales margin: of this information, the company's invested capital was:
A. must, by definition, be greater than the A. $300,000.
firm's net sales. B. $900,000.
B. has basically the same meaning as the C. $1,500,000.
term "contribution margin." D. $7,500,000.
C. is computed by dividing sales revenue E. some other amount.
into income.
D. is computed by dividing income into sales Answer: C LO: 2 Type: A
revenue.
E. shows the sales dollars generated from 15. The information that follows relates to Katz Corporation:
each dollar of income.
Sales margin: 7.5%
Answer: C LO: 2 Type: RC Capital turnover: 2
Invested capital: $20,000,000
10. Which of the following is the correct
mathematical expression to derive a On the basis of this information, the company's sales
company's capital turnover? revenue is:
A. Sales revenue ÷ invested capital. A. $1,500,000.
B. Contribution margin ÷ invested capital. B. $3,000,000.
C. Income ÷ invested capital. C. $10,000,000.
D. Invested capital ÷ sales revenue D. $40,000,000.
E. Invested capital ÷ income E. some other amount.

Answer: A LO: 2 Type: RC Answer: D LO: 2 Type: A

11. Capital turnover shows: 16. A division's return on investment may be improved by
A. the amount of income generated by each increasing:
dollar of capital investment. A. cost of goods sold and expenses.
B. the number of sales dollars generated by B. sales margin and cost of capital.
each dollar of capital investment. C. sales revenue and cost of capital.
C. the amount of contribution margin D. capital turnover or sales margin.
generated by each dollar of capital E. capital turnover or cost of capital.
investment.
D. the amount of capital investment Answer: D LO: 3 Type: RC
generated by each sales dollar.
E. the amount of capital investment 17. All of the following actions will increase ROI except:
generated by each dollar of income. A. an increase in sales revenues.
B. a decrease in operating expenses.
Answer: B LO: 2 Type: RC C. a decrease in a company's invested capital.
D. a decrease in the number of units sold.
12. Webster Company had sales revenue and E. an improvement in manufacturing efficiency.
operating expenses of $5,000,000 and
$4,200,000, respectively, for the year just Answer: D LO: 3 Type: N
ended. If invested capital amounted to
$6,000,000, the firm's ROI was: 18. Which of the following is used in the calculation of both return
on investment and residual income?
A. Total stockholders' equity. 22. The Fitzhugh Division of General Enterprises has a negative
B. Retained earnings. residual income of $540,000. Fitzhugh's management is
C. Invested capital. contemplating an investment opportunity that will reduce this
D. Total liabilities. negative amount to $400,000. The investment:
E. The cost of capital. A. should be pursued because it is attractive from both the
divisional and corporate perspectives.
Answer: C LO: 2 Type: RC B. should be pursued because it is attractive from the
divisional perspective although not from the corporate
19. Consider the following statements about perspective.
residual income: C. should be pursued because it is attractive from the
corporate perspective although not from the divisional
I. Residual income incorporates a firm's perspective.
cost of acquiring investment capital. D. should not be pursued because it is unattractive from
II. Residual income is a percentage both the divisional and corporate perspectives.
measure, not a dollar measure. E. should not be pursued because it is unattractive from
III. If used correctly, residual income may the divisional perspective although it is attractive from
result in division managers making the corporate perspective.
decisions that are in their own best
interest and not in the best interest of Answer: A LO: 4 Type: N
the entire firm.
23. The Magellan Division of Global Corporation, which has
Which of the above statements is (are) true? income of $250,000 and an asset investment of $1,562,500,
A. I only. is studying an investment opportunity that will cost $450,000
B. II only. and yield a profit of $67,500. Assuming that Global uses an
C. I and II. imputed interest charge of 14%, would the investment be
D. II and III. attractive to:
E. I and III.
1—Divisional management if ROI is used to evaluate
Answer: A LO: 2, 4 Type: RC divisional performance?
2—Divisional management if residual income (RI) is
20. The basic idea behind residual income is to used to evaluate divisional performance?
have a division maximize its: 3—The management of Global Corporation?
A. earnings per share.
B. income in excess of a corporate imputed Attractive to Attractive to Attractive to
interest charge. Magellan: ROI Magellan: RI Global
C. cost of capital. A. Yes Yes Yes
D. cash flows. B. Yes No No
E. invested capital. C. Yes No Yes
D. No Yes Yes
Answer: B LO: 2, 4 Type: N E. No Yes No

21. Sunrise Corporation has a return on Answer: D LO: 4 Type: A, N


investment of 15%. A Sunrise division, which
currently has a 13% ROI and $750,000 of 24. The Georgia Division of Carter Companies currently reports a
residual income, is contemplating a massive profit of $3.4 million. Divisional invested capital totals $12.5
new investment that will (1) reduce divisional million; the imputed interest rate is 14%. On the basis of this
ROI and (2) produce $120,000 of residual information, Georgia's residual income is:
income. If Sunrise strives for goal A. $476,000.
congruence, the investment: B. $1,274,000.
A. should not be acquired because it C. $1,650,000.
reduces divisional ROI. D. $1,750,000.
B. should not be acquired because it E. some other amount.
produces $120,000 of residual income.
C. should not be acquired because the Answer: C LO: 2 Type: A
division's ROI is less than the corporate
ROI before the investment is considered. 25. The following information relates to the Mountain Division of
D. should be acquired because it produces Adler Enterprises:
$120,000 of residual income for the
division. Income for the period just ended: $1,500,000
E. should be acquired because after the Invested capital: $12,000,000
acquisition, the division's ROI and
residual income are both positive If the firm has an imputed interest rate of 11%, Mountain's
numbers. residual income would be:
A. $165,000.
Answer: D LO: 4 Type: N B. $180,000.
C. $187,500. E. 40%.
D. some other dollar amount.
E. a percentage greater than 11%. Answer: A LO: 2 Type: A

Answer: B LO: 2 Type: A 30. The ROI is:


A. 6%.
26. Extron Division reported a residual income of B. 15%.
$200,000 for the year just ended. The division C. 20%.
had $8,000,000 of invested capital and D. 30%.
$1,000,000 of income. On the basis of this E. 40%.
information, the imputed interest rate was:
A. 2.5%. Answer: C LO: 2 Type: A
B. 10.0%.
C. 12.5%. 31. The residual income is:
D. 20.0%. A. $30,000.
E. some other figure. B. $36,000.
C. $42,000.
Answer: B LO: 2 Type: A D. $54,000.
E. $82,800.
27. Barber Corporation uses an imputed interest
rate of 13% in the calculation of residual Answer: D LO: 2 Type: A
income. Division X, which is part of Barber,
had invested capital of $1,200,000 and an ROI 32. For the period just ended, United Corporation's Delta Division
of 16%. On the basis of this information, X's reported profit of $31.9 million and invested capital of $220
residual income was: million. Assuming an imputed interest rate of 12%, which of
A. $24,960. the following choices correctly denotes Delta's return on
B. $36,000. investment (ROI) and residual income?
C. $156,000. Return on Residual
D. $192,000. Investment Income
E. some other amount. A. 12.0% $(5.5) million
B. 12.0% $5.5 million
Answer: B LO: 2 Type: A, N C. 14.5% $(5.5) million
D. 14.5% $5.5 million
E. 14.5% $26.4 million

Answer: D LO: 2 Type: A

33. For the period just ended, Price Corporation's Ohio Division
reported profit of $49 million and invested capital of $350
million. Assuming an imputed interest rate of 16%, which of
the following choices correctly denotes Ohio's return on
Use the following to answer questions 28-31: investment (ROI) and residual income?
Return on Residual
The following information pertains to Bingo Concrete: Investment Income
A. 14% $7 million
Sales revenue $1,500,000 B. 14% $(7) million
Gross margin 600,000 C. 16% $7 million
Income 90,000 D. $7 million 14%
Invested capital 450,000 E. None of the above choices shows both the correct
ROI and residual income.
The company's imputed interest rate is 8%.
Answer: B LO: 2 Type: A
28. The capital turnover is:
A. 3.33. 34. Which of the following elements is not used when calculating
B. 5.00. the weighted-average cost of capital?
C. 16.67. A. Before-tax cost of debt capital.
D. 20.00. B. After-tax cost of debt capital.
E. 30.00. C. Cost of equity capital.
D. Market value of debt capital.
Answer: A LO: 2 Type: A E. Market value of equity capital.

29. The sales margin is: Answer: A LO: 2 Type: RC


A. 6%.
B. 15%. 35. The following information relates to the Atlantic Division of
C. 20%. Ocean Enterprises:
D. 30%.
Interest rate on debt capital: 8% weighted-average cost of capital. Assets total $7,000,000
Cost of equity capital: 12% and current liabilities total $1,800,000. On the basis of this
Market value of debt capital: $50 information, Carolina's economic value added is:
million A. $2,408,000.
Market value of equity capital: $80 B. $2,732,000.
million C. $3,668,000.
Income tax rate: 30% D. $3,992,000.
E. some other amount.
On the basis of this information, Atlantic's
weighted-average cost of capital is: Answer: B LO: 2 Type: A
A. 7.3%.
B. 8.3%. 40. The following information relates to Houston, Inc.:
C. 9.5%.
D. 10.8%. Total assets $9,000,000
E. some other figure. After-tax operating income 1,500,000
Current liabilities 800,000
Answer: C LO: 2 Type: A
If the company has a 10% weighted-average cost of capital,
36. The market value of Glendale’s debt and its economic value added would be:
equity capital totals $180 million, 80% of which A. $(200,000).
is equity related. An analysis conducted by B. $530,000.
the company’s finance department revealed a C. $680,000.
7% after-tax cost of debt capital and a 10% D. $970,000.
cost of equity capital. On the basis of this E. some other amount.
information, Glendale’s weighted-average cost
of capital: Answer: C LO: 2 Type: A
A. is 7.6%.
B. is 8.5%. 41. Given that ROI measures performance over a period of time,
C. is 9.4%. invested capital would most appropriately be figured by
D. cannot be determined based on the data using:
presented because the cost of debt capital A. beginning-of-year assets.
must be stated on a before-tax basis. B. average assets.
E. cannot be determined based on the data C. end-of-year assets.
presented because the cost of equity D. total assets.
capital must be stated on an after-tax E. only current assets.
basis.
Answer: B LO: 5 Type: RC
Answer: C LO: 2 Type: A, N
42. When an organization allows divisional managers to be
37. Which of the following measures of responsible for short-term loans and credit, the division's
performance is, in part, based on the invested capital should be measured by
weighted-average cost of capital? A. total assets minus total liabilities.
A. Return on investment. B. average total assets minus average current liabilities.
B. Capital turnover. C. average total assets minus average total liabilities.
C. Book value. D. average total liabilities minus average current assets.
D. Economic value added (EVA). E. average total liabilities minus total assets.
E. Gross margin.
Answer: B LO: 5 Type: RC
Answer: D LO: 2 Type: RC
43. Hayes Division has been stagnant over the past five years,
38. Which of the following elements is not used in neither growing nor contracting in size and profitability.
the calculation of economic value added for an Investments in new property, plant, and equipment have
investment center? been minimal. Would the division's use of total assets
A. An investment center's after-tax operating (valued at net book value) when measuring ROI result in (1)
income. using numbers that are consistent with those on the balance
B. An investment center's total assets. sheet and (2) a rising ROI over time?
C. An investment center's return on Consistent with Produce a Rising
investment. Numbers Return on
D. An investment center's current liabilities. on the Balance Investment Over
E. A company's weighted-average cost of Sheet? Time?
capital. A. Yes Yes
B. Yes No
Answer: C LO: 2 Type: RC C. No Yes
D. No No
39. Carolina Corporation has an after-tax E. Yes Need more
operating income of $3,200,000 and a 9% information
to judge basis of this information, which of the following statements is
most correct?
Answer: A LO: 5 Type: RC A. The profit reported by New York will increase and
the profit reported by Arizona will decrease.
44. The income calculation for a division B. The profit reported by New York will increase, the
manager's ROI should be based on: profit reported by Arizona will decrease, and Thurmond’s
A. divisional contribution margin. profit will be unaffected.
B. profit margin controllable by the division C. The profit reported by New York will decrease, the
manager. profit reported by Arizona will increase, and Thurmond’s
C. profit margin traceable to the division. profit will be unaffected.
D. divisional income before interest and D. The profit reported by New York will increase and
taxes. the profit reported by Arizona will increase.
E. divisional net income. E. The profit reported by New York and the profit
reported by Arizona will be unaffected.
Answer: B LO: 5 Type: RC
Answer: B LO: 6 Type: RC, N
45. To partially eliminate the problems that are
associated with the short-term focus of return 49. Which of the following describes the goal that should be
on investment, residual income, and EVA, the pursued when setting transfer prices?
performance of a division's major investments A. Maximize profits of the buying division.
is commonly evaluated through: B. Maximize profits of the selling division.
A. postaudits. C. Allow top management to become actively involved
B. sensitivity analysis. when calculating the proper dollar amounts.
C. performance operating plans. D. Establish incentives for autonomous division managers
D. horizontal analysis. to make decisions that are in the overall organization's
E. segmented reporting. best interests (i.e., goal congruence).
E. Minimize opportunity costs.
Answer: A LO: 5 Type: RC
Answer: D LO: 6 Type: RC
46. The amounts charged for goods and services
exchanged between two divisions are known 50. A general calculation method for transfer prices that achieves
as: goal congruence begins with the additional outlay cost per
A. opportunity costs. unit incurred because goods are transformed and then
B. transfer prices. A. adds the opportunity cost per unit to the organization
C. standard variable costs. because of the transfer.
D. residual prices. B. subtracts the opportunity cost per unit to the
E. target prices. organization because of the transfer.
C. adds the sunk cost per unit to the organization because
Answer: B LO: 6 Type: RC of the transfer.
D. subtracts the sunk cost per unit to the organization
47. Nevada, Inc., has two divisions, one located in because of the transfer.
Las Vegas and the other located in Reno. Las E. adds the sales revenue per unit to the organization
Vegas sells selected goods to Reno for use in because of the transfer.
various end-products. Assuming that the
transfer prices set by Las Vegas do not Answer: A LO: 6 Type: RC
influence the decisions made by the two
divisions, which of the following correctly 51. Suddath Corporation has no excess capacity. If the firm
describes the impact of the transfer prices on desires to implement the general transfer-pricing rule,
divisional profits and overall company profit? opportunity cost would be equal to:
Las Vegas Profit Reno Profit A. zero.
A. Affected Affected B. the direct expenses incurred in producing the goods.
B. Affected Affected C. the total difference in the cost of production between two
C. Affected Not affected divisions.
D. Not affected Not affected D. the contribution margin forgone from the lost external
E. Not affected Not affected sale.
E. the summation of variable cost plus fixed cost.
Answer: B LO: 6 Type: RC
Answer: D LO: 6 Type: RC
48. Thurmond, Inc., has two divisions, one located
in New York and the other located in Arizona. 52. Tulsa Corporation has excess capacity. If the firm desires to
New York sells a specialized circuit to Arizona implement the general transfer-pricing rule, opportunity cost
and just recently raised the circuit’s transfer would be equal to:
price. This price hike had no effect on the A. zero.
volume of circuits transferred nor on Arizona’s B. the direct expenses incurred in producing the goods.
option of acquiring the circuit from either New C. the total difference in the cost of production between two
York or from an external supplier. On the divisions.
D. the contribution margin forgone from the D. $2.90.
lost external sale. E. $3.00.
E. the summation of variable cost plus fixed
cost. Answer: A LO: 6 Type: A

Answer: A LO: 6 Type: RC 56. Assume the Bottle Division has no excess capacity and could
sell everything it produced externally. Using the general rule,
53. McKenna's Florida Division is currently the transfer price from the Bottle Division to the Cologne
purchasing a part from an outside supplier. Division would be:
The company's Alabama Division, which has A. $2.00.
excess capacity, makes and sells this part for B. $2.10.
external customers at a variable cost of $22 C. $2.60.
and a selling price of $34. If Alabama begins D. $2.90.
sales to Florida, it (1) will use the general E. $3.00.
transfer-pricing rule and (2) will be able to
reduce variable cost on internal transfers by Answer: D LO: 6 Type: A
$4. If sales to outsiders will not be affected,
Alabama would establish a transfer price of: 57. The maximum amount the Cologne Division would be willing
A. $18. to pay for each bottle transferred would be:
B. $22. A. $2.00.
C. $30. B. $2.10.
D. $34. C. $2.60.
E. some other amount. D. $2.90.
E. $3.00.
Answer: A LO: 6 Type: A
Answer: C LO: 6 Type: A
54. AutoTech's Northern Division is currently
purchasing a part from an outside supplier. 58. Transfer prices can be based on:
The company's Southern Division, which has A. variable cost.
no excess capacity, makes and sells this part B. full cost.
for external customers at a variable cost of $19 C. an external market price.
and a selling price of $31. If Southern begins D. a negotiated settlement between the buying and selling
sales to Northern, it (1) will use the general divisions.
transfer-pricing rule and (2) will be able to E. all of the above.
reduce variable cost on internal transfers by
$3. On the basis of this information, Southern Answer: E LO: 7 Type: RC
would establish a transfer price of:
A. $16. 59. Which of the following transfer-pricing methods can lead to
B. $19. dysfunctional decision-making behavior by managers?
C. $28. A. Variable cost.
D. $31. B. Full cost.
E. some other amount. C. External market price.
D. A professionally negotiated, amicable settlement
Answer: C LO: 6 Type: A between the buying and selling divisions.
E. None of the above.
Use the following to answer questions 55-57:
Answer: B LO: 7 Type: RC
Laissez Faire has two divisions: the Cologne Division
and the Bottle Division. The Bottle Division produces 60. The Pro Division of Custom Industries is in need of a
containers that can be used by the Cologne Division. particular service. The service can be obtained from another
The Bottle Division's variable manufacturing cost is $2, division of Custom at "cost," with cost defined as the
shipping cost is $0.10, and the external sales price is summation of variable cost ($9) and fixed cost ($3).
$3. No shipping costs are incurred on sales to the Alternatively, Pro can secure the service from a source
Cologne Division, and the Cologne Division can external to Custom for $10. Which of the following
purchase similar containers in the external market for statements is true?
$2.60. A. Pro should compare $10 vs. $3 in deciding where to
acquire the service.
55. The Bottle Division has sufficient capacity to B. Pro should compare $10 vs. $9 in deciding where to
meet all external market demands in addition acquire the service.
to meeting the demands of the Cologne C. Pro should compare $10 vs. $12 in deciding where to
Division. Using the general rule, the transfer acquire the service.
price from the Bottle Division to the Cologne D. From Custom's perspective, the proper decision is
Division would be: reached by comparing $10 vs. $9.
A. $2.00. E. Both "C" and "D" are true.
B. $2.10.
C. $2.60. Answer: E LO: 7 Type: A, N
what type of transfer price should be set for the
61. Division A transfers item no. 78 to Division B. subassembly?
Consider the following situations: Division A Division B Transfer
Income Income Price
1—A is located in Texas and B is A. Low Low Low
located in California. B. Low High Low
2—A is located in Texas and B is C. Low High High
located in Mexico. D. High Low High
E. High High Low
Assuming that item no. 78 is unavailable in the
open market, which of the following choices Answer: B LO: 7 Type: N
correctly depicts the probable importance of
federal income taxes when determining the 63. Consider the following statements about transfer pricing:
transfer price that is established for item no.
78? I. Income taxes and import duties are an important
Situation 1 Situation 2 consideration when setting a transfer price for
A. Important Important companies that pursue international commerce.
B. Important Not important II. Transfer prices cannot be used by organizations in
C. Not important Important the service industry.
D. Not important Not important III. Transfer prices are totally cost-based in nature, not
E. It is not possible to judge based on the information presented. market-based.

Answer: C LO: 7 Type: N Which of the above statements is (are) true?


A. I only.
62. Division A transfers a profitable subassembly B. II only.
to Division B, where it is assembled into a final C. I and II.
product. A is located in a European country D. II and III.
that has a high tax rate; B is located in an E. I, II, and III.
Asian country that has a low tax rate. Ideally,
(1) what type of before-tax income should Answer: A LO: 7 Type: RC
each division report from the transfer and (2)
EXERCISES

Components of Return on Investment

64. The following data pertain to Corkscrew Corporation:

Income $ 8,000,000
Sales revenue 40,000,000
Average invested capital 50,000,000

Required:
Calculate Corkscrew Corporation's sales margin, capital turnover, and return on investment.

LO: 2 Type: A

Answer:
Sales margin: $8,000,000 ÷ $40,000,000 = 20%
Capital turnover: $40,000,000 ÷ $50,000,000 = 0.8
Return on investment: $8,000,000 ÷ $50,000,000 = 16%

Components of ROI and Residual Income: Working Backward

65. Midland Division, which is part of Courtyard Enterprises, recently reported a sales margin of 30%, ROI of 21%, and residual
income of $220,000. Courtyard uses an imputed interest rate of 10%.

Required:
A. Briefly define sales margin, capital turnover, and return on investment.
B. Compute Midland's capital turnover and invested capital.
C. Ignoring your work in requirement "B," assume that invested capital amounted to $2,500,000. On the basis of this
information, calculate Midland's income and sales revenue.

LO: 2 Type: A, N

Answer:
A. Sales margin—the income generated from each sales dollar.
Computed as: Income ÷ sales revenue.

Capital turnover—the sales dollars produced from each dollar of invested capital. Computed as: Sales revenue ÷
invested capital.

Return on investment—the income generated from each dollar of invested capital. Computed as: Income ÷ invested
capital, or sales margin x capital turnover.

B. Capital turnover:
Capital turnover x sales margin = ROI
Capital turnover x 30% = 21%
Capital turnover = 0.7

Invested capital:
ROI = Income ÷ invested capital
21% = Income ÷ invested capital
Income = Invested capital x 21%

Residual income = Income - (invested capital x imputed interest rate)


$220,000 = Income - (invested capital x 10%)
$220,000 = (Invested capital x 21%) - (invested capital x 10%)
$220,000 = Invested capital x 11%
Invested capital = $2,000,000

C. Income:
ROI = Income ÷ invested capital
21% = Income ÷ $2,500,000
Income = $525,000

Sales revenue:
Sales margin = Income ÷ sales revenue
30% = $525,000 ÷ sales revenue
Sales revenue = $1,750,000

Economic Value Added, Weighted-Average Cost of Capital

66. The following data pertain to Dana Industries:

Interest rate on debt capital: 9%


Cost of equity capital: 12%
Before-tax operating income: $35 million
Market value of debt capital: $60 million
Market value of equity capital: $120 million
Total assets: $150 million
Income tax rate: 30%
Total current liabilities: $15 million

Required:
A. Compute Dana’s weighted-average cost of capital.
B. Compute Dana’s economic value added.
C. Briefly explain the meaning of economic value added.

LO: 2 Type: RC, A

Answer:
A. WACC = [(9% x 70%) x $60,000,000) + (12% x $120,000,000)] ÷ ($60,000,000 + $120,000,000)
WACC = ($3,780,000 + $14,400,000) ÷ $180,000,000
WACC = 10.1%

B. EVA = ($35,000,000 x 70%) - [($150,000,000 - $15,000,000) x 10.1%]


EVA = $24,500,000 - $13,635,000
EVA = $10,865,000

C. Economic value added (EVA) measures the amount of shareholder wealth being created from a company’s activities and
operations. To expand, debt and equity capital are used to fund activities—activities that are hopefully conducted in a
profitable manner. Profits cover the cost of the related capital, with shareholders benefiting from the residual (i.e., EVA).
Improving Return on Investment

67. The following data pertain to Norris Company for 20x1:

Sales revenue $1,000,000


Cost of goods sold 550,000
Operating expenses 400,000
Average invested capital 500,000

Required:
A. Calculate the company's sales margin, capital turnover, and return on investment for 20x1.
B. If the sales and average invested capital remain the same, to what level would total costs and expenses have to be
reduced in 20x2 to achieve a 15% return on investment?
C. Assume that costs and expenses are reduced, as calculated in requirement "B." Calculate the firm's new sales margin.
D. Suggest two possible actions that will improve the company's capital turnover.

LO: 2, 3 Type: A, N

Answer:
A. Sales revenue $1,000,000
Less: Cost of goods sold $550,000
Operating expenses 400,000 950,000
Operating income $ 50,000

Sales margin: $50,000 ÷ $1,000,000 = 5%


Capital turnover: $1,000,000 ÷ $500,000 = 2
Return on investment: $50,000 ÷ $500,000 = 10%

B. New income level: $500,000 x 15% = $75,000

Sales revenue $1,000,000


Less: Income 75,000
Costs and expenses $ 925,000

Therefore, total costs and expenses must be reduced from $950,000 ($550,000 + $400,000) to
$925,000 in order to achieve a 15% ROI.

C. Sales margin: $75,000 ÷ $1,000,000 = 7.5%

D. Capital turnover can be improved by increasing sales revenue and reducing invested capital.

Return on Investment and Residual Income: Calculation and Analysis

68. The following data pertain to the Oxnard Division of Kemp Company:

Divisional contribution margin $ 700,000


Profit margin controllable by the divisional manager 320,000
Profit margin traceable to the division 294,400
Average asset investment 1,280,000

The company uses responsibility accounting concepts when evaluating performance, and Oxnard's division manager is
contemplating the following three investments. He can invest up to $400,000.

No. 1 No. 2 No. 3


Cost $250,000 $300,000 $400,000
Expected income 50,000 54,000 96,000

Required:
A. Calculate the ROIs of the three investments.
B. What is the division manager's current ROI, computed by using responsibility accounting concepts?
C. Which of the three investments would be selected if the manager's focus is on Oxnard's divisional performance? Why?
D. If Kemp has an imputed interest charge of 22%, compute the residual income of investment no. 3. Is this investment
attractive from Oxnard's perspective? From Kemp's perspective? Why?

LO: 2, 4 Type: A, N

Answer:
A. No. 1: $50,000 ÷ $250,000 = 20%
No. 2: $54,000 ÷ $300,000 = 18%
No. 3: $96,000 ÷ $400,000 = 24%

B. Controllable profit margin ($320,000) ÷ asset investment ($1,280,000) = 25%

C. None, as all will lower the current ROI.

D. Residual income: $96,000 - ($400,000 x 22%) = $8,000

This investment is attractive from both Oxnard and Kemp's perspectives. The positive residual
income indicates that the investment income covers the imputed interest charge.

ROI and Residual Income, Investment Evaluation

69. Jasper Corporation is organized in three separate divisions. The three divisional managers are evaluated at year-end, and
bonuses are awarded based on ROI. Last year, the overall company produced a 12% return on its investment.

Managers of Jasper's Iowa Division recently studied an investment opportunity that would assist in the division's future growth.
Relevant data follow.

Iowa Investment
Division Opportunity
Income $12,800,000 $ 4,200,000
Invested capital 80,000,000 30,000,000

Required:
A. Compute the current ROI of the Iowa Division and the division's ROI if the investment opportunity is pursued.
B. What is the likely reaction of divisional management toward the acquisition? Why?
C. What is the likely reaction of Jasper's corporate management toward the investment? Why?
D. Assume that Jasper uses residual income to evaluate performance and desires an 11% minimum return on invested
capital. Compute the current residual income of the Iowa Division and the division's residual income if the investment is
made. Will divisional management likely change its attitude toward the acquisition? Why?

LO: 2, 4 Type: A, N

Answer:
A. ROI = Income ÷ invested capital
Current: $12,800,000 ÷ $80,000,000 = 16%
If investment is made: ($12,800,000 + $4,200,000) ÷ ($80,000,000 + $30,000,000) = 15.45%

B. Divisional management will likely be against the acquisition because ROI will be lowered from 16% to
15.45%. Since bonuses are awarded on the basis of ROI, the acquisition will result in less
compensation. However, before a final decision is made, additional insights are needed concerning
how the investment will assist in future growth and in what magnitude.

C. An examination of the investment reveals a 14% ROI ($4,200,000 ÷ $30,000,000). Corporate


management would probably favor the acquisition. Jasper has been earning a 12% return, and the
investment will help the organization as a whole.

D. Current residual income of Iowa Division:


Divisional income $12,800,000
Less: Imputed interest charge ($80,000,000 x 11%) 8,800,000
Residual income $ 4,000,000

Residual income if investment is made:


Divisional income ($12,800,000 + $4,200,000) $17,000,000
Less: Imputed interest charge [($80,000,000 + $30,000,000) x 11%] 12,100,000
Residual income $ 4,900,000
Residual income will increase by $900,000 ($4,900,000 - $4,000,000) from the acquisition. The RI
measure focuses on the corporate perspective, not the divisional perspective, by integrating the firm's
required return on invested capital.

Using ROI and Residual Income in Operating Decisions

70. Deborah Lewis, general manager of the Northwest Division of Berkshire Enterprises, has significant authority over pricing
decisions as well as programs that involve cost reduction/control. The data that follow relate to upcoming divisional
operations:

Average invested capital: $15,000,000


Annual fixed costs: $3,900,000
Variable cost per unit: $80
Number of units expected to be sold: 120,000

Required:
A. Top management will promote Deborah if she can earn a 14% return on investment for the year. What unit selling price
should she establish to get her promotion?
B. Independent of part "A," assume the unit selling price is $132 and that Berkshire has a 16% imputed interest charge. Top
management will promote Deborah to corporate headquarters if her division can generate $200,000 of residual income. If
Deborah desires to move to corporate, what must the division do to the amount of annual fixed costs incurred? Show
your calculations.

LO: 2, 4 Type: A, N

Answer:
A. A 14% return on investment will require the Division to produce income of $2,100,000 ($15,000,000 x 14%). If X = selling
price, then:
120,000X - (120,000 x $80) - $3,900,000 = $2,100,000
120,000X - $9,600,000 - $3,900,000 = $2,100,000
120,000X = $15,600,000
X = $130

B. If X = fixed cost, then:


[($132 - $80) x 120,000] - X - ($15,000,000 x 16%) = $200,000
$6,240,000 - X - $2,400,000 = $200,000
X = $3,640,000

To achieve her promotion, Deborah must reduce fixed costs by $260,000 ($3,900,000 - $3,640,000).

Basic Transfer Pricing: General Rule

71. Bronx Corporation's Gauge Division manufactures and sells product no. 24, which is used in refrigeration systems. Per-unit
variable manufacturing and selling costs amount to $20 and $5, respectively. The Division can sell this item to external
domestic customers for $36 or, alternatively, transfer the product to the company's Refrigeration Division. Refrigeration is
currently purchasing a similar unit from Taiwan for $33. Assume use of the general transfer-pricing rule.

Required:
A. What is the most that the Refrigeration Division would be willing to pay the Gauge Division for one unit?
B. If Gauge had excess capacity, what transfer price would the Division's management set?
C. If Gauge had no excess capacity, what transfer price would the Division's management set?
D. Repeat part "C," assuming that Gauge was able to reduce the variable cost of internal transfers by $4 per unit.

LO: 6 Type: A

Answer:
A. Refrigeration would be willing to pay a maximum of $33, its current outside purchase price.

B. The general rule holds that the transfer price be set at the sum of outlay cost and opportunity cost. Thus, ($20 + $5) + $0
= $25.

C. In this case, the transfer price would amount to $36: ($20 + $5) + ($36 - $20 - $5).

D. The transfer price would be $32: ($20 + $5 - $4) + ($36 - $20 - $5).
E.
Basic Transfer Pricing

72. Gamma Division of Vaughn Corporation produces electric motors, 20% of which are sold to Vaughan's Omega Division and
80% to outside customers. Vaughn treats its divisions as profit centers and allows division managers to choose whether to
sell to or buy from internal divisions. Corporate policy requires that all interdivisional sales and purchases be transferred at
variable cost. Gamma Division's estimated sales and standard cost data for the year ended December 31, based on a
capacity of 60,000 units, are as follows:

Omega Outsiders
Sales $ 660,000 $5,760,000
Less: Variable costs 660,000 2,640,000
Contribution margin $ ---- $3,120,000
Less: Fixed costs 175,000 900,000
Operating income (loss) $ (175,000) $2,220,000

Unit sales 12,000 48,000

Gamma has an opportunity to sell the 12,000 units shown above to an outside customer at $80 per unit. Omega can purchase
the units it needs from an outside supplier for $92 each.

Required:
A. Assuming that Gamma desires to maximize operating income, should it take on the new customer and discontinue sales
to Omega? Why? (Note: Answer this question from Gamma's perspective.)
B. Assume that Vaughn allows division managers to negotiate transfer prices. The managers agreed on a tentative price of
$80 per unit, to be reduced by an equal sharing of the additional Gamma income that results from the sale to Omega of
12,000 motors at $80 per unit. On the basis of this information, compute the company's new transfer price.

LO: 6, 7 Type: A

Answer:
A. Yes. Gamma is currently selling motors to Omega at a transfer price of $55 per unit ($660,000 ÷ 12,000 units). A price of
$80 to the new customer will increase Gamma Division's operating income by $300,000 [($80 - $55) x 12,000 units].

B. The additional operating income to Gamma is $300,000 [($80 - $55) x 12,000 units]. Splitting this amount equally results
in a new transfer price of $67.50, calculated as follows:

Transfer price before reduction $80.00


Less: Omega's per-unit share of additional income
[($300,000 x 50%) ÷ 12,000 units] 12.50
New transfer price $67.50

Transfer Pricing: Selling Internally or Externally

73. Sonoma Corporation is a multi-divisional company whose managers have been delegated full profit responsibility and
complete autonomy to accept or reject transfers from other divisions. Division X produces 2,000 units of a subassembly that
has a ready market. One of these subassemblies is currently used by Division Y for each final product manufactured, the
latter of which is sold to outsiders for $1,600. Y's sales during the current period amounted to 2,000 completed units. Division
X charges Division Y the $1,100 market price for the subassembly; variable costs are $850 and $600 for Divisions X and Y,
respectively.

The manager of Division Y feels that X should transfer the subassembly at a lower price because Y is currently unable to
make a profit.

Required:
A. Calculate the contribution margins (total dollars and per unit) of Divisions X and Y, as well as the company as a whole, if
transfers are made at market price.
B. Assume that conditions have changed and X can sell only 1,000 units in the market at $900 per unit. From the company's
perspective, should X transfer all 2,000 units to Y or sell 1,000 in the market and transfer the remainder? Note: Y's sales
would decrease to 1,000 units if the latter alternative is pursued.

LO: 6, 7 Type: A

Answer:
A. Division X Division Y Company
Sales at $1,600 $ 3,200,000 $ 3,200,000
Transfers at $1,100 $ 2,200,000 (2,200,000)
Less: Variable costs
at $850 (1,700,000)
at $600                    (1,200,000) (2,900,000)
Contribution margin $ 500,000 $ (200,000) $ 300,000

Unit contribution margin $ 250 $ (100) $ 150

B. Alternative no. 1: Transfer 2,000 units to Division Y:

Company sales (2,000 x $1,600) $3,200,000


Less: Variable costs [2,000 x $850) + (2,000 x $600)] 2,900,000
Contribution margin $ 300,000

Alternative no. 2: Sell 1,000 units in the open market and transfer 1,000 units to Y:

Company sales [(1,000 x $900) + (1,000 x $1,600)] $2,500,000


Less: Variable costs [(2,000 x $850) + (1,000 x $600)] 2,300,000
Contribution margin $ 200,000

Division X should transfer all 2,000 units to Division Y to produce an additional $100,000
($300,000 - $200,000) of contribution margin.

Transfer Pricing; Negotiation

74. Kendall Corporation has two divisions: Phoenix and Tucson. Phoenix currently sells a condenser to manufacturers of cooling
systems for $520 per unit. Variable costs amount to $380, and demand for this product currently exceeds the division's ability
to supply the marketplace.

Kendall is considering another use for the condenser, namely, integration into an enhanced refrigeration system that would be
made by Tucson. Related information about the refrigeration system follows.

Selling price of refrigeration system: $1,285


Additional variable manufacturing costs required: $820
Transfer price of condenser: $490

Top management is anxious to introduce the refrigeration system; however, unless the transfer is made, an introduction will
not be possible because of the difficulty of obtaining condensers in the quality and quantity desired. The company uses
responsibility accounting and ROI in measuring divisional performance, and awards bonuses to divisional management.

Required:
A. How would Phoenix's divisional manager likely react to the decision to transfer condensers to Tucson? Show
computations to support your answer.
B. How would Tucson's divisional management likely react to the $490 transfer price? Show computations to support your
answer.
C. Assume that a lower transfer price is desired. What parties should be involved in setting the new price?
D. From a contribution margin perspective, does Kendall benefit more if it sells the condensers externally or transfers the
condensers to Tucson? By how much?

LO: 6, 7 Type: A, N

Answer:
A. The Phoenix divisional manager will likely be opposed to the transfer. Currently, the division is selling
all the units it produces at $520 each. With transfers taking place at $490, Phoenix will suffer a $30
drop in sales revenue and profit on each unit that is sent to Tucson.

B. Although Tucson is receiving a $30 "price break" on each unit purchased from Phoenix, the $490
transfer price would probably be deemed too high. The reason: Tucson will lose $25 on each
refrigeration system produced and sold.

Sales revenue $1,285


Less: Variable manufacturing costs $820
Transfer price paid to Phoenix 490 1,310
Income (loss) $ (25)

C. Kendall uses a responsibility accounting system, awarding bonuses based on divisional performance.
The two divisional managers (or their representatives) should negotiate a mutually agreeable price.

D. Kendall would benefit more if it sells the condenser externally. Observe that the transfer price is
ignored in this evaluation—one that looks at the firm as a whole.

Produce Condenser; Produce Condenser;


Sell Externally Transfer; Sell Refrigeration
System
Sales revenue $520 $1,285
Less: Variable cost
$380; $380 + $820 380 1,200
Contribution margin $140 $ 85
were set. Given the difference in tax rates,
should Walker attempt to generate the majority of
Basic Transfer Pricing: Domestic and International its income in Pennsylvania or Germany? Why?
Implications

75. Walker, Inc., has a Pennsylvania-based division that LO: 6, 7 Type: RC, A
produces electronic components, with a very strong
domestic market for circuit no. 222. The variable Answer:
production cost is $140, and the division can sell its A. The manager would be unhappy, as the division
entire output for $190. Walker is subject to a 30% is being forced to take a "hit" of $30 per circuit
income tax rate. ($190 vs. $160).

Alternatively, the Pennsylvania division can ship the B. Pennsylvania: $160 - $140 = $20; $20 - ($20 x
circuit to a division that is located in Mississippi, to be 30%) = $14
used in the manufacture of a global positioning Mississippi: $380 - $10 - $120 - $160 = $90; $90
system (GPS). Information about the global - ($90 x 30%) = $63
positioning system and Mississippi's costs follow. Walker, Inc.: $14 + $63 = $77

Selling price: $380 C. Walker's income is unaffected, as the transfer


Circuit shipping and handling fees to price is a wash between the divisions. In other
Mississippi: $10 words, Pennsylvania's revenue is offset by
Labor, overhead, and additional material Mississippi's cost.
costs of GPS: $120
D. Pennsylvania: $180 - $140 = $40; $40 - ($40 x
Required: 30%) = $28
A. Assume that the transfer price for the circuit was Germany: $450 - $20 - $150 - $180 - ($180 x
$160. How would Pennsylvania's divisional 10%) = $82; $82 - ($82 x 45%) = $45.10
manager likely react to a corporate decision to Walker, Inc.: $28.00 + $45.10 = $73.10
transfer the circuits to Mississippi? Why?
B. Calculate Pennsylvania income, Mississippi E. Tax rates are lower in the U.S. than in Germany
income, and income for the company as a whole (30% vs. 45%). Thus, Walker would benefit if it
if the transfer took place at $160 per circuit. generated the majority of its income in
C. Assuming that transfers took place at a price Pennsylvania.
higher than $160, would the revised price
increase, decrease, or have no effect on Walker's Basic Transfer Pricing: International
income? Briefly explain.
D. Assume that Walker moved its GPS production 76. Cheney Corporation produces goods in the United
facility to a division located in Germany, which is States, to be sold by a separate division located in
subject to a 45% tax rate. The transfer took Italy. More specifically, the Italian division imports
place at $180. Shipping fees (absorbed by the units of product X34 from the U.S. and sells them for
overseas division) doubled to $20; the German $950 each. (Imports of similar goods sell for $850.)
division paid an import duty equal to 10% of the The Italian division is subject to a 40% tax rate
transfer price; and labor, overhead, and whereas the U.S. tax rate is only 30%. The
additional material costs were $150 per GPS. If manufacturing cost of product X34 in the United
the German selling price of the GPS amounted to States is $720. Furthermore, there is a 10% import
$450, calculate Pennsylvania income, German duty, computed on the transfer price, that will be paid
income, and income for Walker as a whole. by the Italian division and is deductible when
E. Suppose that U.S. and German tax authorities computing Italian income.
allowed some discretion in how transfer prices
Tax laws of the two countries allow transfer prices to figured that the firm will make $66.40 for each
be set at U.S. manufacturing cost or the selling prices unit transferred and used in the Swiss division's
of comparable imports in Italy. product. Rather than proceed with the transfer,
would Cunningham be better off to sell its goods
Required: domestically and allow the Swiss division to
Analyze the profitability of the U.S. division and the acquire part no. 54 in Switzerland? Show
Italian division to determine whether Cheney as a computations for both U.S. and Swiss operations
whole would be better off if transfers took place at (1) to support your answer.
U.S. manufacturing cost or (2) the selling price of C. Generally speaking, when tax rates differ
comparable imports. between countries, what income strategy should
a company use in setting its transfer prices? If
LO: 6, 7 Type: A the seller is in a low tax-rate country, what type of
price should it set? Why?
Answer:
Alternative no. 1: Transfer at U.S. manufacturing cost LO: 6, 7 Type: A, N
United States: $720 - $720 = $0
Italy: $950 - $720 - ($720 x 10%) = $158; $158 - Answer:
($158 x 40%) = $94.80 A. Courtesy of the shipping fee and import duty,
Cheney Corporation: $0 + $94.80 = $94.80 both of which can be avoided, it is cheaper to
purchase in Switzerland at $125. The shipping
Alternative no. 2: Transfer at selling price of fee and import duty raise the cost to acquire parts
comparable imports from the U.S. operation to $141 ($110 + $20 +
United States: $850 - $720 = $130; $130 - ($130 $11).
x 30%) = $91
Italy: $950 - $850 - ($850 x 10%) = $15; $15 - B. Yes. Cunningham will make $76 ($49 + $27) if
($15 x 40%) = $9 no transfer takes place and part no. 54 is sold in
Cheney Corporation: $91 + $9 = $100 the U.S.

Alternative no. 2 would be more profitable: $100.00 U.S. operation: $110 - $40 = $70; $70 -
vs. $94.80. ($70 x 30%) = $49
Swiss operation: $400 - $125 - $230 =
International Transfer Pricing; Analysis of Operations $45; $45 - ($45 x 40%) = $27

77. Cunningham, Inc., which produces electronic parts in C.When tax rates differ, companies should strive to
the United States, has a very strong local market for generate more income in low tax-rate countries,
part no. 54. The variable production cost is $40, and and vice versa. Thus, if the seller is in a low tax-
the company can sell its entire supply domestically for rate country, it should set a high transfer price
$110. The U.S. tax rate is 30%. (within allowed limits) to increase that country's
income.
Alternatively, Cunningham can ship the part to a DISCUSSION QUESTIONS
division that is located in Switzerland, to be used in a
product that the Swiss division will distribute Disadvantages of Return on Investment and Residual
throughout Europe. Information about the Swiss Income
product and the division's operating environment
follows. 78. Return on investment (ROI) and residual income (RI)
are popular measures of divisional performance. Like
Selling price of final product: $400 any measure, there are disadvantages or
Shipping fees to import part no. 54: $20 weaknesses that are an inherent part of these tools.
Labor, overhead, and additional material Briefly discuss a major weakness associated with
costs of final product: $230 each tool.
Import duties levied on part no. 54 (to be paid
by the Swiss division): 10% of transfer LO: 4 Type: RC
price
Swiss tax rate: 40% Answer:
Divisions with high ROIs apparently are very
Based on U.S. and Swiss tax laws, the company has successful. Top management would therefore like
established a transfer price for part no. 54 equal to these managers to aggressively seek additional
the U.S. market price. Assume that the Swiss division investment opportunities. However, the managers will
can obtain part no. 54 in Switzerland for $125. often reject opportunities that are attractive to the
company as a whole but that have a lower ROI than a
Required: division's current return.
A. If you were the head of the Swiss division, would
you be better off to conduct business with your Residual income (RI) does not have the same
U.S. division or buy part no. 54 locally? Why? weakness as described above for ROI. However, it is
Show computations. difficult to compare divisions of different sizes since RI
B. Cunningham's accounting department has is not a percentage.
Return on Investment: Asset Valuation Negotiated prices are typically consistent with
responsibility accounting; they generally do not
79. Return on investment (ROI) is a very popular tool to require intervention by top management and thus help
evaluate performance. The measurement of ROI is to preserve divisional autonomy. This is important
dependent, in part, on whether fixed assets are since the divisional structure is predicated on the
valued at acquisition cost or net book value. advantage of giving managers a high degree of
responsibility for their unit operations.
List several advantages of acquisition cost and net
book value as ways to value long-lived assets.

LO: 5 Type: RC

Answer:
Acquisition cost:
 The investment base is not affected
by the choice of an arbitrary depreciation method.
 The investment base does not
shrink over time because of accumulated
depreciation. This avoids misleading increases
in ROI or residual income.

Net book value:


 Consistency with the balance sheet
is maintained.
 Consistency with the definition of
income is maintained, as both the numerator and
denominator will reflect depreciation amounts.

General Transfer-Pricing Rule

80. One element of the general transfer-pricing rule is


opportunity cost. Briefly define the term "opportunity
cost" and then explain how it is computed for (1)
companies that have excess capacity and (2)
companies that have no excess capacity.

LO: 6 Type: RC

Answer:
Opportunity cost is the benefit forgone by taking a
particular action. Technically, companies that have
excess capacity are not forgoing profits from business
that has been rejected; thus, the opportunity cost is
zero. In contrast, if a transfer is made in a firm that
has no excess capacity, the firm will have to give up
profits on selected outside transactions. These profits
are measured by computing the contribution margin
on lost sales in external marketplaces.

Negotiated Transfer Prices

81. Although the general rule for transfer prices is the


outlay cost plus opportunity cost, many companies
instead use negotiated prices to price their goods and
services. When are negotiated transfer prices used?
Are such prices consistent or inconsistent with
responsibility accounting? Explain.

LO: 7 Type: RC, N

Answer:
Negotiated transfer prices can be used when no
market price exists for the transferred product or
when a buying division can obtain a cheaper price
outside of the organization.

You might also like